You are on page 1of 541
nd CONTROL AALS Sa AN Tata McGraw Hill Published by the Tata McGraw Hill Education Private Limited, 7 West Patel Nagar, New Delhi 110008, Control Systems Engineering, 2e Copyright © 2010, by Vijay Nicole Imprints Private Limited, No part of this publication may be reproduced or distributed in any form or by any means, electronic, mechanical, photocopying, recording, or otherwise or stored in a database or retrieval system without the prior written permission of the publishers and copyright holders. The program listings (if any) may be entered, stored and executed in a computer system, but they may not be reproduced for publication. This edition can be exported from Indie only by the publishers, ‘Tata McGraw Hill Education Private Limited. ISBN(13): 978-0-07-067193-5 ISBN(10): 0-07-067193-1 Information contained in this work has been obtained by publishers, from sources believed to be reliable. However, neither publishers nor copyright holders guarantee the accuracy or completeness of any information published he and neither publishers nor copyright holders shall be responsible for any errors, omissions, or damages arising out of use of this information. This work is published with the understanding that publishers and copyright holders are supplying information but are not attempting to render engineering or other professional services. If such services are required, the assistance of an appropriate professional should be sought. Laser Typeset at: Vijay Nicole Imprints Private Limited, Chennai - 600 042 Printed at: Saibonds Print Systems Pvt. Ltd, Chennai - 600 095 2QXLCRQFRBRDR, Contents Preface ait CuapTer 1 INTRODUCTION L1__ The History of Automatic Control 1.2 Control System Terminologies 1.3 _ System Representation 1.4 Regulator System 14.1 Example of Open Loop Regulator System 142 Example of Closed Loop Regulator System 15 Example of Servomechanism 1.6 Gomputer Gontrol System 1.6.1 Temperature Control System Using P.C. 1.6.2 Pressure Control System Using P.C. 1.7_Comparison of Open Loop and Closed Loop Systems L21 Advantages of Open Loop Systems 1.72 Disadvantages of Open Loop Systems 1.73 Advantages of Closed Loop Systems 1.74 Disadvantages of Closed Loop Systems 1.8 System Classification 1.8.1 Open and Closed Loop Systems 1.82 Linear and Nonlinear Systems 1.83 Time Invariant and Time Varying Systems 1.84 Continuous and Discrete Systems 185 Deterministic and Stochastic Systems 186 Lumped Parameter and Distributed Parameter Systems 1.8.7 SISO and MIMO Systems 1.8.8 System Classification Based on Components Used _ “Chapter Summary Beenie Cuaprer 2 Maruemarica, Mopeuine or Puysican Systems 2.1 Introduction 21 2.1.1 Mathematical Modeling~Transfer Function Model : 2.1 2.2 Mechanical Systems 2.2 Copyrighted material vim Control Systems Engineering 2.2.1 Mechanical Translational Systems 23 2.2.2 Rotational System 23 22.3 Dynamic Equations of Mechanical Translational System 23 2.2.4 The Step by Step Procedure 26 23 Lever and Gear Arrangements 2.17 ‘on Model for Elecrical Nenwark 2.9 2.5 Transfer Function of Separately Excited D.C. Generator 2.26 fe 9 stiaehawat C DOM 297 2.6.1 __No Load Transfer Function neh eae Condition 27 Transfer Function of Field C: 4DG.1 2.8 Armature Controlled and Field Controlled. D.C. Motor-Comparison 2.30 Chapter Stmmary . 2.40 Bere Cuarrer 3 ExectricaL ANALOGUE ‘B.1__Inroduction’ : al 3.2 Force-Voltage Analogy 3.1 3.3 Force-Current Analogy 32 (Chapter Summary 3.40 Exerciv . 3.40 Cuaprer 4 Brock Diacram RepucTION TECHNIQUE AND SIGNAL FLow Grapu 4.1 Introduction 4) 42° Block Diagram Representation of a System 42 4.3 Basic Connections for Blocks 3 CG G " z 4.3.2 Parallel Connection 42 © 44 BlockDizgram Algebra 44 4.5 Multiple Input System 4.19 4.6 Advantages and Disadvantages of Block Diagram Representation 4.29 4.6.1 Advantages 4.29 46.2 Disadvantages 4.29 4.7 Signal Flow Graphs 4.30 4.8 Definitions of Basic Terms for Signal Flow Graph 4.30 4.9 Basic Rules of Signal Flow Graph 431 Copyrighted mat Contents m vii 4.10 _ Basic Conneation for Signal Flow Graph 431 4.11 Gain Formula For Signal Flow Graph (Mason's Rule) 432 4.12 _ Signal Flow Graph Conversion from Block Diagram 444 4.13 Application of Mason’s Gain Formula between Output Node and Non-input Nodes 4.66 Chapter Summary 4.68 Breereige 4 Cuarrers Time Response or Freppack Conrrou Systems” 5.1__Introduction 5.1 5:2 Second Order System Model 5.2 5.3 Steady State Error for Step Input 5.3 5.4 Steady State Error for a Ramp Input of @, Slope 5.4 ‘55 TDS as _ 5A 5.6 Transient Response (or Time Response or Dynamic Response) of First and Second Order Systems 82 5.7 Test Signals 5.8 5.71 Impulse Signal 58 5.22 Step Signal _ 5.9 5.73 ___Ramp Signal z 5.9 5.74 Acceleration Signal 5.10 5.8 Review of Partial Fraction Expansion 5.10 5.9 Laplace Transform Table 511 5.9.1 Initial Value Theorem 5.12 5.9.2 Final Value Theorem é 5.12 $9.3 Shift Theorem 4 8 5.10 _ Analytical Method of Determining the Residues 5.13 5.11 Graphical Method of Determining the Residues 5.16 5.12 ‘Transient Response of a First Order System 5.20 5.13 _ Performance Characteristics of First Order System 5.21 $13.1 Time Constant 521 5.13.2 Settling Time 5.22 5133 TimeDelay : 529 5.14 Transient Response of a Second Order System 5.22 "5.15 Step Response of 2 Second Order System 5.22 5.15.1 Underdamped Case ¢ <1 5.23 5.15.2 Critically Damped Case £ =1 5.24 5.153 Overdamped Case ¢ >1 5.25 5.16 _ Expression for the Overshoot M, 5.27 viii_m Control Systems Engineering 5.17__ Time Domain Specifications 5.121 __Overshoot 5172 Time Delay ¢ 5.1724 Rise Times, 5.175 Seuling Time? See aoe al een 5.12.7 The Number of Oscillations before Settling Time is Reached 5.18 Impulse Response of a Second Order System 5.19 ‘Type and Order of Feedback System 5.19.1 Static Error Coefficients and Steady State Error 5.20 Steady State Error for Step Input 5.21 Steady State Error for Velocity Input 5.22 Steady State Error for Acceleration Input 5.23 The Generalized or Dynamic Error Coefficients 5.93.1 Alternative Method of Determinin, ic Error Constants 5.24 Steady State Error due to Disturbances 5.95 _ Effects of Adding Poles and Zeros to a Second Order System sp awoke : Sein af a 5.26 PID Controllers 5.27 — PD Connoller 5.28 PU Controller 5.29 Rate Feedback or Tachogenerator Feedback Control 5.20 Reset Contrallers 5.32 _ Effect of Pole Location on Transient Response 5.33 _ The Sensitivity Chapter Stanmary Exar Cuaprer 6 Frequency Domatn Aatysis or ConTrou SysTEMs 6.1__Introduction 6.2___ Advantages of Frequency Response Method 6.3 Frequency Response Plots 63.1 Polar Plot 63.2 Bode Plot 6: Magnitude versus Phase Shift Plot 6.4 — Step by Step Procedure to Draw Folar Plot 5.29 5.29 5.29 5.29 5.30 5.30 5.31 5.31 5.32 5.62 5.62 5.63 5.64 5.65 5.70 5.73 5.81 5.81 5.82 5.83 5.83 5.84 5.85, 5.86 5.89 5.90 5.92 5.96 5.96 Contents m ix 6.5 Sketching a Polar Plot 6.3 6.6 Mini onsen Phage Taasle Punch 7 Minimum Phase Ti Funct 6.8 All Pass Transfer Function CLO 69 Correlation between Transient and Frequency Response Methods 6.16 6.10 Bode Plot 6.20 6.11 __ Advantages of Bode Plot 6.20 6.12_General Rule to Drawa BodePlots 6.13 _ Step By Step Procedure to Draw the Bode Magnitude Plot 6.26 6.14 Bode Plot for Glosed Loop Second Order System 6.38 6.15 Determination of Transfer Function from Bode Magnitude Plot 6.41 6.16 Frequency Domain Specifications 6.48 6.16.1 Phase Margin and Gain Margin from Bode Plots 6.49 6.17 _ Obtaining Closed Loop Frequency Response from Open Loop Transfer Function— 6121 The Constant MCirdes 8 6122 ‘The Constant N Circles 62 6.18 Gain Adjustment for the Desired M, Using Constant M Circle 675 1.19 ination of K fc iver ichol’s 6.20 _ Forced Sinusoidal Response 6.80 Chapier Summary , 6.82 Beerge BS Cuaprer 7 Stasivrry or Linear Controt Systems 721__Introduction 2 72__ Relative Stability 12 72.1 Zero Input Stability 73 72.2 _ Asymptotic Stability : 73 72.3_Marginal Stability 73 73___ Impulse Response Funetion 74 74 Stability Definition via Impulse Response Function 14, 75 Characteristic Equation of Control System, 76 76 Stability Condition 76 7.7___Routh’s Stability Criterion 78 78___Factorising the Polynomial Using Routh-Hurwitz Method 741 729 Determination of RHP, LHP and Imaginary Roots from Routh’s Test 745 710 Determination of Marginal Value of K from Routh Test 77 x Ml Control Systems Engineering 7.1L Determination of Roots to the Right and Left of any Vertical Line other than the Origin 719 7.12__ Routh’s Test for System with Transportation Lag = 7.23 7.13 Conformal Mappings and the Principle of Arguments 724 714 The Principle of Arguments 730 7.15 Nyquist Stability Criterion 731 7.15.1 Nyquist Criterion-Statement 734 7.15.2 Advantages of Nyquist Stability Criterion 7.34 7.16 Procedure to Count WV, 734 717 Step by Step Procedure of Applying Nyquist Criterion 740 718 Nyquist Test for Systems with Imaginary Axis Poles of G()) 758 719 Determination of Stability from Bode Diagram 7.60 720 Poles of GH(3) on the jw Axis 7.62 721 Nyquist Test for System with Time Delay (Transportation Lag) : 7.80 Chapter Summary 782 Exercise 7.84 Cuarrer 8 Root Locus Tecunigue 81 Introduction Q1 es 5 Pre 83 To Determine the Gain Margin and Phase Margin from Root Locus 8.16 8.4 Root Locus with Addition of Pole: R55 8.5 — Root Locus with Addition of a Zero 8.56 8.6 Root Locus for K<0 (Inverse Root Locus) 8.58 Chapter Summary 8.66 Evan 8.67 Cuarrer 9 DESIGN OF CONTROL SYSTEMS IN TIME AND FREQUENCY Domains 9.1 Introduction 9.1 9.2 Cascade Compensators 9.2 9.2.1 Lead Compensator 92 9.2.2 Transfer Function of Lead Compensator 92 9.2.3 Design of Lead Compensator by Frequency Response Method 94 9.2.4 Design of Lead Compensator Based on Root Locus Technique 97 9.2.5 Design of Lag Compensator 99 9.2.6 Phase Lag Compensator Design by Frequency Response Method 9.10 9.2.7 Phase Lag Compensator Design Using Root Locus Technique 9.14 9.2.8 Design of Lag-Lead Compensator " 9.16 9.2.9 Lag-Lead Compensator Design Using Frequency Response Method __9.18 9.2.10 Design of Lag-Lead Network by Root Locus Method 9.22 Contents m xi 9.3. Design of Feedback Controller : Rate or Tachometer Feedback 9.26 94 Design of PID Controllers 9.27 9.4.1 PID Controller Design in the Frequency Domain’ 9.28 94.2 PID Controller Design by Root Locus Method 9.29 Chapter Summary 5 9.33 Exercise 9.34 Cuaprer 10 Conrrou System COMPONENTS 10.1 Introduction 101 10.2 Error Detectors 10.2 Synchros * 102 ‘Transmitter (Synchro Generator) 10.2 Synchro Control Transformer (G:T) (Receiver) 104 ‘Transfer Function of the Synchro 105 A Position Control System Using 2 Pair of Synchros as Error Detector 10.5 Potentiometer 106 Servo System Using Potentiometer as Error Detector . 107 10.3 Servomotors 108 10.8.1 ‘Two Phase AG. Servomotor ‘The Stepper Motor 10.11 Advantages of Stepper Motors 10.11 Disadvantages of Stepper Motors 10.12 ‘Applications of Stepper Motors 10.12 ‘Types of Stepper Motors 10.12 Stepper Motor—The Preliminaries 10.12 Variable Reluctance Stepper Motor 10.14 10.3.9 Rotor Movement 2S 103.10 _ Permanent Magnet Stepping Motor 10.15 10.3.11, Construction of a Two Stack Reluctance Type Stepper Motor 10.17 10.3.12 Haw the Rotor Moves? 2 0B 10.4 Hydraulic Servomotor 2 - 10.20 10.5 Pneumatic Controllers—Advantages 10.20 10.6 Pneumatic Proportional Gontroller—Flapper Valves 10.20 10.7 Gear Trains 10.21 10.8 ‘Tachogenerators 10.92 10.8.1 D.C. Tachogenerator 10.22 10.8.2 A.C. Tachogenerator 10.23 Chapter Summary 10.28 Boeri 2 Copyrighted material Preface Control Engineering which is an exciting and challenging field is a multi-disciplinary subject since many engineers from different disciplines design and implement control systems according to their requirements. An electrical engineer designs a control system to maintain constant output power, a mechanical engineer tries to control the position of a machine’s cutting tool, the chemical engineer develops the design strategy to maintain the speed of a conveyor belt. in all these designs, the engineers need to combine process and control engineering knowledge with design procedures, mathematical skills and analysis tools. The ultimate objective of every one of them is to implement controllers in real time systems. Such an implementation requires very strong mathematical foundation. Ihave written this book with the materials I have collected during my long experience, spanning about four decades of teaching control systems engineering to the undergraduate, postgraduate and doctoral level students in national level reputed institutions, The book in the present form is written to meet the requirements of undergraduate syllabus of Indian universities in general and Anna University in particular, for B.E. degree course in ECE, EEE, CSE, Eé&| and Bio-medical Engineering. Many interesting and advanced topics such as state space modeling and analysis, non-linear systems, digital control systems, Matlab, etc. could not be incorporated in this book as they all have been excluded from the syllabi of most of the universities. The organization of the book is as follows. In Chapter 1, a brief introduction about control system concept is given and various terminologies used are explained. The merits of feedback conceptare illustrated with suitable examples. ‘To understand and contro! complex systems, one must obtain a reasonably good mathematical model. Most of the physical systems are dynamic in nature and they are described by differential equations. In Chapter 2, the method of writing the differential equations for electrical, mechanical, hydraulic and pneumatic systems are explained. Finally, these systems are represented by the classical transfer function model, Mechanical, thermal, hydraulic, pneumatic and electro-mechanical systems can be represented and studied by their equivalent electrical circuits, which can be more easily constructed. In Chapter 3, force-voltage and force- current electrical analogous circuits are constructed for the given mechanical, translational and rotational systems. In different parts of control systems, the physical nature and configuration of different devices and sub-systems may be different. Iti therefore essential to develop a common language for co-ordinated analysis and design as a whole for such systems, This is possible by representing the systems in the block diagram form, irrespective of their physical nature and configuration. Chapter 4 deals with the representation of physical system in block diagram formand its reduction technique to express explicitly the relationship between the input and output variables, To overcome some of the limitations of block diagram respresentaion and tediousness in reduction, signal flow graph hes been suggested. The use of signal flow graph and the application of Mason’s gain formula are also described in this chapter. The ability to adjust transient and steady state performance is a distinet advantage of feedback control systems. To analyse and design a control system, its performance must be defined and measured. The parameters are so adjusted to get the desired response. For the dynamic systems, the performance specifications are given in terms of transient'response and steady state response. In Chapter 5, the transient and steady state responses for standard test signals such as impulse, step and ramp are obtained and the system performance characteristics studied. ‘The steady state response of a system for sinusoidal input is called frequency response of the system, when the frequency is varied for wide range. The frequency response can be used to investigate absolute xiv m Control Systems Engineering stability and relative stability, Important concepts such as phase margin, gain margin, bandwidth etc. are developed in Chapter 6 using polar plot, Bode plot and Nichol’s chart. The stability of feedback control system is very much essential in control systems design. An unstable closed loop system is ofno practical value. It is therefore essential to develop methods to help us analyse and design stable systems. In Chapter 7, Routh-Hurwitz stability criterion, a time domain approach and Nyquist stability criterion, a frequency domain method are discussed and a variety of examples presented. ‘The performance of feedback control system can be described in terms of the locations of the roots of the characteristic equation in the s-plane. By judicious selection of system parameters, the desired response of the closed loop system can be achieved. It is therefore useful to determine how the roots of the characteristic equation move around the s-plane as any one parameter of the system is varied. The graph of the locus of roots of the closed loop systems as one system parameter varies is known as root locus plot and this is discussed in Chapter 8. Determination of closed loop system stability, transient response, phase margin and gain margin are also explained here. & By adjusting the system parameters, the desired response obtained is not sufficient to obtain the desired performance, In such cases we must examine the scheme and obtain a new design that results in suitable systems that give suitable performance. In Chapter 9, such a scheme is described using a compensator. Lead, lag, lag-lead series compensators and PID controllers are designed using both frequency response method and root locus technique. A.closed loop system consists of an error detector, servoamplifier, actuator, gear, feedback transducers, etc. The detailed descriptions of these contorl systems components are given in Chapter 10. The notable features of this book include the following: 1. The syllabus content of control systems engineering for undergraduate level of most of the Indian universities has been well covered. 2. The organization of the chapters are sequential in nature. 3. Theoretical concepts have been supplementedireinforced by worked out numerical examples and presented in a graded manner. 4, Numerous problems that appeared in various university examinations have been solved in a lucid style. Many new problems not discussed earlier have been included. Alternative and simplified methods of solving numerical problems have been given wherever necessary. 5, A large number of problems, short and objective type questions with answers have been added in the exercise of each chapter. 6. The chapter summary given at the end of each chapter, would be helpful to students in summing up essential developments in the chapters. = take this opportunity to thank Shri. P K Madhavan, Managing Director, Vijay Nicole Imprints, Chennai for ‘coming forward to publish this book. { would like to thank Ms. Bindiya Balan, Jr. Acquisitions Editor and others in Vijay Nicole who were associated in the preparation of this book. | also thank the reviewers for their useful suggestions to reshape some of the material contents in the book. | would like to thank my wife Dr S Manimegalai, who was the source of inspiration while preparing this book. Finally 1 would like to dedicate this book to my beloved teachers, (late) Prof. P C Manoharan who introduced me to the fascinating theory of electricity and (late) Dr K L P Mishra who taught me the basics of what the research is about. Their untimely demise indeed had created a big vacuum in my academic life. They were extremely committed to their work. Suggestions and constructive criticism for the improvement of the book are always welcome. Dr S Palani GHAPTER Introduction In earlier days, people relied on muscle power or used animals to do work which needed lot of energy from them. Simple mechanical devices such as wheels and levers were later used to ease their work. Power from natural resources was also utilised to drive ships, wind mills and water wheels to supplement human energy. Since then slow and steady progress is made in the design and development of devices by means of which people.could utilise power ina convenient way. The early devices predominantly had manual control. Soon it was felt that manual control of machines and equipments ‘was very monotonous and control of certain machines was beyond the physical ability of human being. At this juncture the idea of controlling the machines and equipments by themselves was thought of and the first significant work came in the form of automatic control of centrifugal governor for speed control of steam engines by James Watt in 1769. In recent years automatic control systems have become increasingly important as a very strong problem solving discipline. There are a variety of computer systems that require sophisticated control strategies to achieve acceptable performance within an uncertain environment. They find increased applications in manufacturing, process industries, aircraft control, space research, navigation, biomedical engineering etc. Ina sense the advent of automatic control represents the second industrial revolution. Automatic control systems are physical systems which have dynamic behaviour. The study of control is not only: concerned with engineering applications but also extends to other areas like economics, business, political systems and so on. Control of adisease epidemic by inoculation, increased agricultural production by proper use of fertilisers, irrigation and insecticides, containment of forest fires set in by lightning or other natural means and wild life management for a designed ecological balance are examples of automatic control systems with man made controllers. Sensor equipped industrial robots, refrigeration and air conditioning systems, missile, aircraft, spacecraft, ship guidance, chemical process control, automatic washers, dryers and microwave ovens are examples of human devised control systems with human devised controllers. CHAPTER OBJECTIVES 1. Togivea brief introduction to Control Systems and define various terminologies used. 2. To classify the system and discuss about the advantages and disadvantages of open loop and closed loop systems with examples. 1.2m Control Systems Engineering ™ 1.1. THE HISTORY OF AUTOMATIC CONTROL The history behind the theory of automatic control is really a fascinating one. In the year 300 to 1 B.C. the automatic float mechanism was developed in ancient Greece and was used in water clocks and oil lamps. In 1572 the automatic temperature regulator system was used in Europe and in 168] automatic pressure regulators for steam boilers were introduced. These control systems were designed based on intuition and invention. The first significant contribution came from James Watt in 1769 who designed the centrifugal governor for automatic speed control of a steam engine. Only in 1868, a scientific approach was given by Maxwell who developed the mathematical model to control a steam engine. Henry Ford, in 1913 introduced the mechanised assembly machine for automobile production. The United States of America and USSR were two countries that contributed significantly to the development of automatic control theory eventhough they had different approaches. H. W. Bode in 1927 and H. Nyquist in 1932 who worked together in BELL Telephone System, USA, did pioneering works in the frequency domain approach. During the same period the Russians too showed their dominance in the time domain approach. However, during World War II, the engineers and scientists were subjected to tremendous pressure and it became necessary for them to design and construct automatic airplane pilots, gun positioning systems, radar antennas and other automatically controlled war weapons. They used feedback control theory and successfully completed the jobs. ‘The frequency domain approach dominated in the 1940s. However, in 1948, W. R. Evans developed the root locus technique which is a time domain approach. Frequency domain and root locus together formed the core of classical control theory which then gained prominence. They were soon found powerless when applied to multivariable and non-linear systems as they were in general not optimal. Analtenative to classical control theory was thought of for quite sometime and it wag successfully achieved in 1970. The system was described by » first order differential equations and the concept of state was introduced. When the system dynamic equations were converted into ‘state’ equations, the model so developed was called State Space Model. Based on this model, the very popular optimal control theory was developed by L. S. Pontryagin in the former Soviet Union and R. Bellman in the USA independently. This modem control theory was able to cope up with the stringent requirements on accuracy, weight, cost, space etc. In 1980, Robust controllers were developed which effectively rejected the internal and extemal disturbances and parameter variations. When the system complexity increased, control of system variables became more and more difficult. Digital computers were then intentionally introduced to monitor and control the systems and they were in use since 1970, Control techniques based on the theory of Artificial Neural Network, Fuzzy Logic, Genetic Algorithm or a combination of the above have been developed and used since 1990s along with digital computers. These techniques are called recent control techniques. Now, controllers are successfully designed using Ant Colony and Particle Swarm techniques with much improved performance. These Controllers are called intelligent controllers. Introduction m 1.3 @ 1.2 CONTROL SYSTEM TERMINOLOGIES The following terminologies, very frequently used in control system study are defined as follows: System A system is defined as a set (collection) of interconnected objects with a definite relationship between the objects and attributes. The interconnected components provide the desired function. Objects are the parts or components of a system. For example, switches, spring, masses, dash- pots, transistors, transformers, generators, inductors, capacitors, resistors etc. are the objects. Attributes are the characteristics of objects. Given below are some of the objects and their attributes. Objects Attributes Inductor Voltage across and current through inductor Mechanical spring | Spring tension and displacement Switches Speed of operation and state Motor Speed and torque Consider a simple electrical network such as shown in Fig. 1.1 which comprises of a resistor, an ‘inductor and a capacitor connected in series. The network is connected to an external voltage source V,, We call this as a system because there are three objects namely the resistor, the inductor and the capacitor and the attributes of these objects are V, and i, V,.and i and V,. and i respectively. The definite relationship between the object and the attributes are: V,=iR Fig. 1.1 Electrical System Control Systems Control systems are the systems that implement certain objectives. For example we need to control" the speed of an electric motor. The objective here is to maintain constant speed irrespective of load 1.4 m Cohtrol Systems Engineering variation, Accessories like potentiometer, tachogenerator ete., connected to the motor form the control system. The motor is the controlled object of the system and its speed is the attribute. Input Input to a system is an attribute of a system varied directly and independently by an operator. In the example cited in Fig. 1.1, voltage source V, is the input because it is the attribute of the system and could be varied independently by the operator. Output Output of a system is an attribute which varies as a result of variation in the input of the system and the output could be observed but not varied directly by the operator. In the example shown in Fig. 1.1, Vi» V, and V,. are taken as outputs. Variables Variables are attributes of asystem. These could be input variables or output variables or intermittent variables. In Fig. 1.1, V, is the input variable and V,, V, and V,, are the output variables. Current i is the intermittent variable. System Parameters Parameters are properties of the objects. In Fig. 1.1 R, Z and C are system parameters. Servomechanism Servomechanism represents the branch of feedback control system in which the output is either mechanical positioning or its time derivatives such as velocity or acceleration. Automatic positioning of anti-aircraft gun, radar antenna, rudders in ships are some examples of servomechanism. Servomechanism is meant as a faithful servant. Regulator System Regulator system is a system which maintains system output at a prescribed level. Speed cantrol of A.C. and D.C. motors, voltage control of A.C. and D.C. generators are examples of regulator systems. @ 1.3 SYSTEM REPRESENTATION All physical systems are described by mathematical relationships and they are represented in block diagrams. The block diagram contains input, output and control system components represented in blocks. Each block also contains the functional relationship between the output and the input. Consider the following equation. C(s) = Ris) G(s) (uy where R(s) is the Laplace transform of the input variable 7(?), C(s) is the Laplace transform of the output variable c(¢), and G(s) is the functional relationship between the output variable and the input variable. Equation (1.1) is represented as a block diagram in Fig, 1.2(a) and this is called an open loop system, Now consider the following equations: Introduction m 1.5 C(s) = Els) G(s) Els) = R(s) - C(s) H(s) (1.2) Forward path element Ris) ‘or power actuator cw Reference input variable oe [Output or controled (Objective or Control action) variable (Result) (a) Error detector Reference Fo) Ee co input +N = as) Quien Erroror (Control action) oF (actuating signal) Hs) |e Foodback element 0) Fig. 1.2 (a) Block Diagram of Open Loop System (b) Block Diagram of Closed Loop System Equation (1.2) is represented as a block diagram in Fig. 1.2(b). This system is called a closed loop or feedback or automatic control system. Fig 1.2(a) representing open loop system is converted into feedback control system by introducing a transducer (feedback element) and an error detector. When the output of the feedback element gets subtracted from the reference input, the system is called negative feedback control system. If the feedback output gets added to the reference input, the system is called positive feedback system. The following definitions which are useful for further study are given below. Open Loop System An open loop system is one in which the control action or actuating signal is independent of the output or the desired result as shown in Fig, 1.2(a). Closed Loop System A closed loop system or feedback control system is one in which the control action depends on the | output. Here there is a definite relationship between the output and input. By comparing/them the difference is obtained and used as a means for control (Fig. |.2(b)). 1.6 w Control Systems Engineering Control Action Control action refers to the actuating signal of the system. It is also called by the name error signal. Error Detector It is the comparator which compares the output of the feedback element with the reference input signal and the difference of these two signals appears as the output of the error detector. Feedback Element Feedback element converts the controlled variable to a form which is comparable to the reference input signal. For example, if the reference input is voltage and the controlled variable is speed, then the feedback element (called transducer) converts speed into voltage which is compared with the reference input signal which is also in the form of voltage. Examples of Open Loop System sy Qa | Fig. 13 Toilet Bowel Flushing System (Open Loop) In Fig. 1.3 the most commonly used toilet bowel flushing mechanism is shown. Here the water inlet valve is manually opened to fill water in the tank. As soon as the tank is full, the water flow Q,, is stopped manually by closing the inlet valve. If one forgets to close the input valve it would result in infinite flow of water into the tank and thus resulting in loss of water apart from flooding the area. This is an example of open loop system since correction of control of liquid level is not automatically done when the desired level is reached. Pressure Colbrats: Controller +] Actuator | Damper | Furnace: > Input Fig. 1.4 Furnace Pressure Control Sysiem (Open Loop) Fig. 1.4 depicts another example of an open loop control system represented as a block diagram. The pressure inside furnace is controlled by the rotational position of the damper. The controller is calibrated in terms of pressure and for a given setting adjusts the position of the damper through the actuator. Introduction m Examples of Closed Loop Systems | Dau Fig. 1.5 Toilet Bowel Flushing System (Closed Loop) Consider the toilet bowel mechanism shown in Fig. 1.3. This open loop system could be converted into a closed loop system by adding a float which acts as the feedback element as shown in Fig. |.5. ‘When the water level in the tank decreases, the float would come down allowing more water to flow. Once the desired level of water in the tank is reached, the float moves up pushing the inlet valve up and restricting the flow of water into the tank. This is an example of closed loop system, Pressure > Controller Acuator +—> Damper }—»| Fumace Transducer Fig. 1.6 Furnace Pressure Control System (Closed Loop) Consider the pressure control open loop system shown in Fig, 1.4. This also could be converted into a closed loop system by introducing a transducer in the feedback path as shown in Fig. 1.6. Here the pressure inside the furnace is measured by a pressure transducer whose output is proportional to the pressure applied and is fed back and compared to the calibrated input. The error detectorcompares these two and the error is applied to the controller. When the furnace pressure falls, the transducer output also falls. However, the error signal is increased when applied to the actuator through the controller. For higher error signal the rotational position of the damper is increased which increases the furnace pressure. This process is continuous and acts very fast till the desired pressure is restored inside the furnace. On the other hand, if the furnace pressure increases due to some obvious reason, the transducer output is also now increased which results in the fall of error signal for a negative feedback system. This ultimately decreases the rotational position of the damper thereby decreasing the furnace pressure. This automatic corrective measure continuously acts in the system until the desired pressure of the furnace is restored. Thus, the pressure of the furnace is always retained automatically to the set level. If the desired pressure is changed from one set value to another set value, the feedback path has to be opened and the reference input has to be changed. After getting the desired pressure, the 1.8 m Control Systems Engineering feedback path is closed and automatically corrects any change in pressure in the furnace. This is an example for a feedback system. Eyes > action steering >| ‘Muscular Car Fig. 1.7 Biological System Abiological system along with a machine (a motor car) is shown in Fig. 1.7.A driver is steering a car.9, is the desired position and @, is the actual position of the car in the road. The driver is supposed to run the car within the boundaries of the road. To avoid any collision with vehicles coming, from the opposite direction, he should have full control over 6,. By changing the steering of the wheel, he fixes 8, . If there is any deviation between 6, and 8, the eyes of the driver senses this as error ‘e’. The objective of the driver is to make the error ‘e’ zero. If it is not zero, the eyes send actuating signal to the brain. The brain sends signals to the muscle arms which contro! the steering wheel. Once the error “e” becomes zero, the muscle action stops. The eyes of the driver act as the error detector and this is an example of combination of biological and machine feedback control system. If the eyes are closed, the feedback loop is broken and the system becomes an open loop. ™ 1.4 REGULATOR SYSTEM @ 1.4.1 Example of Open Loop Regulator System ¥ (@) 1gRig + Armature Reaction Drops Va Fig. 1.8 (a) Regulator System (Open Loop) (b) Block Diagram of Open Loop regulator System Introduction m 1.9 A regulator system is one which maintains the system output at a prescribed level. In Fig, 1.8(a) a simple voltage control system is shown. The block diagram of this system is shown in Fig. 1.8(b). ‘The reference voltage V, is applied to the field winding of a D.C. generator which is run by a constant speed prime mover. The field excitation induces a voltage V across the generator terminals. When the generator is loaded, the terminal voltage V, falls down gradually as the load current J, increases. This fall in terminal voltage is due to /,R, and the armature reaction drops. As J, increases _ further and further, the terminal voltage V, would fall down very much due to severe armature reaction drop and the /,8, drop. There is no automatic correction in the system to maintain the terminal voltage constant. Therefore to compensate for these drops the potentiometer setting is adjusted thereby increasing the reference voltage , which in turn increases the induced voltage V,. As the load increases, potentiometer setting has to be correspondingly increased manually so that the armature reaction drop and the 1,8, drop is compensated. Hence, at all times the potentiometer setting has to be changed according to the load condition and so close inspection of the system becomes absolutely necessary. Here there is no feedback to the system. This is an example of open loop system. ™@ 1.4.2 Example of Closed Loop Regulator System fa) laRe iof(Ry+Ro} (b) Fig. 1.9 (a) Regulator System (Closed Loop) (6) Block Diagram of Closed Loop Regulator System Consider Fig. 1.9(a) where the system shown in Fig. 1.8(a) is converted into a closed loop by incorporating feedback arrangement. The block diagram of the system is shown in Fig 1.9(b). The only difference between Fig.'1.8(a) and Fig. 1.9(a) is that in Fig. 1.9(a) a part of the output voltage is fed back to the input side and it is compared with the reference input V,. Suppose the terminal 1.10 m Control Systems Engineering voltage /, isto be maintained at 220 volts from no load to full load. Initially no feedback is given and on no load, the reference voltage Vis adjusted such that V, is 220 volts. Now the required feedback is given to the system as shown in Fig. 1.9(a). When the generator is loaded, ,R, drop and the armature reaction drop will tend to increase resulting in decrease of V,. The error voltage, e-r-(n ge] A+R increases which in turn increases the excitation of the generator. Therefore yy is automatically increased, This feedback provides automatic correction and the regulation depends on the amount of feedback. @ 1.5 EXAMPLE OF SERVOMECHANISM Output Potentiomoter Fig. 1.10 Servomechanism A simple anti-aircraft positional servomechanism is shown in Fig. 1.10. 6, is the desired output position of the anti-aircraft gun which is driven by a separately excited D.C. motor whose armature is connected to a constant current source. The input potentiometer arm is given an angular displacement of 4, 10 move the gun through an angle 9, . The output potentiometer arm is connected with a gun, The gun in turn is connected through a worm gear. When the input potentiometer arm is rotated through 6, , and if the output is @, , an error voltage e proportional to (8, ~0,) appears. This error voltage is amplified and applied to the field winding of the D.C. motor. The motor moves in such a direction until 0, = 0,. Under this condition e= 0 and the motor stops rotating. This is an example of servomechanism, The output here is the position of the anti-aircraft gun. Automatic position control of radar antenna, positioning of rudders in ships are also examplesof servomechanism. {ntroduction m 1.11 @ 1.6 COMPUTER CONTROL SYSTEM Computers are very much in use in industries such as iron and steel, paper, petrochemical, pharmaceutical etc. Temperature control of a process, level control of a water column, flow control ina pipe line etc. ate all done by the use of personal computers (P.C.). Sometimes, various plants ip large scale industries like paper, iron and steel are controlled by a single computer which is located in acentre place. Monitoring and control of the plants which are located at far off distances are done by the computer using the appropriate software. Few applications of computer control system are briefly described below. ™ 1.6.1) Temperature Control System Using P.C. Pe Inlet ¥ Outlet TT - Temperature transmitter R - Rotameter Thy - Thyristor Fig. 1.11 Computer Controlled Temperature Control System In Fig. 1.1] the temperature control system using P.C. is shown, It is desired to keep the temperature of the water in the tank constant. Cold water is pumped into the water tank as and when required. ‘The water in the tank is heated by aheater coil. Hot water is taken from the outlet of the water tank. The temperature of the outlet water is sensed by the temperature transmitter (TT) and this information is fed into the computer. The computer software is so developed that when the outlet temperature of the liquid is less than the desired temperature the output signal from the computer changes the firing, angle of the thyristor which in turn is connected to the heater coil. This increases the flow of current to the heater coil and thus maintains the outlet liquid temperature constant. The reverse process happens when the temperature of the outlet liquid increases. Thus, monitoring and control of the process variable (temperature) is done by the personal computer. ™ 1.6.2 Pressure Control System Using P.C. The pressure inside a tank is to be maintained constant and at this pressure, air is to be supplied to several plants. Air is pumped into the tank through Control Valve (CV). A Pressure Transmitter (PT) is connected from the pressure tank and the output of the PT is fed into the P.C. A software is developed and a signal from the P.C. is taken, proportional to the pressure in the tank. This output signal from the P.C. is connected to a pressure converter (E/P) which in turn is connected to the control valve. When the pressure inside the tank falls, the P.C. senses this through the PT and sends out a voltage signal which is higher. This in turn makes the pressure output of E/P converter to a 1.12 m Control Systems Engineering higher value. This opens the control valve (CV) and allows more air to flow thereby increasing the pressure of air in the tank. The reverse process happens when the pressure level in the tank increases dueto less consumption in the output which is manually controlled by the Gate Valve (GV). Flow and level control monitoring could also be done in a similar way using P.C. P. PT | PL —_ eP - ov Det Outlet. PT. Pressure Transmitter EIP - Current to pressure Converter Pressure tank CV -Control Valve GV -Gate Valve inet Fig. 1.12 Computer Conirolled Pressure Control System @ 1.7 COMPARISON OF OPEN LOOP AND CLOSED LOOP SYSTEMS ™ 1.7.1 Advantages of Open Loop Systems 1, They are the simplest and most economical. 2. They are easier to build. 3. Generally they are stable. . 1.7.2 Disadvantages of Open Loop Systems |. They are less accurate and unreliable. They do not adopt to variation of environmental conditions or to external disturbances. @ 1.7.3 Advantages of Closed Loop Systems |. They are more accurate than the open lodp system owing to feedback. 2. Closed loop systems reject external disturbances and noise. 3. Because of feedback, the closed loop system acts very fast which is essential for certain class of control systems. 4. Because of increased bandwidth of the closed loop system the range of frequencies over which the system responds is increased 5. The sensitivity of the closed loop system for parameter variation is made small by increasing the feedback loop gain. They are less sensitive to noise, disturbances and environmental changes. 6. If the open loop system is unstable, it is possible to make the system stable by a feedback mechanism : Introduction @ 1.13 ™ 1.7.4 Disadvantages of Closed Loop Systems 1. Because of feedback, the system tries to correct itself which sometimes leads the system to oscillate. Closed loop systems are generally less stable than open loop systems. 2. Because of feedback, the overall gain of the system is reduced. 3. Because of additional components required for’ the construction of the closed loop system, the cost is higher and the system is more complex. @ 1.8 SYSTEM CLASSIFICATION Control systems are classified in a number of ways. They are classified as follows: ™ 1.8.1 Open and Closed Loop Systems A system in which the output has no effect on the control action is called an open loop system. These systems operate on a time basis. A system in which the contro! action depends on output is called a closed loop system or feedback control system. Here, the output or part of the output is compared with the reference input and the difference (called control action) is applied to the actuator which changes the output variable. @ 1.8.2 Linear and Non-Linear Systems According to the differential equation describing the dynamics of the system, a system is classified linear or non-linear. Linear system _ A linear systemis one in which the principle of superposition holds good. (ie) it obeys the principles of homogeneity and additivity. Thus if, is the output for r, input, and c, is the output for r, input then, for (a,r,+a,r,) input, the output would be a,c, +a,c,..Such systems are called linear systems and they are described by linear differential equations. Forexample the following differential equation represents a linear system, dx dk “Seat sora fe ag LO (13) In the above equation, x is the output variable, f()) is the input variable and a and b are system parameters. Non-linear system A non-linear system is one in which the system dynamic is described by non-linear differential equation. For example consider the folfowing differential equation. dx dk —> ta— +bsinx = f(t) . ita ft) (4) The above equation represents a non-linear system because of the term sin x. Here the principle of superposition is not satisfied. 1.14 m Control Systems Engineering @ 1.8.3 Time invariant and Time Varying Systems > If c(t) is the output of the system for r() input, and c(¢+ 7) is the output of the same system when the input is r( + T), then the system is called time invariant. Here T is the time by which the input is shifted. In such systems, the coefficients of the differential equation describing the system dynamics do not vary with time, Equation (1.3) represents linear time invariant system. Here the parameters aand bdo not vary with time. If a or b in equation (1.3) varies with time the system is called a time Varying system. —™ 1.8.4 Continuous and Discrete Systems In continuous (analogous) system, the plant or the process to be controlled and the actuator are all described by differential equations which could be linear or non-linear. The signals that appear in any part of the system are continuous in nature. On the other hand, in discrete (digital) control system the signals appear at discrete intervals of time and the components connected in the system respond only to discrete signals and the variables appear at discrete intervals. Such system dynamics are described by a difference equation which is given below. Wk = 2) + ay(k- 1) + by) = ul) . (1.5) In equation (1.5) »(¥) is the output variable, u(k) is the input variable and k= 0, 1, 2... . The above equation represents a second order discrete system. @ 1.8.5 Deterministic and Stochastic Systems If the systems have fixed parameters and inputs, they are called deterministic systems. Systems which have randomness in parameters or input are called stochastic systems. @ 1.8.6 Lumped Parameter and Distributed Parameter Systems Systems which have their parameters lumped are described by differential equations. Ifthe parameters are distributed, the dynamics of the systems are described by partial differential equations and such systems are called distributed parameter systems. The resistance, reactance and capacitance in an clectric power transmission line are distributed from sending to receiving ends. However, for simplicity they are treated as lumped parameters. @ 1.8.7 SISO and MIMO Systems If the single input variable is applied to the system and the output variable is also single then the system is called Single Input Single Output (SISO) system. On the other hand if the input and output variables are more than one, then the system is called multi input multi output (MIMO) system. ™@ 1.8.8 System Classification based on Components Used Depending upon the components connected in the system, they are classified as electrical, mechanical, electromechanical, hydraulic, pneumatic, thermal etc. Systems could also be classified by the combination of the above classifications. For example a system could be called a linear, time invariant, discrete, lumped parameter, SISO electrical system. Introduction @ 1.15 Example 1.1 For the simple voltage regulator system shown in Fig. 1.9(a), the gain of the amplifier 4= | amp/voltand the generator gain is 100 volts/field ampere. Find the reference voltage V, to have ano load voltage of 250 volts at the generator terminals with a feedback ratio of 0.2. Determine the change in the generator terminal voltage when the load current is increased by 20 amperes if the system is operated (a) open loop (b) closed loop with the feedback ratio of (1) 0.2 and (2) unity. Keep V,, constant. R, = 1 ohm: (A.U., May 2005) Solution Given : A=1; G= 100; ¥,=250; R, = 1 1,= The block diagram of the present scenario is as shown in Fig, 1.9(b). From this figure, (a) V, = EAG=(V,-02V) AG (For feedback ratio of 0.2). On no load, /, = 0 and therefore V,= V, v, =“ 40.27, ng HOH, = 22 0.24250 =525 volts. 100 When the system is open loop, V,+/, R,= AGV, Differentiating V, with respect to /, we get, AV, =—A/,R, = -20volts, Here A/, = 20A (given) (0) With feedback ratio of 0.2, V.+1,R,=(V_-0.2V)AG Differentiating V, with respect to /,, we get AV, +AI,R, = -0,2AV,AG Since AI, =20A -0.952 volts. When the feedback ratio is 1, From the above results it is evident that the closed loop system is more accurate than the open loop system in maintaining the constant output. As the feedback ratio is increased, the deviation of the output from the desired one is decreased. However, the increase in gain may diminish the margin of stability. : 3 Example 1.2 An amplidyne is employed to control the applied voltage of a motor as shown in Fig. 1.13(a). The amplidyne generates an output voltage proportional to the difference between 1.16 m Control Systems Engineering reference field current and control field current. The current amplification of the amplidyne is 250 and the generator generates an output voltage of 200 volts/field ampere. The armature resistance of generator is 1 ohm and that of the motor is 1.5 ohm. The control field resistance is 500 ohm and the motor develops a back emf of 200 volts. (a) If the motor takes a current of 20 amperes, determine the current required in the reference winding. (b) Ifthe back emf of the motor is increased to 220 volts, find the change in current in the motor armature winding. WWW 2 (cs) wena § + | 12, | 1500 0 Ve] (a) Vp = 200 ir V, k Ey}, -250 >{G = 200 Me > 13.5 > 4 Ney ws00 f+ (e) Fig. 1.13 Voltage Control using Amplidyne Solution (a) The block diagram of Fig. 1.13(a) is shown in Fig. 1.13(b). The Amplidyne is nothing but aD.C. generator whic has more than one field winding. From Fig. 1.13(b) 1,3.5+200 (vv, = 13.5) 200x250 Introduction m 1.17 pledge “500 500 [seem L oom (1, =20A) Bat * *=1"390%250 * 500 =(2.70+20)x2=45.4mA 45.4mA V3.1 +h, +i sae AG * 200x250 i, (b) = Differentiating V, with respect to /,, we get, = RAL +AN , AL, 200x250 ” 500 But, AV, = 20volts 103.5 AI, =-AV, =-20 ‘The armature current in the motor circuit is reduced by 0.193 A. In examples 1.1 and 1.2 only the steady state behaviour of the system is studied. To study the transients one has to take into account the effect of energy storing elements like inductance, moment of inertia etc. CHAPTER SUMMARY 1. The fascinating history of automatic control system is given very briefly. Various terminologies used in control system theory are defined. Open loop and closed loop system concepts are explained with examples and their merits and demerits are also discussed. 4. Typical examples of regulator system and servomechanism are given. Computer controlled systems such as pressure control and temperature control are explained. 6. A brief description of how systems are classified is given at the end. EXERCISE Give short answers in about 3 lines 1. State and explain the principles of superposition. Ic, is the output for’, input and ¢, is the output for r, input and when r, and r, are simultaneously applied to the same system, the output is (¢, + ¢,) for the input (r, + r,). This is the principle of superposition. 1.18 m Control Systems Engineering 2, How are feedback systems classified? Feedback systems are classified as a) Linear or non-linear b) Time variant or time invariant c) Lumped parameter or distributed parameter 4) Continuous or discrete 3. What is the effect of positive feedback on stability? With positive feedback all physically realisable systems become unstable. 4, Identify each term in; E(s) = R(s) - C(s) E(s) = Laplace transform of error variable R(s) = Laplace transform of input variable C(s) = Laplace transform of output variable. 5. Define the term Servomechanism. Give one example of a servo. Servomechanism is the branch of study of automatic control in which position or its time derivatives such as velocity or acceleration is the output. Position control of automatic gun, radar antenna, rudders in a ship are examples of servomechanism. 6. State the properties of a linear system. For linear system superposition theorem holds good. If ¢, and c, are output responses for r, and r, inputs, when applied separately, then due to (a7, + Br,) inputs the output response is (otc, + Be,) .In otherwords, for a linear system, properties of homogenuity and additivity are applicable. The stability is independent of the input. 7. Distinguish between open and closed loop control systems. Inan open loop system, the control action is independent of the output. In a closed loop system, the control action depends upon the output. Control action is the difference of the reference input and output or part of output, 8. What is a time invariant system? If c(¢) is the output of a given system for the input r(¢) and c(¢ + 7) is the output of the same system when the input is r (¢ + 7) then the system is called time invariant. Here T is the time by which the input is shifted or delayed. In such systems the coefficients of the differential equations describing the system do not vary with time. 9. Give an example of open and closed loop control systems. The action of a washing machine is an example of an open loop system. Soaking, washing and rinsing are done without measuring the cleanliness of the clothes which is the output. Controlling the temperature of a room using thermostat is an example of closed loop system. The thermostat puts the heating system on or off according to the set temperature by comparing it with a reference temperature. 10. What are the characteristics of feedback? ‘The characteristics of feedback are: System response is insensitive to external disturbances. Effect of parameter variation is minimum. Accuracy is high. Stability is less. However, unstable open loop system is made stable by feedback. System response is faster. Introduction m 1.19 « Define actuating signal. Actuating signal is the signal which is the difference between the reference signal and the feedback signal. 12. Give four example for open loop control system. Four examples of open loop control systems are: a) Traffic control by means of signals operated on a time basis b) Speed control of D.C. motor without feedback c) Control of pressure inside a furnace by the rotational position of a damper d) Voltage control of a D.C. generator without feedback. 13. Define a regulator system. A regulator system is one in which the output is maintained at a prescribed level. 14, What do you understand by control action? The “control action” refers to the actuating signal of the system. 15, What is the necessity of using an amplifier in a simple closed loop system? The actuating signal is comparatively a weak signal and therefore it becomes necessary to amplify this before it is applied to 2 power actuator. 16, Define ‘system’. A system is defined as a set (collection) of interconnected objects with a definite relationship between the objects and attributes. 17, Why negative feedback is used in control sytems? Positive feedback: has the tendency to make the system unstable. On the other hand, negative feedback makes even the unstable system stable. Hence, negative feedback is used in control systems. 18, Name three applications for feedback control systems. Automatic pressure control in a boiler, automatic voltage control in A.C. and D.C. generators and automatic positioning of anti-aircraft gun control are the three applications of feedback control systems. 19, Name three reasons for using feedback control systems and at least one reason for not using them. The closed loop system acts very fast. Accuracy is high. If the open loop system is unstable, it is possible, by proper feedback to make the closed loop system stable. The cost of feedback system is high. Choose the correct answer 1. A linear system is one which is described by a) Simultaneous equations b) Differential equation ¢) Differentiat equations obeying superposition theorem d) Equations with constant parameters 2. Consider the following equation Px dk . 42S 4sinx= fe mrtg tins SO 1.20 w Control Systems Engineering The above equation represents a a) Linear system g b) Non-linear system c) Time varying system d) Discrete system 3. Which of the following equations represent a linear time invariant system and linear time varying system respectively? dx dy , ax de. a) Gr tas tbe=s0) b) Gita th =f) d ak a’: ak ‘ 9) Feta S +aor= £0 &) Fra shsine= sO Ans: lc 2b 3.a&e Give long answers 1. For thé system shown in Fig. 1.14 find the battery voltage V,, to have the output terminal voltage V,= 250 volts if the load resistance XR; = 20 ohm. Find the terminal voltage y, when the load is thrown off. Draw the block diagram, If the load current is 20 ampere what should be the battery voltage to maintain V, = 250 volts? WW WWW Re R “ot | 1G) " Fig. 1.14 Voltage Control Using Feedback Given: R, = 200 ohm, R, = 100 ohm, R, = | ohm, volts volts G,=s00—°S_ ag, =200. fieldamp > fieldamp (Ans: 302.5 volts, 252.1 volts, 364 volts) 2. Describe any two types of open loop system and explain how they could be converted into closed loop system. 3. Describe with a neat diagram a position control system. * Describe with a neat diagram a regulator system. 5. Describe with a neat diagram a P.C. based level control system. - GHAPTER Mathematical Modeling of Physical Systems CHAPTER OBJECTIVES 1. Toidentify the system variables and write down the dynamic equations incorporating all these variables. 2. To identify the input and output variable of each component and determine its Transfer Function (TF). The T.F. which represents the functional relationship is put inside the block. 3. Todetermine the overall T-F. of the given system which is interconnected by morethan one component. ™@ 2.1 INTRODUCTION ‘System model represents certain property ofa system in graphical, pictorial, analytical or mathematical form. Photograph of a person is his pictorial model. Block diagram and signal flow graph are the graphical models. In example (1.1) and example (1.2) systems were represented under steady state, in mathematical form, ignoring dynamic aspect of the system. The effect of energy storing elements such as inductor and capacitor in electricl systems, mass and spring in mechanical system etc. are considered when one is interested in the bsehaviour of the system with respect to time. The mathematical model ofa dynamic system plays a vital role in the analysis and design of physical systems, Mathematical model of a system represents the dynamics of the system by a set of differential equations. The accuracy of system modeling depends on the individual’s ingenuity and itis to be noted that for a particular system, the model is not unique and could have many mathematical models. @ 21.1 Mathematical Modeling —Transfer Function Model ‘The mathematical description of the dynamic characteristics of a physical system is called mathematical modeling. The two most commonly used models are: 1. Transfer function model (Classical model) 2. State space model (Modern approach) In deriving the transfer function model, the following assumptions are made: 1. The system has Single Input and Single Output (SISO). 2. The environment does not affect the system parameters. 3. The system is linear and tisme invariant. 4. The system has zero initial conditions. 2.2 m Control Systems Engineering Consider a linear second order system which is described by the following linear differential equation. ax a Set 20, Za; = 02 (0) Q1) Here, x = System output variable, AO. = System input variable; ¢ and @, = System parameters. Taking Laplace transform on both sides of (2.1), the following equation is obtained. X(s) o =G(s)=——_“__ FG) 9) [F + 20,0403] In equation (2.2), G(s) is called transfer function. The transfer function is defined as the ratioof Laplace transform of the output variable to the Laplace transform of the input variable with all initial conditions being assumed to be zero. 2.2) It is to be noted here that the transfer 'function depends on the system parameters and it is independent of the input. Further it is noted that transfer function is not defined for non-linear and time varying systems. In equations (2.1) and (2.2), the time function x(t) and f(¢) are denoted by small letters and the corresponding Laplace transformed functions are denoted by capital letters as X(s) and F(s) respectively. This notation is strictly followed everywhere as a convention. Equation (2.2) is represented as a block diagram in Fig, 2.1. Fis) h Xis) (9? + 2Gons + oh) Fig. 2.1 Second Order System Representation Note Equation (2.2) represents a standard second order system where ¢ is called the damping factor and «, is called the natural frequency of oscillation of the system. We derive below the transfer function model for mechanical, electrical and electro-mechanical systems, The analysis part would be discussed later. @ 2.2 MECHANICAL SYSTEMS Mechanical systems are of two kinds: 1. Translational system which consists of mass, spring, dash-pot and lever arrangement. 2. Rotational system which consists of inertia, torsional spring, dash-pot and gear arrangement. The following symbols are used for these systems. Mathematical Modeling of Physical Systems m 2.3 @ 2.2.1 Mechanical Translational Systems 1. ft) = Applied force (input), (N). 20M = Mass, (Kg). 3B = Viscous friction coefficient of dash-pot, (N/m/sec) 4K = Spring stiffness constant, (N/m). “5.x = Displacement, (m), a 4 A 6 "a Linear velocity, (m/sec). e 1. a=£> = Linear acceleration, (m/sec’) B 2.2.2 Rotational System 1 7] = Applied torque (input), (N.m). 2 od = Moment of inertia, (Kg.m’). 3B = Rotational friction coefficient of dash-pot, (N.m)/(rad/sec). 4K = Spring stiffness constant, (N.m / rad). 5-6 = Angular displacement, (rad). ao 6. = = Angularvelocity, (rad/sec). do &O _ . % 77 Ge ~ Angular acceleration, (rad/sec*). @ 223 Dynamic Equations of Mechanical Translational System In mechanical translational system, mass is the element which stores kinetic energy, linear spring stores potential energy and dash-pot dissipates energy and provides damping to the system. When a force f(t) is applied to a mass M, it stores kinetic energy and develops an opposing force = uit i which acts in the direction opposite to the applied force as shown in Fig. 2.2(a). According to Newton’s law of motion, the sum of the applied forces is equal to the sum of the reaction forces which act opposite to that of the applied forces. The free body diagram for mass M is also shown in Fig. 2.2(b) which represents applied force and reaction force. The dash-pot and spring are represented in Fig. 2.2(c) and Fig. 2.2 (c) respectively. Their free body diagrams are shown in Fig. 2.2(d) and Fig. 2.2(f) respectively. From the free body diagrams, the following equations are easily obtained. * a) H wee 4 [8 BaD @ (b) 2.4 m Control Systems Engineering bex(t) fit), ay }——t ax 3 Be at © @ kK exh o raves —e—> (4 LJ Kx. e © Fig. 22. Free Body Diagrams for Mechanical Elements d?x : emt For mass M, fo=m a For dash-pot B, f= B= For linear spring K, I= Ke : 03) a s ‘ ss 4 : 7 H(t) nT zy, 5, fit) 7 Ot (4) = 2) @) (b) Fig. 2.3 Dash-pot not Connected to Reference Frame Now consider a dash-pot shown in Fig, 2.3(a). The free body diagram of Fig. 2.3(a) is shown in Fig. 2.3(b). At pointa,, the force ft) is applied. Point a, moves witha displacement x, and point a, moves with a displacement x,. The opposing force generated by the dash-pot is proportional to the velocity difference at a,. The final velocity is the velocity of the point under consideration and the initial velocity is the velocity at the other erid. Thus the opposing force developed by the dash-pot at point a, is proportional to the difference between final and initial velocity and the constant of proportionality is B. Thus the following equation is written at point a,. f= 84(3,-x) (24) a a ay ) rope 52 tee Ht) 5, 5a io KO ‘2) fa) ) Fig. 2.4 - Spring not Connected to Reference Frame Now consider Fig. 2.4(a) where the spring is subjected to force f()) at point a, which moves with a displacement x, and point a, moves with a displacement x,. The free-body diagtam of the spring at point a, is shown in Fig. 2.4(b). The opposing force developed by the spring at a, is proportional tothe difference of final displacement x, and initial displacement ,. Thus the following equation is written at point a, of the spring. Rb) = K(x, -x) 25) Mathematical Modeling of Physical Systems m 2.5 Note When one considers point a, in Fig, 2.3 or Fig. 2.4, and writes equation of motion ata,, the final displacement is to be considered as x, and the initial displacement as x,. ‘Now consider the mechanical system shown in Fig. 2.5(a) where all the three elements described above are combined and form a system. The free body diagram representing various forces at different points in the system is shown in Fig, 2.5(b). From the free body diagram we easily obtain the following equation of motion by equating the sum of the forces acting to the right to the sum of the forces acting to the left. Thus for Fig 2.5(b) we obtain the following equation. — M f——— 10) wx (b) (a) Fig. 2.5 Sine i Mechanical System and its Free Body Diagram f= uf Pip Boge a Taking Laplace transform on both dis ‘of the above equation we get the following transfer function. F(s) = Ms?X (s) + BsX (s)+KX(s) . AG), om F (5) (Msi+Bs+K) 26) Now consider the mechanical elements M, B and K connected in a different form as shown in Fig, 2.6(a). The free body diagrams are shown in Fig, 2.6(b) and Fig. 2.6(c). — tye BS -m) <4 agi 2s a m LK at 82 Mae Kix; -0) «J (b) () Fig. 2.6 Mechanical System with Dash-pot not Connected to Reference Frame 2.6 m@ Control Systems Engineering The following points are to be noted while drawing the free body diagram. sa 4 : 5s agua 1. The mass moves with displacement x and hence the opposing force generated by Mis M > which acts opposite to fl). 2. One end of dash-pot is connected to mass Mat a, and at that point the displacement is x only. 3. The other end of dash-pot is connected to one end of a spring at a,. At point a, the displacement "is different from that of a,. Label that displacement as x,.The reaction force generated by the dash-pot at point a, is proportional to the difference of final velocity and the initial velocity. At a& a, point a,, for the dash-pot, the final velocity is an and the initial velocity is 1 Hence ata, the reaction force due to B is aL(e-x) as shown in the freebody diagram of Fig. 2.6(b). 4, Now consider forces acting at point a,. The reaction forces are due to the dash-pot B and spring K. The final displacement for both of them is x, only. However, for B, the initial displacement is xand for K the initial displacement is zero since the left hand portion of the spring is connected to point a, which is called the reference point from where measurement is made. Hence, point a, has zero displacement. Therefore, at point a,, the force generated by the dash-pot B is B(final velocity-initial velocity). Thus for the dash-pot, d yo BT (a3) For the spring, F = K (Final displacement - Initial displacement) = K (x, - 0) = Kx, This is also represented in the free body diagram of Fig. 2.6(c). From the free body diagrams, the following dynamic equations are written. ax d f()=M Ss 846-5) BA (x, —x)+ Kx =0 F(x i Taking Laplace transform on both sides of the above equations, the following transfer function model is obtained. F (3) =(Ms? + Bs) X(s)— BsX,(s) (Bs +K)X,(s) = BX (s) Eliminating X,(s), the following transfer function is obtained. X(s)_ [8s+K] F(s) s[MBs?+ MKs+ BK] @ 22.4 The Step by Step Procedure The above examples illustrate how the transfer function model of a mechanical system is obtained by representing the system by its free body diagram and applying Newton’s law of motion. The following. step by step procedure is recommended. Mathematical Modeling of Physical Systems m 2.7 1. For each mass give one eel say x),1).. ete. The sepeins force due to mass M, with displacement x, is 2. The ends of the dtemnerts connected to ita reference fia ‘thave zero i pnabenisat, 3. The ends of the elements connected to a particular mass moves with the same displacement as that of the mass. 4. Identify the ends which are neither connected to the reference frame nor to any mass. These points move with displacements say y,, y,, ¥,, ete. Write down the differential equations to all masses and take Laplace transform. 6. Write down the differential equations for the points identified in step four. It is to be noted that while writing down the differential equation at a particular point, the force generated by a particular element is proportional to difference in displacement in the case of a spring and it is proportional to difference in velocity in the case of a dash-pot and acts opposite to the direction of motion. Here the final displacement or velocity is the one corresponding to the point under consideration. The initial displacement or velocity is due to the other end of the element. Now apply Newton’s law of motion. Take Laplace transform on both sides of the equations, 7. Consider the Laplace transform of all variables obtained in steps 5 and 6. Eliminate all the variables except the input variable and the output variable of interest by solving the simultaneous equations. 8. The transfer function model is obtained by the ratio of Laplace tranform of the output variable to the Laplace transform of the input variable. The above procedure is followed for mechanical rotational system also where linear displacement x is replaced by angular displacement @, mass M is replaced by inertia J and springs and dash-pots have their appropriate units. Example2.1 _ Foramechanical system shown in Fig. 2.7(a) find the transfer function G(s) = 2103). a ke a ae Kr Bj F->4 fry | be @ KY -%) 8 S05 ae d ay % fae Bd <— Kxo <«—_l ©) © Fig. 2.7 Mechanical System and Free Body Diagram of Example (2.1) 2.8 m Control Systems Engineering Solution 1. In Fig. 2.7(a), the junction point between K, and B, has to be identified with a variable for of the system. Let y be the displacement at point a, complete descripti 2. Atpointa, the following equation is written: d K,(y-3,) +8, qf) 0 (Free body diagram 2.7(b)). Taking Laplace transform on both sides of the above equation the following equation is obtained. [K,+2,s]¥ (s)-K.¥, (s)- BX, (s)=0 3. At pointa, the following equation is written: (Free body diagram 2.7(c)) ad By “(10 -»)+ Kalo -0)=0 ‘Taking Laptace transform on both sides, the following equtaion is obtained [K,+Bs]Xo(s)-Bs¥(s)=0 4. Eliminating ¥(s) in equations under steps 2 and 3, the following equation is obtained. x, (s) KBs [8(K.+K,)5+K,K,] Example 2.2 Find the transfer function relating displacement y and x for the mechanical system shown in Fig, 2.8(a). By a, 2y-»+— hat " on & ; « fe ye Y FOTIIN- bx Hey «— (a) (b) Fig. 2.8 Mechanical System and Free Body Diagram of Example (2.2) Solution 1, All the variables necessary for the complete description of the system are given. At point a,, the following equation is written. (Free body diagram 2.8(b)). d ad Bea ly—3)+ B geo K(y-0)=0 2. Taking Laplace transform on both sides of the above equation, the following equation is obtained. (B, +.8,)s¥(s)+ KY(s) = 8.5K (s) ¥(s)_ Bs X(s) [(Q+8,)s+K] Mathematical Modeling of Physical Systems. m 2.9 a Example 2.3. For the mechanical system shown in Fig, 2.9(a) find the transfer function (5) t—> xt use Ky a M pry ype Ke (x-y) <—] Ky <— (b) () Fig. 2.9 Mechanical System and Free Body Diagram of Example (2.3) Solution Accomplete description for this system is possible only by putting a variable y between the junction of K, and K, at point a,. 1, For mass M, the following equation is written: (Free body diagram 2.9(b)) ar FU) =MEE+K (9) ‘Taking Laplace transform on both sides of the above equation, the following equation is obtained. F(s)=[Ms? + K, ]X(s)-A,¥(s) 2. Atthe point a,, the following equation is written: (Free body diagram 2.9(¢)).. K,(y-x)+K,(y=0)=0 Taking Laplace transform on both sides, the following equation is obtained. KX (s)=(K,+K,)¥(3) 3. Eliminating ¥(s) in equations under steps | and 2, the following equation is obtained. X(s)_ [K+] F(s) [M(K,+K,)s*+ Example 2.4 For the mechanical system shown in Fig. 2.10(a) determine the 7.F. 2. s (A.U,, Dec. 2003) Solution The free body diagram of the mechanical system is shown in Fig. 2.10(b). From this diagram the following equations are obtained. Mi, +BY, + Ky, + KO,- y= LO 2.10 m Control Systems Engineering ove on fre " eK ee Keya-y) M® My M, ‘ | | n 7 + Me We @ Kalys = Ye) @ ) Fig. 2.10 Mechanical System and Free Body Diagram of Example (2.4) Taking Laplace transform on both sides, the following equation is obtained. [ Mis? + Bs +(K, + K,)]¥(3)— K.Y,(8) = F(s) For mass M,, M,¥, + K,(y,-— 9%) =0 Taking Laplace transform on both sides, the following equation is obtained. (Mys? + Ky )¥,(8) = KY, (3) Substituting for ¥,(s) in the previous equation, the following equation is obtained. ; IMs! +B +(K, gf M8] bye- Ae ? 4 a 2 ¥%G) M,M,s' + MBs" +{M,(K, +K,)+,K,} 5" BRS +R R= F(s) %)_ FOS) [Ms + M,Bs? + (MK, 4K) + MK} 9° + BK + KK, ] Xx, Example 2.5 For the mechanical system shown in Fig. 2.11 determine =?(s) Solution I. In this mechanical system there are two masses M, and M,, Hence, these masses are given displacements x, and x, respectively from the reference frame. 2. The ends of the two springs and the dash-pot are connected either to these two masses or to the reference frame. Hence, the output variables for these elements are x, and x, only. 3. The input variable is /{0) and is applied to mass M, Mathematical Modeling of Physical Systems m 2.11 4 : at Ky ke pm Ms | Ho} —> ty Se la) x x al < aa [> Kix1 2) > Ky0%9- 1) on — d |» o, 0) Me > Bp Sin Mh at & at |e v9 [> ar 0X) ) Fig. 2.11 Mechanical System and Free Body Diagram 5f Example (2.5) The free body diagram for mass M, is shown in Fig, 2:11(b) and for mass M, it is shown in Fig. 2.11(c). From Fig, 2.11(b) the following equation is obtained. S()=M, %) 27 ae, i 8B Ks +8 (The end of the spring K, connected to mass M, moves with a final displacement of x, and initial displacement of x,). Taking Laplace transform the followifig equation is obtained. F(s)=[Mys? + Bys +(K, # Ky) ]14(5)-K X69) 2.8) From Fig. 2.11(c) the following equation is obtained, Here the end of the spring K, connected to mass M, moves with a final displacement of x, and initial displacement of x,. Hence 2 uy, hae, Sisk, (x, (2.9) Taking Laplace transform on both sides of (2.9), the following equation is obtained. 1 (Mis? + Bs + K,) X(s) = KX, (8) (2.10) Eliminatating X,(s) from (2.10) and substituting in (2.8), the following relationship is obtained: xX, Ms*+Bs+K, Fl MM, 8" +(M\B, + M,B,)s° +1M,(K, +K,]+M,K, +8.) }s +{K,B,+(K,+K,)B,}s+K,K, 2.12 m Control Systems Engineering Example 2.6 For the mechanical system shown in Fig.2.12(a) find 20). [—>x 8 5 a y | rave» 1) M ‘ joao —| Ky A ——> ttt) ue. _| @ yA on ——Kix-y) © Kiy-9) f(t) uy — <4 }e— Ke bc1~x4) a at? my Px, d Mz fe— Mp 8, 204-8) [8a 30) a? at Ks (~~ ¥5) <4 (o) : J Kaxe () a By (45-9) Kyg<—— Bor Oa-xa) <— ey STA . x tp Xs #C 8) 5-4) Ke (44-1) << Ko xs-t2) << " © @ © Fig. 2.13 Mechanical System and Free Body Diagram of Example (2.7) Solution ‘1. The applied (input variable) force f(2) is first identified. 2. The displacement of mass M, and M, are x, and x, respectively. At points A, B and C there are variables. The ends are not connected to any mass or reference point. Let these variables be x,, x, and x, respectively. Now from the free body diagrams, just by inspection, the following differential equations are written: 3. For mass M,, (Fig. 2.13(b)) M, <3 Sa x )+B, fi =x,)+K,(%-x)=0 a d 4. For mass M,, (Fig. 2.13(c)) uy, +B Tm — x $K,(x,-%)+ Ky = f(0) > ae 2.14 m Control Systems Engineering 5. Atpointa, (Fig. 2.13(d)) 8, 6. AtpointB, (Fig. 2.13(e)) 24, 7. AtpointC, (Fig. 2.13() a5 Example2.8 Consider the mechanical system shown in Fig. 2.14(a). Identify the variables in the system and write down the differential equation incorporating all the variables. Solution ote M, meat M 3 vere —Sa}—| | 5. 5B POISE 4 Y ky My @ Ky 4 Me @ bo, pe d Je Keo (2-2) I 8s — (ea— 2 (a a Mi Kete-m) <4 My d a Bre 92) 85 O2-m) <4 J Kx to) fe) Le mxy >, ax | wf el 4 dt g My [> 10) Me 8, Gex,) 4 Ky (xs x) <— Seem | (a) fe) 4 4 8, 2% 3) 8 eee 5 %6 Kixs eI Ka Ge— 3) 0 ® Fig. 2.14 Mechanical System and Free Body Diagram of Example (2.8) Mathematical Modeling of Physical Systems m™ 2.15 The output variables for each mass M,, M,, M, and M, are identified as x,, x,, x, and x, respectively. Since the points A and B are free to move (these points are not connected to any mass or reference point) their displacements are identified as x, and x, respectively. For mass M,, the following equation is written. (Fig. 2.14(b)). dx gd & M, Era Ge ~45)4 By FEB TO —H)+K (HH) =0 For mass M,, the following equation is written. (Fig. 2.14(c)). dx, 34 d MG Baa) + Kae a) + Bay) =O For mass M,, the following equation is written. (Fig. 2.14(d)). d@’ M+ K (xa) = S10 For mass M, the following equation is written. (Fig. 2.14(€)). ax, ad d Maat Bag (a) + Bs GH) 20 At point A, the following equation is written. (Fig. 2.14(f). 0,2 (s5 =m) Kia, =0 At point B, the following equation is written. (Fig. 2.14(g)). d B a -x,)+K, (3, -%)=0 Example 2.9 Consider the mechanical system shown in Fig. 2.15(a). Identify the variables in the system and write down the differential equation incorporating all these variables. Solution 1 % The variables of masses M, and M, are identified as x, and x, respectively. Since points A and B are free to move and not connected to any mass or reference frame their displacements are identified as x, and x, ea For mass M,, the equation is written as: M, ised A(x, ~x,) = £(0 (from Fig. 2.15(b)). “dt For mass M,, the following equation is written: (Fig. 2.15(c)). @x,, d a d 4 TB A(x, n+ 8 E(m—n) +B, O(n) =0 2.16 m Control Systems Engineering fox, 3 [x ox, o ac uo “a J 82 Gi e-xs) at mM, > 1) My a a a J 8, Stem) ©) © a 8, {eax — raha - kp Kz (x3) <— Kyx4 (6) (e) Fig. 2.15 Mechanical System and Free Body Diagram of Example (2.9) 5. For point A, the following equation is written: (Fig. 2.15(d)). 4d B, ale =x,)+K,x, =0 6. For point B, the following equation is written. (Fig. 2.15(e)). “dt Example 2.10 For the mechanical rotational system shown in Fig. 2.16(a) identify the variables and write down the differential equations incorporating these variables. —x,)+K,x, =0 8, . a 8 Sate st batt oH, rye a (0-0) be bad agt (0,-0.4 res 2 Bek, 9p Ph ke@-8) By Ks, © 6 Fig. 2.16 Mechanical Rotational System and Free Body Diagram of Example (2.10) Mathematical Modeling of Physical Systems m 2.17 Solution 1. The angulardisplacement of the inertias J,, J, and J, are identified as 0,0, and 0, respectively. These variabes are enough for complete description of the system. 2. For,/,, the following differential equation is written. (Fig. 2.16(b)). a6, dé, T= dF + B74 K, (0-8) 3. For./,, the following differential equation is written. (Fig. 2.16(c)). J, +8, * dt 4. For J, the following differential equation is written. (Fig. 2.16(d)). +K,0, +K,(8,-6,)=0 By taking Laplace transform and solving the three simultaneous equations the transfer function + K,(8,-8,)+ K,(8,-8,)=0 & T ™ 2.3 LEVER AND GEAR ARRANGEMENTS (s) or £9 or Qovis obtained. Rh eb i i di +b" Tha x > ia) (b), Fig. 2.17 Lever Arrangement Ny Tt 6 Be Tat) Ne Fig. 2.18 Gear Arrangement A gearora lever is amechanical device that transmits energy from one part of the system to another part in such a way that force, torque, speed and displacement may be altered. These devices are used as matching devices to attain maximum power transfer. @ Control Systems Engineering A lever arrangement is shown in Fig. 2.17(a). For Fig. 2.1 7(a), the following equation is written. E,xl,-F, x1, =0 xy From Fig. 2.17(b) the equation is written as 7 = j— ty Consider the gear arrangement shown in Fig. 2.18. The primary gear has N, teeth and the secondary gear has N, teeth. The torque applied to the shaft of N, gear is 7\(s) and the torque available at the shaft of N, gear is T,(). The following equations are written for Fig. 2.18. 70, =7,8,; NG, =N,8, 10 _M, ae a Ny ‘The gear arrangement is similar to lever arrangement. The dynamic equations for rotational systems are written on similar lines for translational system. The linear displacement x metre is replaced by angular displacement @ radians, the force j{¢) N. is replaced by the torque 7(¢) N-m. Example 2.11 For the electro-mechanical system shown in Fig. 2.19(a) determine the transfer r function 1;(9); The polarised solenoid produces a magnetic force proportional to the current in the LO i coil as = K,()A where K, in N/A. The coil has resistance R and inductance L. (A.U., 2003 & 2005) jo x a ] tom . — L, A My i hs K, = N/A, | v x 8 My rm 8, Solenoid (@) Mathematical Modeling of Physical Systems m 2.19 a ¢ a wae 8S ¢ ! . L R- Mo ve ™ TT] oy ow Fy= Ke! — at tye (b) © @) be— 1; oe — “J ] 8 Fa Fe ) Fig. 2.19 Electromechanical System and Free Body Diagram of Example (2.11) Solution 1. Writing the loop equation for the solenoid, we get (Fig. 2.19(b)), v(t) = Ris ean, & at dt where K, is the back emf constant of the solenoid coil expressed in volts|mlsec. Since back emf constant is not given, it is assumed to be zero. Taking Laplace transform, the following equation is obtained. V(s)=(R+ Ls)(s) 2. The force generated by the solenoid is Fw Ks) =k, “(R+Ls) 3. Let F, be the resultant force at point A and it acts downwards. (Fig. 2.19(c)). 160 10-( Taking Laplace transform on both sides we get, KE — (Ms? +B,s)X(s) (2.13) 2.20 m Control Systems Engineering 4. The reaction force acting at point C is downwards and is given by (Fig. 2.19(d)). fom, 2. “ Sea D Ky F,=(MM,s? + Bs+K) ¥(s) 5. Taking momentat the pivot point B, the following equation is obtained. (Fig. 2.19(e)). Fly = Fly =0 (0) Fah = Fly KV (s) (R+ Ls) (2.14) 6. When the force is railing the solenoid down, the following equation is written using similar triangles. (Fig. 2.19(f)). bob Ms + Bs + K)¥(s) or —(Ms? + Bs) X (s)+ eV opgele nye? Substituting for x in equation (2.14) we sate following equation. Ks) R+Ls ¥(3)_ K, vs) (ata? +63) 2 (Mas B+ | = [me +B) ith (41,37 +aex))r) Example 2.12 Consider the electromechanical system shown in Fig. 2.20(a). Assuming that the electrical force acting on the core of mass Mis proportional to the current i in the inductor, obtain the transfer function of the system. The back emf constant of the coil is K, vim/sec. Solution f co) RLM, Ke, Ke “ ett) [ae 8 kx }—» poe at (b) (c} Fig. 2.20 Electromechanical System and Free Body Diagram of Example (2.12) Mathematical Modeling of Physical Systems m 2.21 1. Applying Kirchhoff’s voltage law to the electrical side of the system, we obtain the following equation. (Fig. 2.20(b)). e(teineta, ie ae dt 2. fy =K, iwhere f{t) is the force acting on the mass and K, is the proportionality constant. For the mechanical side of the system we conan the folowing equation. (Fig. 2.20(c)). f= ue coy os kx 3. Taking Laplace transform on both sides ofthe ‘above equations we get, E(s)=(R+Ls)I(s)+K, 5X(s) F(s)=K,I(s) F (s)=(Ms? + Bs+K).X(s) [E(s)-K, sX(9)] 1 Reb Substituting for F(s) and [(s) we get the following equation. X(s) _ K, E(s) (Rt Ls(Ms? +Bs+K)+K,K,s ™@ 2.4 TRANSFER FUNCTION MODEL FOR ELECTRICAL NETWORK Consider the electrical network shown in Fig, 2.21. Fig. 2.21 Electrical Network The following loop equation is written assuming the load current is zero. 1 a (t)= Rit Ritz Jie Taking Laplace transform on both sides of the above equation, the following equation is obtained I E(s) -[& +R, Sho The following equation is written for the output variable e,(1) 1 t)= Ryit+— lide es()= Re fi Taking Laplace transform on both sides of the above equation, the following equation is obtained. 5 )-[a+d}re 2.22 m Control Systems Engineering Eliminating the intermittent variable [(s), the relationship between the Laplace transforms of the output and the input variable is obtained. Thus (14+R,Cs) 14(R,+R)Cs} ‘The above result could be obtained in the following way also. ty 2 Eis) Ey Eo(s) Fig, 2.22 Electrical Network with Impedance Consider the network shown in Fig. 2.22 which is represented in the impedance form. For Fig. 2.22, the following equation is written. £,(s) = 1(s)[2,(5) +Z,(5)] , @ E,(s)=1(s)Z,(s) ) Dividing (b) by (a) the following equation is obtained. E,(s) 2,(s) & [2,()+2.(9] For the Fig. 2.21, Z()=R Z()=R+2 _ (140s) co -_ 4) (1+8,Cs) “@+Z,01 [H(R+R)cs Example 2.13 Determine the transfer function for the following network shown in Fig. 2.23. Hence c 9— +9 o—>s R, Re Fig. 2.23 Electrical Network of Example (2.13) Solution For Fig. 2.23, Z,(s) = Parallel combination of R, and C Mathematical Modeling of Physical Systems m 2.23 7 a (1+ RCs) R [I+ RCs] [(R+R)+ RRC] Example 2.14 Anelectrical circuit is shown in Fig. 2.24 (a). Obtain the transfer function relating the output voltage e,(¢) to the input voltage e(¢) in the form, Po K, (4+sT) (+57) E\s) 2yo] Fels) (®) (b) Fig. 2.24 Electrical Network of Example (2.11) Solution 1. The components R, and C, form one parallel combination and R, and C, form another parallel combination and are represented in Fig, 2.24(b). 2. The parallel combination of R, and C, has the following impedance function. R, Z)--—2— = TRG 3. The parallel combination of R, and C, has the following impedance function. Z,(s)= (+R,C,5) 4, From Fig. 2.24(b) the following relationship is easily obtained. 5 Zs68) 2 (9) =a) _ Bo ZG) +28) 2.24 w Control Systems Engineering 5. Substituting Z,(s) and Z,(s) in the above equation the following relationship is obtained. R Bis) = (1+ R,C,s) R+ RCs) ~ E, __R RT RFR FRRGStR, (+RCs) (1+R,C,9) _ RIF RCs) (RRG*RRC, | t+ won (+R) Bye, GD E, (457, where Ky = “2: 7. = RC, and 7, +R) (CR, Ry Example 2.15 For the network shown below, with a buffer amplifier, determine the T.F. Gt Go i T| aw [Tt Jf vy, RS v2 | ampifir | Vs eS Vo K Fig. 2.25. Electrical Network of Example (2.15) Solution From Fig, 2.25, the following equations are written. RGs Rl. MRGS) s % Ks where, Mathematical Modeling of Physical Systems m 2.25 Example 2.16 Write the simultaneous differential equations forthe translational mechanical system shown in Fig, 2.26(a) and hence find X\(s). € (A.U., Noy. 2004) M,=3 @ Men ss "dt ” Va, poix,-x) a tt [Lt ¥ ™, M, i % | | Oo) hw *e (b) {e) Fig. 2.26 Translational Mechanical System of Example (2.16) |. For the mass M, the following equation is written (Fig, 2.26(b)). fs M, hee Ley- xt Kx, =A(O Taking Laplace transform on both sides we get, (Ms? + Bs + K,)X,(s)— Bs X,(s) = F,(s) Substituting the numerical values and putting, F(s) = ‘© , the following equation is obtained. 5 (Bs? +454 5)X(9)- 45X05) = 2. For the mass M, steillesoneapatint is written. (Fig. 2. 2640). ae eae fos, =x) + Kx, = £0 2.26 m Control Systems Engineering Taking Laplace transform on both sides of the above equation, we get (M,8* + Bs + K;)X,(s)- Bs X(s) = F,(s) 6x5 30 Substitutingthe numerical values and putting F(s) = $35 = Ge 5 the following equation i obtained, 2 . 30 Gs" 445 +604, (8) ~484(8)= 5 3. Fromstep 1, X,(s) = glee +4545)X,(s)— 10] Substituting for X,(s) in n step 3, the following equation is seine from step 2. Ge Ae 462 fs +45+5)¥,(s)-10]-45¥,(s) == |" saree (s? 5) 120s? Gs? 445+ 6)[s@s? +454+5)X,(s)— 10} -168.X\(s) = +25 1205? (s? +25) _[10(s? +25) Gs? +45 +6)+ 1205" ] (s? +25) Ty py 2054 +403? +9305? + 10003 +1500) _| KO)= TF DOs 249) 43358 +s +30) ™@ 2.5 TRANSFER FUNCTION OF SEPARATELY EXCITED D.C. GENERATOR A D.C. generator is commonly used in control systems for power amplification. The field of the D.C. generator is separately excited by a constant voltage source and the generator is run by constant speed prime mover. Let the inductance and resistance of the field winding be L,and R, respectively and that of the armature be Z, and R,. The generator supplies a load Z,. The system is shown in Fig, 2.27(a). [s(3s?-+ 45 +6)(s? +45 -+5)—16s? ]X,(s) = (3s? +45 +6)10+ Re be [s I f ae TO a \ 2 z fa) | (14st) (b) Fig. 2.27 Simple D.C. Generator and its Block Diagram Mathematical Modeling of Physical Systems @ 2.27 ™ 2.5.1 No Load Transfer Function Under no load condition /, = 0 and the generator induced voltage is equal to the terminal voltage V, and under this condition the transfer function is £, (s)/£, . For the field circuit the following loop equation is written. F ay e)=i,R, +L, a (2.15) ‘Taking Laplace transform on both sides we get, Ep(s)=(Ry +8Ly EAs) (2.16) IFK, is the constant of the generator which gives no load voltage for unit field ampere expressed in volts per field ampere, = K, iO £,(s)= K/1,(8) My Substituting equation (2.17) in equation (2.16) we get, : B= S49) 5 gq. Hence the no load transfer function is, E(s)___K K : E, (R,+sl,)_ R,(I+s7,) (l¥sT,) (2.18) where K = K,{R, and T, = , /R, Which is the electrical time constant of the field coil. Equation (2.18) is represented in block diagram as shown in Fig. 2.27(b). Note Instead of a field winding, iffa permanent magnet is used to generate flux and if mechanical position is the input, the transfer function of the D.C. generator or D.C. tachogenerator is Fes) Ks where K, = volts/ rad./ sec. @ 2.6 TRANSFER FUNCTION OF ARMATURE CONTROLLED D.C. MOTOR The armature controlled D.C. servo motor under no load condition is shown in Fig. 2.28(a). Fig. 2.28(b) represents the motor when it is externally loaded by J,, the load moment of inertia and ,, theload friction. ; Bn bm bm T ow fr ‘0 = | L atl cy | LT mr t (@) (b) Fig. 2.28 Armature Controlled D.C. Motor without and with Load 2.28 m Control Systems Engineering The motor is called armature controlled motor since the error signal from the servo amplifier is applied to the armature of the motor. The following assumptions are made The field is given constant excitation. The back emf developed is directly proportional to speed. The torque developed is directly proportional to armature current. The coefficient of friction is constant at all speed. 5. The motor runs on no load. ae e ™@ 2.6.1 No Load Transfer Function Consider Fig. 2.28(a). The following equation is written e g(t) = ey +igRn thn ae (2.19) 6, where e, is the back emf of the motor which is proportional to speed and & = Ky ap where é A dO . fs K,= volis/rad/sec is a constant of the machine arid =>" is the speed of the motor. Taking Laplace transform on both sides of equation (2.19), the following equation is written. E,, (8) = p80 + 1g () Ry + Ln Sty (8) (2.20) ‘The torque developed by a D.C. motor is proportional to the product of armature current and field current. Since the excitation of the motor is kept constant, the torque developed by the motor can be written as, T,= Ki, where X, is the constant of the motor having the unit Newton-meter/armature ampere. Taking Laplace tranform for the above equation, the following equation is obtained. 7, (8)=K,1, (s) (2.21) This developed torque is used to drive the motor whose inertia is (Kilogram-meter’), and the load if any. In addition, this developed torque is also to be used to overcome the losses due to friction. (2.22) where / is called the coefficient of friction expressed in Newton-meter/rad/sec. Taking Laplace transform on both sides of equation (2.22), the following equation is obtained. 7, (5) = (Js? +f), (s) (2.23) But Tals)=7,(6) <.K,1,, (8) =(Js? + £56, (8) (2.24) Substituting equation (2.24) in equation (2.20) for I,(s), we obtain the following equation E, (s)= K,s6, Ete cg, +s) (2.25) K, aa You have either reached a page that is unavailable for viewing or reached your viewing limit for this book. aa You have either reached a page that is unavailable for viewing or reached your viewing limit for this book. ‘Mathematical Modeling of Physical Systems -m 2.31 5. Field controlled operation requires constant current source in the armature and the armature controlled operation requires constant voltage source in the field. Getting a constant voltage source is easier than a constant current source. Example2.17 For the system shown in Fig.2.30 determine the open loop transfer function 2) . How isthe system closed? Represent it in a block diagram. The system parameters are given below. Generator emf= 1000 volts/field ampere Motor torque constant = 0.3 newton metre/armature amp. Motor emf constant = | volt/rad/sec Total inertia of the load and motor =0.4 « 10‘ kg-m* Raz tog La=0.1H Peo Fd lee L eI = Pe (b) > GAs) >| Load > oo, (ce) Fig. 2.30 Simple Speed Control System The open loop speed control system is shown in Fig, 2.30(a). Voltage V, is generated by the getierator G and applied to the Motor M which is connected to the load. By adjusting the excitation of the generator, Vis increased or decreased. V, in turn increases or decreases the speed ©, of the motor. Let G(s) and G,(s) be the T-F. of generator and motor respectively. 2.32 mw Control Systems Engineering Solution 1. As already derived ft the D.C. generator, from equation (2.18). G()=FOezZ ey where-X, is a constant expressed in volts/field ampere. 2. (Fe~Kys)= (LR + hL.8) (i.e.) Applied voltage ~ back emf = Armature resistive drop + reactive drop. flo7Ks86) On Rast) where T, ak 3. Torque developed by the motor = K, J, where K, is the constant of the motor in newton-metre/ armature ampere. ‘The torque supplied by the motor is (/s? + /&) 6, where / is the combined inertia of the load and the motor and fs the friction constant. Equating the supplied torque to the developed torque we get, ; KLl)=( +5), [s@=c4] Stee) 1878, = Js, (f notgiven and hence neglected) 4. Substituting for V,, from step 1, we get K, KY, | To Kite | = ame oe aad . , pee Rik SLAC) 5. Rearranging the like terms, 145+ (14577) |" " RoR, (145m, (+57, 6. Hence the open loop transfer function is, G(s)=2(s) = % +57, )(1+ 57, )] Js 7. Given R, = 100 ohms, R,= 200 ohms, ’ K,= | volt/rad/sec. K,= 1000 volts/field ampere J 0.4 x 10 kg-m? K,= 0.3 N-m/armature ampere Mathematical Modeling of Physical Systems m 2.33 8. Substituting these values we get, a (s)2 1000%0.3 10x 200(1+0.01s)(1+0.025s)| 4x10" 5 + 10(1+0.01s 0.15 (1+0.0255)(4 x10" s?+4x10%s +0.03) |e ‘The system shown in Fig. 2.30(a) is closed as shown in Fig. 2.30(b) with tachogenerator feedback. The block diagram of the closed loop system is shown in Fig. 2.30(c). In the closed loop system, the error detector is potentiometer. In the block diagram, G,(s) is the transfer function of the D.C. generator, G,(s) that of the motor and G,(s) is the transfer function of the tachogenerator which generates voltage proportional to the speed with which it is run. Its transfer function is given by v, 7 & volis/rad/sec. Suppose itis desired to maintain the speed of D.C. motor constant irrespective ofthe load variation, The load torque increases when the speed of the motor tends to decrease, which results in decrease of V, The net error voltage (V, - V,) increases, resulting in increase of motor voltage. Now the speed of the motor increases and is brought to the original speed, The accuracy depends on the feedback. Similarly, when the load torque decreases, motor speed tends to increase. Correction is automatically made so that the motor applied voltage decreases which in turn decreases the speed of the motor and restores its original speed. Example 2.18 For the system shown in Fig. 2.31 (a), draw the block diagram. Indicate a block for each main component of the system, Given A = 20; 7,= 0.02 sec; R,= 1 ohm; R,= 100 ohms. The generator is driven at a constant speed so that its generated voltage is 100 i, volts. The motor is separately excited so that its counter emfis 1.0.0, . The tacho gives a voltage of 6 voltsat w, = 1004. The torque of the motor is 0.95 i, newton-metre. The motor and load have J = 6.8 kg-m?. Friction is negligible. ¢, is the reference voltage (a) Determine the transfer function of each block. (b) Determine C(s) / R(s). (c) If e,is 10 volts, what is the value of steady state actuating signal. Solution Given: A= 20; 7, = 0.02; R, = lohm; R= 100 ohm; K,= 100; K, = 1; K, = 6/100 = 0.06; J= 6.8; 2.34 @ Control Systems Engineering | ~ Ey A >| Gis) Loot Geis) Oo > fe, ~ &| “ a [= + —— ® Fig. 2.31 A Simple Speed Control System The block diagram of Fig. 2.31(a) is shown in Fig, 2.31(b). In Fig. 2.31(b) A is the amplifier gain, G,(s) is the transfer function of the D.C. generator, G,(s) that of the D.C. motor and 7(s) is the transfer of the tachogenerator. 1, To determine G,(s) If X, is the voltage generated for unit field current then, _ KE (s) E,( le taar) (From 2.18) K, i 100 R,(1+s7,) 100(1+s0.02) ~ (1¥0.028) 2. To determine G, (s) If K, is the torque developed for unit armature current, then ‘ K,(e,.-4) kK, TERI Rl ~K,@) This torque has to overcome Js@ (Friction is neglected). K, Jd, le ~ Kya) = a Ky, dey, KiKy0 R, dt R Taking Laplace transform on both sides of the equation we get, K, KK, Keto-[40 x = Jon or G,(s)2 1 0.95 E, ® (x, Bs) (0.95+6285) 1 © (147.165) 3. Todetermine Tis) The tachogenerator generates voltage proportional tothe speed w,. Hence, £, 7(s) = Le) - x, = © 0.06 veltshadisec 100 Mathematical Modeling of Physical Systems @ 2.35 20 (1+0.02s)(1+7.16s) 4. (a) To determine the T.F. of each block. The T.F. of each block is determined as G,(s),G,(s) and 7s). ) c(s) G(s) R(s). [1+GT(s)] C(s) _ 20 R(s) [(1+0.02s)(1+7.16s) +12] (c) The actuating signal £(s) from Fig.2.31(b) is derived as, (s)_ £(s) ’ 2 =TanG ) ay 12 (1+0.028)(1+7.16s) (1+0.025)(1+7.16s) £Q)= pea (1+7.163)+1.2] 5G) Let G(s) = AxG,(s)xG,(s)= 5. Ife, is a step function, then £,(s) = a for an input of 10 volts. The steady state actuating signal is given by £,, = Lt s£(s) (using final value theorem). 310(1+0.02s)(1+7.16s) ov s[(1+0.025)(1+7.16s)+1.2] 10 141. = 4.55 volts Example 2.19 Consider the hybrid mechanical system shown in the Fig. 2.32(a). Taking output as x, and taking the input 6, (a) Draw the block diagram showing each component of the system. Indicate the input and output of each block clearly (b) Determine the transfer function of each block diagram, TW k, 4% (a aa You have either reached a page that is unavailable for viewing or reached your viewing limit for this book. aa You have either reached a page that is unavailable for viewing or reached your viewing limit for this book. 2.38 @ Control Systems Engineering 8 8 4 sey L}- 4% way? oa 7H J b at Kaze) KO I} oe 1 dt (b) (e) Fig. 2.33 Solution 1. Let J, rotate with an angular position @. The free body diagram for J, and J, are shown in Fig.2.33(b) and Fig, 2.33(c) respectively. From Fig. 2.33(b), the following equation is written. aids AoA Ro, 0)=70) Taking Laplace transform on both sites we get, (d,s? + K)8(s)- KOs) =T(s) From Fig. 2.33(c) the following equation is written. 1 BeoBsKe- @)=0 Taking Laplace transform on both Sides we get, (d,s? + Bs +.K)O(s)=KO,(s) Substituting for @, (5) in the equations obtained is steps 2 and 3, the following equation is written. Joor=re 2 [iets EBTO_« K [yes" 4I,Bs' +(,K +, K)s? + KBs](5)=KT(s) Hence the requried T-F. is, K Sd + BP (K+ I,K)s 1 KB] Example 2.21 Consider the mechanical rotational system shown is Fig. 2.34(a). Identify the variable in the system so that the complete system dynamics could be described. Explain how the T-F. where @ isthe output variable and 7(¥) is the input variable is determined. 2 44 ar 2 e000 +s hi Ke @ Mathematical Modeling of Physical Systems m 2.99 & ®, 2 ey 2 4 gi 6182-87} a pr @ 0-0} ug ra a 0 LY Kio; 0) Hier ag o) © Fig. 2.34 1. The angular displacement 8, and 9, are to be given to inertias J, and J, respectively. This is enough to write down the dynamic equations of the complete system. 2. The free body diagram for J, is represented in Fig. 2.34(b). From this Figure, the following equation is written. 2, de, SFE FG -0)=T10 Taking Laplace transform on both sides, the following equation is obtained. (Js? + Bs+ K,)8(s)— K,8,(s)=7(s) 3. From the freebody diagram of J, (Fig.2.34(c)), the following equation is written. SpE + K (0, -0)*K(0,-0) =0 Taking Laplace transform on both sides we get, [vos? +(K, +K,)]8,(s)= KO, (8) +K,0(8) 4, For Fig. 2.34(d), the following equation is written. @o. , dé By a K,(@-,)=0 Taking Laplace transform on both sides of the above equation, we get, [ays?-+ Bys + K, ]O(s)=Ks0,(s) 5. Toestablish the relationship between 9, the output variable and 7, the input variable, we have to eliminate 0, and @, in the equations written in steps 2.3 and 4. 6. Combining the equations obtained in step 2 and step 3, eliminate 8, . 7. With the equation obtained in step 6 and equation obtained in step4, eliminate 6, and expressthe _ 8 = ratio (8). aa You have either reached a page that is unavailable for viewing or reached your viewing limit for this book. aa You have either reached a page that is unavailable for viewing or reached your viewing limit for this book. aa You have either reached a page that is unavailable for viewing or reached your viewing limit for this book. Mathematical Modeling of Physical Systems m 2.43 3. For the mechanical system shown in Fig. 2.37 write the differential equation. [> PE Hage Me ‘ My Bo -—I Pox ty) ———> M, B, Ky Fig. 2.37 Translational Mechanical System ax, dy d . f= M, prea a BN (4-5) b. ees =) +82 G,-n) + Kite -x,)=0 +82 (x, -m)4k, (y-x,)=0 4, For the mechanical system shown in Fig. 2.38 write the differential equation and hence find XU). 5 B — M,

M, RB, A Ry B, le My CUky | cpi/ky hoy ip a Vp RoB ty ° (iia) Cat/K Fig. 3.3(6) Force-Voltage Analogous Circuit Rove ea re any lau Fig. 3.3(c) Force-Curremt Analogous Circuit Solution Fores-Voltage Analogous Circuit 1. Masses M, and M, of the mechanical system shown in Fig. 3.3(a) will have v, and v, velocities. These are the only two dependent variables in the system, 2. Corresponding to v, and v, loop currents i, and i, are created. ‘The force /;(1) is applied on mass M,. The mass M,, spring K, and the dash-pot B, all have the same velocity v,. Hence, current i, flows through a series combination of ¢ (:)-» f(e, L+M,C ++ and 2 + 4, The force f(1) is applied on mass M,. The mass M,, spring K, and the dash-pot B, all have the same velocity v,. Hence, current i, flows through a series combination of e,—»/,(0), Ly > My. G aa and Ry > By 5. The velocity difference of (», ~¥:) is applied across the spring Kand the dash-pot B. Hence the current (j,i) should flow through the series combination of C—>-- and R-» 2. The entire circuit is shown in Fig. 3.3(b). For the mechanical system shown in Fig. 3.3(a) the following equations are written. M, Sisk, frrde eK, (0,- 04) + & fl, = )de= KO G.11) M, Fa ay ok, Pratt BO, -¥) 4K [(m-v)ar= LOs (3.12) aa You have either reached a page that is unavailable for viewing or reached your viewing limit for this book. aa You have either reached a page that is unavailable for viewing or reached your viewing limit for this book. aa You have either reached a page that is unavailable for viewing or reached your viewing limit for this book. Electrical Analogue 3.11 For the nodal system shown in Fig, 3.4(c) the following equations are written. At node e,, co Boz beat flee) y flee Jat = ic) (3.23) At node e,, o,f Tle e)dr+ fered 0 (3.24) At node e,, ‘ qe-at T fle -e)ar=0 (3.25) Equations (3.23), (3.24) and (3.25) of the electrical nodal system is analogous to equations (3.17), (3.18) and (3.19) respectively. Hence Fig. 3.4(c) the electrical system is analogous to the mechanical system shown in Fig, 3.4(a). Example 3.4 For the mechanical system shown in Fig. 3.5(a) draw the torque-voltage and torque-current electrical analogous circuit. on) ~) &) D4 tm mim, ae als ole Fig. 3.5(a) Mechanical Rotational System Ri>B, Lipp Rp Be lye in wg ig > wy C,50K, CK, Ry By yb) Y (ais) Fig. 3.5(8) Torque-Voltage Analogous Circuit Solution Torque-Voltage Analogous Circuit 1. The three inertias /,, J, and J, have angular velocities «@,,@, and @,. Corresponding to these velocities, the currents are marked as i,, i, and i,, The dash-pots B,, B, and B, also rotate with the velocities «,,«, and «, respectively. 2. The torque 7(1) is applied on inertia /,. The voltage source e(f)->7(), the inductance L,—> J, and resistance R,—> B, are all connected in series and current i, flows through this series combination. 3. The inductance L, > J, and resistance R,—» B, are connected in series and current i, flows through this since the velocity of J, and B, is «, aa You have either reached a page that is unavailable for viewing or reached your viewing limit for this book. aa You have either reached a page that is unavailable for viewing or reached your viewing limit for this book. aa You have either reached a page that is unavailable for viewing or reached your viewing limit for this book. Electrical Analogue @ 9.15 easy, 2? avy e+e scm, Aw BoM, Fig. 3.6(c) Force-Current Analogous Circuit Force—Current Analogous Circuit 1. In the mechanical system given, there are two velocities and corresponding to these velocities nodal points with voltage e, and e, are created as shown in Fig. 3.6(c). 2. The mass M,, spring K, and dash-pot B all move with velocity v, and the voltage is applied to M,. Hence one end of the current source i(t)~» f(¢),C, > Mj.) ++ and R at are connected to e,-> v,and their other ends are connected to the common point. 3. Mass M, moves with a velocity v,.Hence one end of C; > M; is connected to e,-> v, and its other end is connected to the common point. 4. The spring K, moves with a velocity difference of v,-v,). Hence L, ~» 1 is connected across the nodes e, and e, where the voltage difference is (e, -e,). The complete foce-current analogous circuit is shown in Fig. 3.6(c). For Fig, 3.6(c), the following nodal equations are written. Node ¢, det 4 1 Opes + are fla-a)ameo 6.39) Node le, L OG +z Me-a)ano (3.40) Equations (3.39) and (3.40) are analogous to equations (3.35) and (3.36) of the mechanical system shown in Fig. 3:6(a). Hence, it is proved that they are anologous to each other. Example 3.6 For the mechanical system shown in Fig. 3.7(a), draw the force-voltage and forcecurrent analogous electrical circuits. Fig. 3.7(@ Mechanical System aa You have either reached a page that is unavailable for viewing or reached your viewing limit for this book. aa You have either reached a page that is unavailable for viewing or reached your viewing limit for this book. aa You have either reached a page that is unavailable for viewing or reached your viewing limit for this book. Electrical Analogue m 3.19 Fig. 3.9{a) Mechanical System Fig. 3.9(b) Force-Voltage Analogous Circuit Force-Voltage Analogous Circuit 1. Mark v, as the velocity of Mand v,, the velocity of the junction point of the springs K, and K, 2. Corresponding to v,and v, current i, and i, are identified in the electrical circuit. Mmoves with a velocity v, and f(t) is connected to M. Hence in Fig. 3.9(b), a branch is created for the flow of current i,—> v, and in this branch e(t) > f(#) and L — M are connected in series. 3. The velocity difference (v, ~ v,) appears across K,. Hence a branch current (i, ~ i,) is taken and to this branch C; > a is connected 4. The velocity difference across K, is v, and the branch current i, created will flow through C, +L. The complete analogous circuit is shown in Fig. 3.9(b).. written for mass M. For the mechanical system shown in Fig. 3.9(a), the equatior a, MEK IO- 1, )dt = £0) G51) The following equation is written at the junction point of the springs X, and X,. K,J(, -y)dt+ K,fyyat=0 (3.52) For the electrical circuit the following equations are written. Loop e-L-C, di, 1 Lag, Mawar =e G.53) Loop C,-C, le Ly, ale “iat fia =0 3.54) Equations (3.53) and (3.54) of electrical circuit are analogous to equations (3.51) and (3.52) of mechanical system. Hence Fig. 3.9(b) is analogous to Fig. 3.9(a). Force-Current Analogous Circuit 1. For the velocities v, and v,, two nodal points at e, > v, and ¢,—> v, are created. 2. Mass M moves with velocity v, and force is applied on mass M. Hence, one end of the current source i(t)— f(t) and C > M are connected to the node e, and their other ends are connected to the common point. 3. The velocity across the spring K, is (v, - v,). Hence, , + Lis connected between the nodes K, e, and e,. aa You have either reached a page that is unavailable for viewing or reached your viewing limit for this book. aa You have either reached a page that is unavailable for viewing or reached your viewing limit for this book. aa You have either reached a page that is unavailable for viewing or reached your viewing limit for this book. Electrical Analogue m@ 3.23 = iy ig Re aBe RomsBs 4, ani i i » RB a R78, Coatk, bom ig LgaMz ig, bya aq) >) Fig. 3.11(6) Force-Voltage Analogous Circuit Solution Force-Voltage Analogous Circuit I Nv There are three masses M,, M, and M,. These masses move with velocities v,, v, and v, respectively. Current i, >» flows through £ > M,C > $a — B, through the voltage source e(r) > /(0), since they all move with v, velocity. From the branch current j,, two branches with i, and (i,—i,) currents are created. The current 4, ~ v, flows through L, > M2. The current (i,-i,) flows through R, > B;, since the velocity difference across B, is (v,-v,). Branch currents (i,~/,), (i,~i,) and i, are created. These currents satisfy Kirchhoff’s loop current law. Current i, flows through L, > M,. ‘The current [i,~/,]—> {»,-¥,] flows through R— B. The current [i,-i,]> [v;-»,] flows through R, > 8;. The complete force-voltage analogous circuit is shown in Fig. 3.11(b). The following equations for the mechanical system shown in Fig. 3.11(a) are written Mass M, iM, Bes By, +B, (vt BO, =H )+ KJ ydt= 0) G.63) Mass M, dy, My tBa (v4) BY, —%) =O 3.64) Mass M, M, Sosa, (¥ -¥,)+B(y, -v,) =0 (3.65) Pa fh load | con system of Fig. 3.11(b), the folliigeiguaicis ae wei: Loop e-R,-C-L,-R-R, eh ee ee nae Rie Rl J+ R(i-i)+cfid= ao G.66) aa You have either reached a page that is unavailable for viewing or reached your viewing limit for this book. aa You have either reached a page that is unavailable for viewing or reached your viewing limit for this book. aa You have either reached a page that is unavailable for viewing or reached your viewing limit for this book. Electrical Analogue m 3.27 3. To the ev, node, C, + M, is connected. 1 4. The velocity difference across spring K, is (¥,~ ,). Hence 4, ~» 5 is connected in between e, and e, nodes. 5. From the current source 12)» /(#) another current transformer with a turns ratio (a + & Bb : (a+ 5) is connected. In the secondary of this transformer, nodes ¢,->v, is created. L, > z 1 is connected to node e,. 6. Tothee, node, C, +> M, is connected. The complete force-current analogous circuit is shown in Fig. 3.12(0). (arbre) (a+b) en eg Louk, ® q g 1k M1 Faw, Ke envy le =O, em, {atd+c):a Fig. 3.12(c) Force-Current Analogous Circuit For the Fig. 3.12(c), equating the ampere turns of the secondary to the ampere turns of the primary, the following equation is written. oc org fedtar oe tft —2,)dta=i(t)(a+b+c) (3.76) For the currents at node e,, the following equation is written. ded - agai eat =0 (.77) Equations (3.76) and (3.77) of the electrical network are analogous to equations (3.72) and (3.73) of the mechanical system. Hence, the analogy is proved. Example 3,12 Consider the mechanical system shown in Fig. 3.13(a). The differential gear is , 8, ss described as 9, oP tee . Draw the torque—voltage and torque—current analogous circuits. aa You have either reached a page that is unavailable for viewing or reached your viewing limit for this book. aa You have either reached a page that is unavailable for viewing or reached your viewing limit for this book. aa You have either reached a page that is unavailable for viewing or reached your viewing limit for this book. Electrical Analogue m 3.31 Example 3.13 For the mechanical system shown in Fig. 3.14(a) draw the force—voltage and foree—current analogous circuits. Fig. 3.14(a) | Mechanical System LyaMp ip Liam, iy ett) 9 0, Fig. 3.14() Force-Voliage Analogous Circuit Solution Force-Voltage Analogous Circuit 1 ‘There should be five currents /,,i,, i, i,and /, corresponding to the velocities v,, v,, Vy, ¥, and v, respectively, for M,, M, and points A, B and C. Current i,->¥, should flow through L, > M, which is in series with the voltage source e(1). One end of the spring K, is connected to the mass M, and the other end is connected to the 1 reference frame. Hence, C, + through them is i,>v, is connected in series with L, + M, and the current passing Current i,>v, should pass through L, -» M,. Hence, a branch with current i, is created. The current /, flows through 4 M,. From the branch current i,, two branch currents i,->v, and (i,~i,)->(v,-v,) are created. Single current i, flows through C, +—-. The current (i,—i,) flows through R, -» By. aa You have either reached a page that is unavailable for viewing or reached your viewing limit for this book. aa You have either reached a page that is unavailable for viewing or reached your viewing limit for this book. aa You have either reached a page that is unavailable for viewing or reached your viewing limit for this book. Electrical Analogue m 3.35 CuK, i+ fT) ewony Fig. 3.15(b) Force-Voliage Analogous Circuit Single current i, should pass through L, -> M,. Hence a branch current i, is created in which L, is connected. The velocity difference across the dash-pot B, is v,. Hence R, is connected in series with L, and the current flow here is i,. From i, branch current, two branch currents i, and (i, — i.) are ereated. The velocity difference across the spring K, is v,, Hence in the branch where the currenti, flows C, > is connected. The velocity difference across the dash-pot B, is (v,- v,). Hence R, -> 8, is connected in the branch where the current flow is (i, ~i,). Again from the branch current i, two branch currents i, and (i, ~i,) are created. Single current i, should flow through L) > M,. Hence L, is connected in this branch. The velocity difference across the spring K, is v,. Hence C, oe is connected in series with L,. The velocity difference across the dash-pot B, and the spring K, is (v,-v,). Hence , — B, and ). a> ¢ are connected in series and included in the branch where the current flow is (i, - 2 aa You have either reached a page that is unavailable for viewing or reached your viewing limit for this book. aa You have either reached a page that is unavailable for viewing or reached your viewing limit for this book. aa You have either reached a page that is unavailable for viewing or reached your viewing limit for this book. Electrical Analogue m 3.39 > fit e(t) Fig. 3.16(0) Analogous Mechanical System Solution In the voltage-force analogue e(t) > /(0),R > B,C > z and LM, Let the current through the inductance Z be i,->v, current through the capacitance C be ( i,~i,)—>(v,-v,) and the current through the resistance R be ( i,~i,)-»(v,-v,) and the source current be i,>v,. Single current must be marked in the electrical circuit where the inductance L isconnected. Hence MM — L moves with a velocity v,. To one end of the mass M, the one end of the spring x 1 is connected where the - velocity is v,. Hence, the spring K moves with a velocity difference of ( v,-v,)-»(i,-i,). To the right end of the spring K, the dash-pot B-» R is connected. Since the current flowing through the resistance R is (i,—i,)->(v,—v,), the one end of the dash-pot is connected to the spring K and the force f(t) —» e(/) is applied to the other end of the dash-pot. The entire mechanical system is shown in Fig. 3.16(b). For the electrical network shown in Fig. 3.16(a), the following loop equations are written. Loop L-C alee, ; Leaf ~i,)dt=0 (3.120) oop C-R HIG -iar+ RG -i)=0 3.121) Loop e(1)-R Ri.) =e) (3.122) For the mechanical system shown in Fig. 3.16(b), the following equations are written. Mass M a, M+ K[O-y)dt=0 3.123) At point a K f(r. -)dt-+ BO, -%,)=0 (3.124) Atpoint 6 BE, =) = SO (3.125) , Equations (3.120), (3.121) and (3.122) of electrical network of Fig. 3.16(a) are analogous to equations (3.123) (3.124) and (3.125) respectively of mechanical system of Fig. 3.16(b). Hence these two systems are analogous. aa You have either reached a page that is unavailable for viewing or reached your viewing limit for this book. aa You have either reached a page that is unavailable for viewing or reached your viewing limit for this book. aa You have either reached a page that is unavailable for viewing or reached your viewing limit for this book. Electrical Anclogue m- 3.43 a) Capacitance b) Reciprocal of capacitance c) Inductance d) Reciprocal of inductance 14. The dash-pot in a mechanical system is analogous to the following in a loop system of an electric circuit. a) Resistance b) Conductance © c)_ Inductance d) ‘Capacitance 15. The dash-pot in a mechanical system is analogous to the following in a nodal system of an electric circuit. a) Resistance b) Conductance ¢) Inductance d) Capacitance 16. The lever arrangement which has length ratio of é in mechanical system is analogous to the following in the ioop system of an electric circuit. My, , Oy, a) A voltage transformer with turns ratio Ne b) A voltage transformer with tums ratio ae A 1 Ny, ©) A current tranformer with tums ratio a 4) A current transformer with tums ratio 2 \ N, 17. A gear arrangement which has teeth ratio NL ig analogous to the following in the nodal system of an electric circuit. y, : N, ) A voltage transformer with turns ratio {' _b) A voltage transformer with turns ratio 2 f 2 y, 18. Ina hydraulic system, the pressure drop is analogous to the following in the loop system of an electric circuit. a) Voltage drop Sb) Rate of voltage drop c) Current flow d) Rate of change of current flow 19. Ina hydraulic system, the rate of discharge is analogous to the following in the loop system of an electric circuit. N, r WN, c) A current transformer with turns ratio wy d) A current transformer with turns ratio’ — a) Voltage drop b) Rate of voltage drop ¢), Current flow d) Rate of change of current flow 20. The current flow in a loop sysem of an electric circuit is analogous to the following in a pneumatic system. a) Mass flow rate 6) Volumeric flowrate c) Pressure drop 4) Heat flow rate 21. The current flow in a loop system of an electric circuit is analogous to the following in a thermal system. . a) Mass flow rate b) Volumetric flow rate c) Temperature difference. d) Heat flow rate 22. Which of the following system does not have inertia? a) Mechanical sysem b) Hydraulic system c) Pneumatic system d) Thermal system An. i.b 2b 3.a 4c Re 6d 2d 8b %b We Ia 12b 13d 14a 15.b 16b 17.d 18a 1%¢ 20a 2d 224 aa You have either reached a page that is unavailable for viewing or reached your viewing limit for this book. aa You have either reached a page that is unavailable for viewing or reached your viewing limit for this book. aa You have either reached a page that is unavailable for viewing or reached your viewing limit for this book. Electrical Analogue ™@ 3.47 C+ uk hy Ly 4h PIO SOOGES tk ‘« fe ‘ boty Cot eT eee iy Fig. 3.26(a) Electrical Force-Voltoge Analogue of Figure 3.21 Ry 178, Fig. 3.26(6) Electrical Force-Current Analogue of Figure 3.21 aa You have either reached a page that is unavailable for viewing or reached your viewing limit for this book. aa You have either reached a page that is unavailable for viewing or reached your viewing limit for this book. aa You have either reached a page that is unavailable for viewing or reached your viewing limit for this book. Block Diagram Reduction Technique and Signal Flow Graph m 43 in parallel. The single equivalent block having the same terminal input and output replaces n blocks in parallel and is shown in Fig. 4.4. Here G,, = G,+G; +G,...+G, Input Output R oa c Fig. 4.4 Blocks in Parallel ™ 43.3 Feedback Connection Input ® Output “ Cc () Fig. 4.5 Block Diagram of Feedback System and its Equivalent Two blocks shown in Fig. 4.5(a) are said to be connected in feedback when the output of the first block is fed back to become the input to the second block. The output of the second block is compared with the reference input and the difference becomes the input to the first block. If the output of the second block aids the reference input, the feedback is known as positive feedback. If the output of the second block epposes the reference input, the feedback is known as negative feedback. Usually in the first block G is called the forward path transfer function or open loop transfer function and is represented by G(s). H is called feedback transfer function and is usually represented by H(s). GH{(s) is usually called loop transfer function. The single equivalent block for the above feedback system, for the same terminal input-output is shown in Fig. 4.5(b). The equivalent feedback transfer function is derived as follows: E=R£HC (4.1) C=GE aa You have either reached a page that is unavailable for viewing or reached your viewing limit for this book. aa You have either reached a page that is unavailable for viewing or reached your viewing limit for this book. aa You have either reached a page that is unavailable for viewing or reached your viewing limit for this book. Block Diagram Reduction Technique and Signal Flow Graph m 4.7 Successive block diagram reduction is shown in Fig, 4.6(b), (c), (d) and (e). In Fig. 4.6(a), the block containing G, has an unity feed-forward path. This is combined in a single block as (1+G,). Similarly the blocks with G, and G, are in the feed-forward mode and they are combined in a single block as (G, + G,), This is reprensented in Fig. 4.6(b). The blocks (1+G,) and (G,+G,) are in cascade and with this combination the negative feedback loop with a gain H, is connected. The cascade combination is combined and represented in a single block as (1+G,) (G,+G,). This combination with negative feedback gain H, is reduced and shown in Fig. 4.6(c¢). Q+G,)G;+G,) . % . 7 The block G, and 71Z(14c,\G.+G,)H,] M8 in cascade and they are simply multiplied and the resultant block is shown in Fig. 4.6(d).. 4, The two blocks in Fig. 4.6(d) are in feedback and the ratio C/R is obtained as given in Fig. 4.6(e). Example 4.2. For the block diagram shown in F IC GG + GG, + Gs) R_ [0+ 0+ GG, + GH} ++ GNG, +A} c a) find Solution aa You have either reached a page that is unavailable for viewing or reached your viewing limit for this book. aa You have either reached a page that is unavailable for viewing or reached your viewing limit for this book. aa You have either reached a page that is unavailable for viewing or reached your viewing limit for this book. Block Diagram Reduction Technique and Signal Flow Graph @ 4.11 Solution ) R. G Se ”| T58.GiHe oo «) GiG,G c 14G,GsH2+G,GH, Fig. 4.9 Block Diagram Reduction of Example 4.4 |. The input of #, block is shifted to point A. While shifting, the block has to be multiplied by G, to maintain the same output. This is shown in Fig. 4.9(b). 2. The blocks G, and H,G, are in feedback mode and is reduced using the formula 7——-> . This is TE shown in Fig. 4.9(c). 3. The block G,-and Team are in cascade and they are combined as Teen This combination is in negative feedback mode with the block H,. Using the formula <2. itis GG, GG ____ reduced 8 (14G.G,H,+GG,H,)° 4. This block is in cascade combination of the block G,. The successive reduction is shown in Fig. 4.9(b), 4.9(c) etc. e GEG, R (1+G,G,H,+6,G,H,)| aa You have either reached a page that is unavailable for viewing or reached your viewing limit for this book. aa You have either reached a page that is unavailable for viewing or reached your viewing limit for this book. aa You have either reached a page that is unavailable for viewing or reached your viewing limit for this book. Block Diagram Reduction Technique and Signal Flow Graph m 415 Solution 2 (eae }—t [we] HG (G2G5+G5) Gy [| [he © (B2Gs+Gs)G Gq HiGe (G2Go+G.)G, (a) (G2Ga+ Gs) G,Gy c (1 + Gig {Ga Hy ~ GaGgGqHo— GyGe] {e) Fig. 4.12 Block Diagram Reduction of Example 4.7 1. The input of 4, block is shifted from E, to E, point. Now the gain of the H, block is changed to H,G,, The block G,G, and G, are in feed-forward mode and are represented by its equivalent block with a gain of (G,G,+G,). This is in cascade with G, block and hence multiplied and represented with an equivalent block as shown in Fig. 4.12(b).. 2. The block HG, is shifted from point E, to point C, Now the blocks G, and (G,G,+G,)G, are in cascade and in the forward path. The forward path gain is (G,G,+G,)G,G,. aa You have either reached a page that is unavailable for viewing or reached your viewing limit for this book. aa You have either reached a page that is unavailable for viewing or reached your viewing limit for this book. aa You have either reached a page that is unavailable for viewing or reached your viewing limit for this book. Block Diagram Reduction Technique and Signal FlowGraph ® 4.19 and (G, +G,) are in cascade and they are combined and represented 3. The blocks -GG.m as shown in Fig. 4.14(c). 4. Fig. 4,14(c) is in negative feedback mode and is reduced as shown in Fig. 4.14 (d). a om GGG) 3s The GRrations RO) 10-GG,H,)+ GGG, +6) @ 4.5 MULTIPLE INPUT SYSTEM Sometimes the system is given more than onc input.The output response is found for cach input separately assuming other inputs are absent. Then the total output response is found by adding the responses due to individual inputs. This is illustrated by the following examples. Example 4.10 For the Fig, 4.15(a) find C(s). c ® S-E 2 ® [=] o) © (@ Fig. 4.15 Block Diagram of Multiple Input System L. If we assume R; =0, Fig, 4.15(a) reduces to Fig, 4.15(b). From Fig. 4.15(b) GG, OR THGG, 2. If we assume R,=0. Fig. 4.15(a) reduces to Fig, 4.15(c). R,G, ij see FromFig. 4.15) = Ate aa You have either reached a page that is unavailable for viewing or reached your viewing limit for this book. aa You have either reached a page that is unavailable for viewing or reached your viewing limit for this book. aa You have either reached a page that is unavailable for viewing or reached your viewing limit for this book. Block Diagram Reduction Technique and Signal Flow Graph 4.23 D G,G, Gy & "| TG, nGH, Hy i) D Gs Cy +) "| i GH, ~ GiGaH, 1+ Gy (9) D Galt+Gy) (14G3H))0+6,}+G ,G.G,H,) (h) Fig. 4.17 Block Diagram Reduction of Example 4.12 Assume the disturbance D=0. Successive block diagram reductions are shown in Fig. 4.17(b). Now the output is C, and is given by c= RG,G,G, [CFG 0+ GH) +GG,G,4,] Now let us assume R = 0. Successive block diagram reductions are shown in Fig. 4.17(e). Now the output is C, and is given by, . DG,(1+G,) (U+G))0+G,H,)+6,G,G,4,] Cr When R and D are simultaneously present, the output is C=C, +Cy GRG,G, + DU+G,)] [0+ G)0+G,H)+G,G,G,H.] aa You have either reached a page that is unavailable for viewing or reached your viewing limit for this book. aa You have either reached a page that is unavailable for viewing or reached your viewing limit for this book. aa You have either reached a page that is unavailable for viewing or reached your viewing limit for this book. Block Diagram Reduction Technique and Signal Flow Graph m 4.27 2. Assuming R,= 0, and ignoring C,, the input/output relationship at C,, is obtained as shown in Fig. 4.19(c). ~G,G.G.R, Gre 1-GG.G,G, 3. When R, and R, are simultaneously present, the output C, due to R, and R, is C,=C,' + G". = [AR =9G8)] (I-GG,GG,) 4. Assuming R, = 0, and ignoring C,, the input/output relationship at C, is represented as C)' as shown in Fig, 4.19(4).. GR, (1-G,G,G,G,) 5. Assuming R, = 0, and ignoring C,, the input/output relationship at C, is represented.as.C,” as shown in Fig, 4.19(e). -G,G,G,R, cr AGsGR, * (1-GG,6,G,) 6. When R, and R, are simultaneously present the output C, due to R, and R, is given by C= C+ . G(R, -G,G4R)] (I-GG,G,G,) Example 4.15 Using block diagram reduetion technique find the T.F. from each input to theoutput C for the system shown in Fig, 4.20(a). (A.U,, May 2005) X(s) Gy “QR -R-ELY Solution = {s,| = [33] 3 = af a; ror aa You have either reached a page that is unavailable for viewing or reached your viewing limit for this book. aa You have either reached a page that is unavailable for viewing or reached your viewing limit for this book. aa You have either reached a page that is unavailable for viewing or reached your viewing limit for this book. Block Diagram Reduction Technique and Signal Flow Graph m@ 4.31 @ 49 BASIC RULES OF SIGNAL FLOW GRAPH 1. Asignal always flows along the branch in the direction of the arrow. 2. When the signal flows through a branch, its strength gets multiplied by the branch gain. 3. The value of the signal at a node is the sum of all signals entering the node. 4. The value of the signal at a node is present on every branch leaving the node. ™ 410 BASIC CONNECTION FOR SIGNAL FLOW GRAPH Cascade Connection Three or more nodes are said to be connected in cascade if all the connected branches are pointing in the same direction and the cascade connection is reproduced by two terminal nodes and one equivalent branch gain which is equal to the product of all the branch gains of the cascade connections [Fig, 4.22(a)] o_o» 0 = o>» 0 x 8 De oH ab x @ = oe x an asd Xe . (b) s rs x % MM a Xe GH -H ) Fig. 4.22 (a) Cascade Connection of Signal Flow Graph (b) Parallel Connection of Signal Flow Graph (c) Feedback Connection of Signal Flow Graph Paratiel Connection If two nodes are connected by two or more branches such that one is the pure input node and the other is the pure output node, then the graph is called a parallel graph. The whole combinations are replaced by a single branch with a gain equal to the sum of branch gains of all the branches connecting to nodes. Refer Fig. 4.22(b). Feedback Connection The usual form of basic single I6op ~ve feedback system is represented by a signal flow graph shown in Fig. 4.22(c) The above signal flow graph is replaced by x, and x, with equivalent branch gain _@ __. The above formula is alternatively written in the form. 1+GH Forward path gai T= Loop ga aa You have either reached a page that is unavailable for viewing or reached your viewing limit for this book. aa You have either reached a page that is unavailable for viewing or reached your viewing limit for this book. aa You have either reached a page that is unavailable for viewing or reached your viewing limit for this book. Block Diagram Reduction Technige end Signal Flow Graph m 4.35 Solution ™ Xe % Xs Xs Xe a b e d e x Xp X Xs x % ° ° a 9 h Fig. 4.25 Signal Flow Graph of Example 4.18 aa You have either reached a page that is unavailable for viewing or reached your viewing limit for this book. aa You have either reached a page that is unavailable for viewing or reached your viewing limit for this book. aa You have either reached a page that is unavailable for viewing or reached your viewing limit for this book. Block Diagram Reduction Technique and Signal Flow Graph m 4.39 Solution ‘There is only one forward path. The forward path gain is, T= GGG, ‘S.No._| Feedback Loop LoopGain 1 L,=-GG,H, 2: Hmm L, =-G,H, 3. %-%—%a 1, =-G,H, Ta hm —% 1, =-G,G,G,H, 2 All the feedback rae are in touch with the forward path. Hence, A, Lb, Cs) dete ee letermine Fr Fig. 4.29 Signal Flow Graph of Example 4.22 aa You have either reached a page that is unavailable for viewing or reached your viewing limit for this book. aa You have either reached a page that is unavailable for viewing or reached your viewing limit for this book. aa You have either reached a page that is unavailable for viewing or reached your viewing limit for this book. Block Diagram Reduction Technique and Signal Flow Graph m 4.43 Example 4.25 Considering the signal flow graph shown in Fig. 4.32, find = xs Si G5 As, Xn W% Gs &, & “ @ " & Fig. 4.32 Signal Flow Graph of Example 4.25 Solution There is only one forward path with a branch gain of 7, =G,G,G,G,. The Mason’s gain formula becomes, Xe TAL x OA Identification of loops SI.No. Loop Leop Gaia WD aea ona b= GGG, 2. x, -%\—%, X57 Xs LDP ath the Lebthth G, Gy +GyG\, + GG,GG,G, + GGG,G,, +G,G,, + GGG, +Gy] Loops | & 7,2 & 7 and 5 & 7 are not touching. Hence the sum of gain product of these two non-touching loops is given below. - LL, = G,G,G,G, Lyby = GG Gy LL, =G,G,G, Hence, D Par = GGG + GyG.Gy + GG. aa You have either reached a page that is unavailable for viewing or reached your viewing limit for this book. aa You have either reached a page that is unavailable for viewing or reached your viewing limit for this book. aa You have either reached a page that is unavailable for viewing or reached your viewing limit for this book. Block Diagram Reduction Technique and Signal Flow Graph m 4.47 1. For the block diagram shown in Fig. 4.35(a), the variables £,, £,, £,, and E, are marked in addition to the input variable R and output variable C. 2. For each variable noted in step 1,a node is marked. To node £,, node & is connected with branch gain 1 and node £, is connected with branch gain ~H,. Thus at £,, the relationship £,= R—E,H, is satisfied. 3. To node E,, node C is connected with the branch gain -H, and node E, is connected with a branch gain 1. This satisfies the E, relationship E, = E,— CH, as in the block diagram. 4. Tonode E,, node £, is connected with the branch gain G, and node £, is connected with the branch gain I. This satisfies the relationship £, ~ E,G,~ as in the block diagram. 5. Tonode C, node E, is connected with a branch gain G,. This satisfies the relationship C = E,G, in the’block diagram. 6. Tonode £,, node C is connected with the branch gain H7, to satisfy the relationship £, = CH, as in the block diagram. The signal flow graph of Fig. 4.35(a) is shown in Fig. 4.35(b). There is only one forward path and forward path gain 7, =G,G,. From Fig, 4.35(b), Since there are no non-touching loops, A=1-(L,4+1,+L,)=1+G,H,+GG,H, +G,H,G,H, Since all the feedback loops are broken when removed from the forward path Ay=1 14G,H, 4G, Example 4.28 Convert the block diagram representation of a system as shown in Fig. 4.36(a) into a signal flow graph. Accordingly, find the .F. oH (A.U,, April 2004) “&e&) abil os) (a) aa You have either reached a page that is unavailable for viewing or reached your viewing limit for this book. aa You have either reached a page that is unavailable for viewing or reached your viewing limit for this book. aa You have either reached a page that is unavailable for viewing or reached your viewing limit for this book. Block Diagram Reduction Technique and Signal Flow Groph @ 4.51 Solution (b) Fig. 4.38 Block Diagram and Equivalent Signal Flow Graph of Example 4.30 The signals x, to x, are marked in the block diagram as shown in Fig. 4.38(a) just by inspection. The six variables are represented by six nodes and the functional relations by branches. The equivalent signal flow graph is constructed as shown in Fig. 4.38(b). Hdentification of Forward Paths S.No. Forward Path Forward Path Gain Lo] Rex-x-4 0 -C T, =GG.G, 2 | R-x,-x, he ‘The Mason Gain formula is c TA, +Tabe Calculation of A S.No. Loop Gain 1 1, =-G,0,G,H, 2. 3, 4 [xy C-yenny 5. xy-x,-C—% “Ly =-G,GjH, (L,+L, + by +L, +L) =1+G,G,G,H, +H, +G.G,H,-GH\H, + G.G,H, ze There are no two non-touching loops, since all the loops are in touch with each other 5)?,,=0 Hence, A =1+G,G,GH, + H,+G,G,H, -G,H,H, +G,G,H, . aa You have either reached a page that is unavailable for viewing or reached your viewing limit for this book. aa You have either reached a page that is unavailable for viewing or reached your viewing limit for this book. aa You have either reached a page that is unavailable for viewing or reached your viewing limit for this book. Block Diagram Reduction Technique and Signal Flow Graph m 4.55 Example 4.32 Using Mason’s gain formula, obtain C/R ratio of Fig. 4.40. Fig 440 Signal Flow Graph of Example 4.32 Solution 1, Identification of forward paths. S.No. Forward Pati Forward Path Gain lL. R-¥-%-4- GW T=6,G,G, | i Rx 7x -% 4 -C T,=G,GG, | 3 T= G,G,G, | 4. T,=G,G.G, 5. T,=-G,GH,GG, 6 T, =-G,GHGG, 2. Identification.of feedback loops S.No. | Feedback Loop Loop-Gain 1 L,=-G,H, 2 L, =-G.H; 3 X,-- — 1, =+GHGH, 3. Sum of individual loop gains DP ath thy = GH, -GH, +G\H,GA, 4. The loops | and 2 are not touching each other. Hence DP hh, = GUGM, aa You have either reached a page that is unavailable for viewing or reached your viewing limit for this book. aa You have either reached a page that is unavailable for viewing or reached your viewing limit for this book. aa You have either reached a page that is unavailable for viewing or reached your viewing limit for this book. Block Diagram Reduction Technique and Signal Flow Graph m 4.59 There are two combinations of two non-touching loops. They are I and 3 and 2 and 3. DP, hl, + Ll, =G,H,H, +H, AMIAE A) HEA: AMGH, + H+ Ha) + (GH Hy + HH) 34 UGG,G.G.G,)+(GG,G.H.)+ GH A+ GH, + Hy +H, +(G,H,H,)+ (HHH Pox, (1+G,H,- HHH, +G,H, +H, +H, +(G,H,H, ~ H\H, HH, +G,H,A, + HA, +G,H,H, +(GH,HH,)) x, x Calculation of |: To obtain yan artificial node x; =x, is created and the Mason’s gain formula x, 7 applied for input node x, and output node «/ as shown in Fig. 4.42 1. Identification of forward paths S.No. Forward Path Forward Path Gain 1 | =m -5-¥ 7,=GG, 2 [soa ee T= -GHM, TA +h: A The value of Ais same as given earlier since 4 is the property of the whole signal flow graph Calculation of A, If the nodes constituting the first forward path are removed, the following loops are not broken. They are x,—x, and x, —x,. DP = (+h) =, +4) Dhan = (ba) = AA, Hence, 4, (Lt hy) + LL, = 1+ (Hy + Hy) MH Calculation of A, If the nodes constituting the 2nd forward path are removed there is only one loop not broken and its loop gain Z,=-H,. Hence A, =(I-L,)=1+, 33 IGG, {1+ (A, + Hy) + HH }-GiHy Hs (U4 A] x, (+G,H,—H,H,H, +G,H, +H, + H, +(G,H,H, +G,H,H, + HH, - HHH, + GAH, + GH) HH) Example4.35_ For the electrical network shown in Fig. 4.43 draw the signal flow graph and hence find V,/V. The current source produces current a times the current that is passing through the resistance R,. (A.U,, Apr. 2004) aa You have either reached a page that is unavailable for viewing or reached your viewing limit for this book. aa You have either reached a page that is unavailable for viewing or reached your viewing limit for this book. aa You have either reached a page that is unavailable for viewing or reached your viewing limit for this book. Block Diagram Reduction Technique and Signal Flow Graph m 4.63 Loop | and 2 are non-touching, Hence L,., s?, There are no 3 non-touching loops. (h tht hth =(1F RCs + RCys + R,C,8 + RRCCyS*) 4. Calcualtion of A, With the forward path 1, if the connections of the feedback loops are removed, the loops are all broken. Hence, there are no feedback loops for the forward path | and so 4,=1. 5. Calculation of Ay With the forward path 2, if the connections of feedback loops are removed, feedback loop | and 3 are not broken. However, these loops are touching each other. Hence, sand LL, =0 Hence, a, L, +1.) =(14+R,C,s + R,C,s) 6. T=RRCCs° T=1 7. Using Mason’s gain formula, we get, Veg Tibi hay ¥, a RR CC,s? +14 RCs + B,Cys) * [IF RC s+ RCS + RCS + RRC S | Fae [RAC +R (C, +C,)s +1] KTR RCs BC+ RG RG )s+t] ¥, Example 4.37 For the network shown below, draw the signal flow graph and find 7. Ry 2 Ro ic pare 1 2 3 Fig. 4.45. Signal Flow Graph of Electrical Network of Example 4.37 aa You have either reached a page that is unavailable for viewing or reached your viewing limit for this book. aa You have either reached a page that is unavailable for viewing or reached your viewing limit for this book. aa You have either reached a page that is unavailable for viewing or reached your viewing limit for this book. Block Diagram Reduction Technique and Signal Flow Groph m 4.67 Itis to be noted that fora signal flow graph, A is the same for any output node to any input node. The suffixes a and & denote the nodes between x, tox, and x, tox,respectively. Substituting the Xe . ss above relationships in the expression for ,*. we get the following equation. eS Towne * OD Tu Aan a ‘Thus, Mason’s gain formulas applied between output node and non-input node. The determination of A is not required which is somewhat tedious. The following example illustrates the above procedure. Example 4.39 Consider the signal flow graph shown in Fig. 4.47. Determine ** x Fig. 4.47 Signal Flow Graph of Example 4.39 To determine > Tan. (from x, tox, node) SNo. | ForwardPath Forward Path Gain 1 Xray BBM, Ta=G,G,G,G,G, 2 Xr, Te=G66,G,G, All the 3 feedback loops are touching the first forward path. Hence when their connections are removed they are all broken. Hence 4), =! With the second forward path, the loop with gain -H, alone is not touching, Hence 42, =(I+ 2) bn 5 Tab = Tahia + Fybre ™ =G,6,6,G,G,+G,G,G,G,(1+H,) To determine ¥ Ty,nu (From x, to x, node) There is only one forward path between x, to x, node and the forward path gain 7,, aa You have either reached a page that is unavailable for viewing or reached your viewing limit for this book. aa You have either reached a page that is unavailable for viewing or reached your viewing limit for this book. aa You have either reached a page that is unavailable for viewing or reached your viewing limit for this book. 13. 14. 15. 16. 17. 18. 19. Block Diagram Reduction Technique and Signal Flow Graph ™ 4.71 input and becomes input to the first block. If the output of the second block aids the reference input the feedback is known as positive feedback. If the output of the second block opposes the reference input, the feedback is known as negative feedback. If the output at a point is obtained by summing more than one variable, such variables are said to be in feed forward connection. What do you understand by multiple input system? Sometimes, the system is given more than one input. The output response is found for each input separately assuming other inputs are absent. Then the total output response is found by adding the response due to the individual inputs. Such systems are called multiple input systems. What are the advantages of block diagram representation? (a) Easy to form the overall block diagram for the whole system by of each component. (b) The functional operation of the system is readily visualised by examining the block diagram rather than the physical system itself. What do you understand by non-interaction of blocks? ‘The basic assumption in the block diagram representation of a system is that what is inside the block is not affected by what is outside of the block except for the inputs. This means there is no interaction between blocks. Explain how important functions are concealed in block diagram representation. A serious drawback of block diagram representation is the possibility of important functions being concealed (omitted or hidden) within the walls of the block. This is due to manipulations of the physical equation for obtaining the T.F model and putting it in the block diagram form. ply connecting the blocks Explain the terms reciprocity and non-reciprocity with reference to block diagram representation? Passive symmetrical pi networks are reciprocal because input and output are interchangeable and any aniplifier network is non-reciprocal because input and output are not interchangeable, but both are represented by block diagrams without any reference to the above property. What is an input node, output node and mixed node of a signal flow graph? (a) Input node A node which has only outgoing branches is called input node. Input node represents independent variable or source. . (b) Output node A node which has only incoming branches is called output node. Output node represents dependent variable or sink. (c) Mixed node A node which does not belong to either input or output is called mixed node. It is necessary to note the above definition of input or output node because it often involves the drawing of artificial duplicate input or output node when input or output node appears as a mixed node in the graph. Define path and loop of a signal flow graph. A path is a traversal of branches connected by nodes in the direction of the arrows. If no node is encountered more than once, then the path is called open path. If the path ends at the starting node and does not encounter any node more than once, it is called closed path. If the path ends at the starting node and does not encounter any node more than once, it is called a loop. aa You have either reached a page that is unavailable for viewing or reached your viewing limit for this book. aa You have either reached a page that is unavailable for viewing or reached your viewing limit for this book. aa You have either reached a page that is unavailable for viewing or reached your viewing limit for this book. Block Diagram Reduction Technique and Signal Flow Graph @ 4.75 6, For the block diagram shown in Fig. 4.56 determine oa by block diagram reduction technique. Gs R Ols) EQ c la-Bre eho Hs, Fig. 4.56 Block Diagram of Question 6 i, +G,G,) iG, +G,GH, ~GyH\H,) cl 7. Consider the signal flow graph shown in Fig. 4.57. Determine oo using Mason’s gain formula. cls) Fig. 4.57 Signal Flow Graph of Question 7 Forward path gains are 7, =G,G,G,G,G,G, and T, =G,G,G,G,G,G, Loop gains (1) £,=G,H, (2) £,=G,H; (3) L, CE) _ G,G,GGG(Gs + G,) RG) [(-G,H, GH, -G,G,H, CGH, + GH\G,H, +G,G,H, +G,G,H,)] VGH, (4) Ly = yy 4, = Land A, =1 8. Obtain the ratio C/R for the system represented in the block diagram of Fig. 4.58. >| G3 a >| Go Hy Fig. 4.58 Block Diagram of Question 8 (G,+G,) 2H(G,+G M+ Ay) Fis) ow) > aa You have either reached a page that is unavailable for viewing or reached your viewing limit for this book. aa You have either reached a page that is unavailable for viewing or reached your viewing limit for this book. aa You have either reached a page that is unavailable for viewing or reached your viewing limit for this book. aa You have either reached a page that is unavailable for viewing or reached your viewing limit for this book. aa You have either reached a page that is unavailable for viewing or reached your viewing limit for this book. aa You have either reached a page that is unavailable for viewing or reached your viewing limit for this book. aa You have either reached a page that is unavailable for viewing or reached your viewing limit for this book. Time Response of Feedback Control Sysiems m 5.7 Solution The input applied is a ramp of slope @, F ©, As already derived, £,, = 10X27 | o5radisec Fo, _102«10 «1.05 E, | * 05x 513 =1.23x107 N.m/rad error For this controller gain and for other specifications given in the problem the damping factor cannot be 0.35.unless some external damping is added. Let F' be the new frictional coefficient. Then, Ks: Fl = 02S KS =2x0.35V 1.23107 «18x 10% 33010" N.mv/rad/see. Already there exists a friction of 102 x 10-* N.m/rad/sec. Hence the required external friction to be added to the system for the above specifications is, (330-102) x10 =228x 10° N.m/rad/sec ™ 5.6 TRANSIENT RESPONSE (OR TIME RESPONSE OR DYNAMIC RESPONSE) OF FIRST AND SECOND ORDER SYSTEMS The transient response or time response or otherwise called dynamic response of a system gives the variation of the system output variable with respect to time. Consider the block diagram shown in te) Fis) £8) oe Output H(e) Feedback Fig. 5.4 Block Diagram Representation of a Feedback System In Fig.5.4, R(s) is the input variable, C(s) is the output variable, E(s) is the error, G(s) the forward path transfer function and H(s) is the transfer function of the feedback path. From Fig.5.4, the following relationship is easily obtained. £(s)= 6.9) aa You have either reached a page that is unavailable for viewing or reached your viewing limit for this book. aa You have either reached a page that is unavailable for viewing or reached your viewing limit for this book. aa You have either reached a page that is unavailable for viewing or reached your viewing limit for this book. aa You have either reached a page that is unavailable for viewing or reached your viewing limit for this book. aa You have either reached a page that is unavailable for viewing or reached your viewing limit for this book. aa You have either reached a page that is unavailable for viewing or reached your viewing limit for this book. aa You have either reached a page that is unavailable for viewing or reached your viewing limit for this book. aa You have either reached a page that is unavailable for viewing or reached your viewing limit for this book. 5.16 m Control Systems Engineering ™@ 5.11 GRAPHICAL METHOD OF DETERMINING THE RESIDUES Let K(s+ + PMs+ Pr)(s+ r)(s+ rs) Let p, and p, be real and p, and p, be complex. The above expression is written as Pala) Fig. 5.9 Graphical Method of Determining the Residues First let us locate the poles and zeros of the given transfer function in thes plane. The pole is marked by a cross x and a zero is marked by a sinall cricle o. The pole-zero configuration is shown in Fig, 5.9. Suppose, we ate interested in determining the residue 4, atthe pole p,. Vectors are drawn from other poles and zeros to p,. The vector length to , from other poles and zeros are marked as A, B, C and D and the corresponding angles these vectors make with the reference lines are @;,6;,90°,@). The residue at p, is given by K x Product of directed distances from each zero to P, Product of directed distances from all other poles to F, KDz6, © AZ0,BZ0,C L390" The angles 0,,0,,0, and 90° should be measured positive in the anti-clockwise direction from the reference and negative in the clockwise direction. Reference line at any point is drawn horizontally to the right, from where the vector starts. aa You have either reached a page that is unavailable for viewing or reached your viewing limit for this book. aa You have either reached a page that is unavailable for viewing or reached your viewing limit for this book. aa You have either reached a page that is unavailable for viewing or reached your viewing limit for this book. 5.20 m Control Systems Engineering @ 5.12 TRANSIENT RESPONSE OF A FIRST ORDER SYSTEM Example 5.9 Derive the step response for a first order system. 0.6esT T=1 aT aT atts 5T Fig. $.13 Step Response of a First Order System Any first order system is characterised by the following transfer function c K <(s)=G(s)= Z=99 (s +a) where K and a are free parameters of the first order system. For unit step input A(s)= 1. The 5 following equation is written. . atl eal Taking inverse Laplace transform for the.above equation the following expression for c/1) is obtained. The step input response curve is as shown in Fig. 5.13. Example 5.10 Derive the ramp response of a first order system and plot the response. (A.U., Nov.2004) c(t) Fig.'5.14 Romp Response ofa First Order System aa You have either reached a page that is unavailable for viewing or reached your viewing limit for this book. aa You have either reached a page that is unavailable for viewing or reached your viewing limit for this book. aa You have either reached a page that is unavailable for viewing or reached your viewing limit for this book. 5.24 m Control Systems Engineering The constants 4,, 4, and A, are determined by referring to Fig. 5.15. From Fig, 5.16, (i-<") g R90°-8 “a ee -@)\90° “af o) A, = Conjugate of 4, aire where @=2-@ C(]R i 2ft-2)(erto, 70.) 2k Anis + j0,) Taking Laplace inverse, the following equation is obtained. = Rwhere tang = (60-10, -f +9} ostinato 58) L'e(s}=R| emo a 2 elt) = Since 0=(x-¢), (5.17) c(t)= Rl-— Te sin(w,t+9)] 5.15.2 Critically Damped Case ¢=1 where | tang= For the critically damped case of ¢ =1, for step input, the following expression for c(t) is derived on similar lines. et) = R[I-™! ~ 4 te] (5.178) aa You have either reached a page that is unavailable for viewing or reached your viewing limit for this book. aa You have either reached a page that is unavailable for viewing or reached your viewing limit for this book. aa You have either reached a page that is unavailable for viewing or reached your viewing limit for this book. 5.28 w Control Systems Engineering ‘The amount of maximum overshoot is obtained by substituting ¢ = on (-<) sing ee aise?) Since the steady state final value is 1, the first maximum overshoot My =C-l (6.21) Ifthe input R is considered, the first maximum over-shoot is, ie =Re WS M,=Re (5.22) The percentage maximum overshoot is given by. sei * 100 4 Ree %M , =—e eR abn My = eS 100 (5.23) Hence the overshoot depends on the damping ratio only. The curve damping ratio versus the percentage maximum overshoot is shown in Fig. 5.17. % Over-shoot 8 pi Fig. 5.17 Overshoot as a Fuinction of Damping aa You have either reached a page that is unavailable for viewing or reached your viewing limit for this book. aa You have either reached a page that is unavailable for viewing or reached your viewing limit for this book. aa You have either reached a page that is unavailable for viewing or reached your viewing limit for this book. 5.32 i Control Systems Engineering ™ 5.18 IMPULSE RESPONSE OF A SECOND ORDER SYSTEM Sometimes systems are subjected to shock inputs. For example, when a gun is fired, there isa reaction that has a large magnitude occuring for a very short duration of time. Such a disturbance occurring for a small interval of time is called an impulse. It has been already derived that the Laplace transform of an impulse is 1. For unit step, the Laplace tranform is I/s. The letter ‘s' is nothing but d/dt. Hence the response of the system to an impulse is nothing but the derivative of response of the system to step input. In the usual way of finding the transient response, the impulse response for the critically damped case is derived. For other cases, it is derived from step response by taking the derivative of it. By plotting the impulse response curve, one could conclude that the system is stable if the area under this curve is finite (see Chapter 6). Case I: Critically damped case ‘¢=1 The input-output equation for a second order system is, c(s : R(s) For an unit impulse R(s) =1, For.critical damped case ¢. (s) (s+e,) Hence, taking Laplace inverse for C(s) we get e=bCs)=oe™ The same result is derived by taking the derivative of the unit step resonse for the criti case. For unit step, of critically damped case, the output response is derived as (see equation (5.17a)). -e™ (14,0) de(t) at =0-[-c,e™ (I+0,1)+0,e™ ] el)=au Case Il: Underdamped case <1 For a second order underdamped system for unit impulse input, the following equation is written. 2 Oy (s° +2C0,s+03) oo, 4 (t0,4 70,40, =J0,) (+80, +70,) ” (+E0, ~70,) es aa You have either reached a page that is unavailable for viewing or reached your viewing limit for this book. aa You have either reached a page that is unavailable for viewing or reached your viewing limit for this book. aa You have either reached a page that is unavailable for viewing or reached your viewing limit for this book. 5.36 m Control Systems Engineering ett) Fig. 5.20 Transient Response Curve of Example 5.12 Solution From equation (5.5) the steady state error for unit step input is, Ek K It is given that £, = 0.1 rad and T= 1. Hence K= 10 From Fig. 5.21 the damped frequency of oscillation a rad oy= “°° 0.12 sec But (6.32) From equation (5.7), the steady state error for velocity input is © Buy = 26 se a a id It is given that for velocity input the steady state error is 2° for @; = = pie 2 and 5 “Oe 57g M4 (@, = lrad/see and £, =2°) “0, =57.36 28 Note :1 rad = 57.3° 10, = 57.36 5 (Note :1 rad = $7.3°) (5.33) Substituting equation (5.32) in equation (5.33), we get srt (ine) = salle )=ai5 ¢4-¢7+021=0 67 =0.7 0r 03 + $= 0.835 or 0.547 aa You have either reached a page that is unavailable for viewing or reached your viewing limit for this book. aa You have either reached a page that is unavailable for viewing or reached your viewing limit for this book. aa You have either reached a page that is unavailable for viewing or reached your viewing limit for this book. 5.40 m Control Systems Engineering Taking Laplace inverse transform, we get e(t) =0.524[0.1-0.1e7" -1e™ ] et) = 0.524[ 01-2" (0.14 trad] Example 5.15 A unity feedback control system has an open loop TF. 10 Ge) = 4D) Find the rise time, percentage overshoot, peak time, time delay and settling time for a step input of 12. units. (A.U, Dee, 2003) Solution 10 G9) Ty Cs)__ Gs) 10 Ris) 1+G(s) s?+2s+10 Comparing this with the standard second order system equation, we get oe) Rs) 8+ 2a,s+07 162 rad/sec. The rise time 0, =0,\1- 2 =3.162V1- 031? =3rad/sec + cos" =9 SS 5100 =e%-™ 100 = 36% %Overshoot Actual peak overshoot M, = 0.36 x 12= 43.2 units For estimating the settling time unless otherwise mentioned, the permissible error is taken as 2%. For 2% error, the expression for the settling time is, =m. aa You have either reached a page that is unavailable for viewing or reached your viewing limit for this book. aa You have either reached a page that is unavailable for viewing or reached your viewing limit for this book. aa You have either reached a page that is unavailable for viewing or reached your viewing limit for this book. 5.44 @ Control Systems Engineering Let , be the damping factor for 75% overshoot and ¢, for 25% overshoot. ie ev 20.75 or wee” 75 = -0.2876 Solving the above equation, we get 6, = 0.0911 025 or In0.25 = -1.386 Solving the above equation,-we get £, = 0.4037 From the expression derived earlier for ¢, , we get the following equations 1 = 0.0911 or K,7 = 30.12 (K,7-) £29.12 1 (K,T -1) = 0.53 Dividing one by the other, we get (KT) _ 29.12 _ 5 (K,T-1) 053 . (KF! =55 (KT) Example 5.19 The open lcop -F ofa unity feedback control system is given by G(s) = — _ = ses By what factor should the amplifier gain A be multiplied so that the damping ratio is increased from 0.2 to 0.67 : Solution The closed loop TF. of the given system is written as, C(s)__ Gs) A RG) 1G) Ts? +544 A CO) RY 4d gr4 aa You have either reached a page that is unavailable for viewing or reached your viewing limit for this book. aa You have either reached a page that is unavailable for viewing or reached your viewing limit for this book. aa You have either reached a page that is unavailable for viewing or reached your viewing limit for this book. 5.48 @ Control Systems Engineering (14K Ky) or = 2K +K Ky _ (26, -1) _(2x0.456x3.53~1) : K 12.46 Ky = 0178 The rise time But 0, =0, {i$ =3.53xVi-0456 = sec = cos! ¢ =cos™! 0.456 = 62.87° or I. trad Hens |g = 2h 0.65 se z For 2% error, m= 4 and time constant 7 =—1- =! __ 0.621 sec. Co, 0.456x3.53 Settling time 1, = mT =4%0.621 = 2484500 Answers K=12.46 K, =0.178 1, =0.65sec - 1, =2.484sec. Example 5.23 A closed loop servo system is represented by the differential equation. He ode 5 i ” P atte = 64e where c is the displacement of the output shaft, r is the displacement of the input l shaft and e = rc. Determine the undamped natural frequency, damping ratio and percentage maximum overshoot for the step input. aa You have either reached a page that is unavailable for viewing or reached your viewing limit for this book. aa You have either reached a page that is unavailable for viewing or reached your viewing limit for this book. aa You have either reached a page that is unavailable for viewing or reached your viewing limit for this book. 5.52 m Control Systems Engineering The transient response of the given system and that of the standard second order system is same at all time / except that the given system output is multiplied by a factor 2 at all times. Now, comparing the characteristic equations, we get w, = V5 and 2a, =4 or 6 = 0.894 m 0.89470 =3.8x10" Overshoot M,, = eVi-C” evi-0.8947 M,, = 3.810 38 %M, x10° x100= 0.19 Answers $=0894 M,, =3.8x10° %M, = 0.19% Example 5.27 Determine the damping ratio, undamped natural frequency for the system shown in Fig,5.25. What is the response c(¢) of this system to a unit step function excitation r(/) = u(r) when all initial conditions are zero? Ris) ote Fig. 5.24 Block Diagram of Example 5.27 Solution Compare the given G(s) to the standard second order system equation . 25 s(v+260,) s(s-+2) imal = Spm Lil or ¢ 3 * og = N10. 9rad /sec. The step function response of a second order system is derived as, : anos) cialis S° or 1.37 rad Since w = 1, and o=cos'f [e(t) = [1 1.026" sin (4,91 + 1.37], aa You have either reached a page that is unavailable for viewing or reached your viewing limit for this book. aa You have either reached a page that is unavailable for viewing or reached your viewing limit for this book. aa You have either reached a page that is unavailable for viewing or reached your viewing limit for this book. 5.56 m@ Control Systems Engineering Taking Laplace inverse, we get é c(t) ‘Taking the initial equilibrium value 1, we get the total response as, e(t)=1+e" -e™ The transient response curve is shown in Fig. 5.27. Example 5.30 Determine the values of K and a, of the closed loop system shown in Fig. 5.28, so that the maximum overshoot in unit step response is 25% and the peak time is 2 sec. Assume that J=Ikg.m?. (A.U,, Nov. 2004) Gs) Fig $.28(a) Block Diagram of Example 5.30 cs) Fig $.28(b) Reduced Block Diagram of Fig. 5.28(a) Solution From Fig .5.28 (a) Cs) __Ki ds he The given block diagram represented in Fig, $.28(a) after successive reduction is shown in Fig. 5.28(b). From Fig. 5.28(b), the following closed loop T.F. is obtained. = s(Js+ Ka) Compare this with the standard second order equation. Co) Rs) (5? +260,8+ 0,7) aa You have either reached a page that is unavailable for viewing or reached your viewing limit for this book. aa You have either reached a page that is unavailable for viewing or reached your viewing limit for this book. aa You have either reached a page that is unavailable for viewing or reached your viewing limit for this book. 5.60 m Control Systems Engineering Time constant Alternative method For unit step input the Laplace transform of the output variable is written as, 1 @.4s+1) 3 (st +54l) A, +A) As? +s+1)+4,5+ +541) s(s? +541) ci)= Comparing the numerators, we get the following equation. (O4S+1I)= Als’ +5+1I+AS+AS B(AtA)S H(A +A4)S+ 4, Comparing the coefficients of s?we get 4,+ 4, =0 or 4, =—A, Comparing the coefficients of s, we get (4 +4,)=04 Comparing coefficients of the free term, we get 4, =1 From the above we obtain that 4, = |; 4. = —0.6 and 4; = —1. Substituting these values in the equation for Cis), we get, (s+0.540.1) GET OO). (9? +541) S +541 =(s? +5+0.8667 +.25) But, 2 2 = (s+.5)) +0.866° 1_f 0.1 6 c++ -} 4 + FS Henee, 5” |(x05) +866 * 0.866 (540.5)? + 8607 +05 si ye, 050.8661 We know that Tea snee 0-88 =e 5! sin 0.8661 and, (s+.5)? +0866" Hence, eft) = £'C(s) = 1-0" {0s 0.8661 + 0.1 15 sin 0.8667} ale Ors? {sina 86+ 5 i} Answer elt) = 1-2 sin(0,8661 +1.46) aa You have either reached a page that is unavailable for viewing or reached your viewing limit for this book. aa You have either reached a page that is unavailable for viewing or reached your viewing limit for this book. aa You have either reached a page that is unavailable for viewing or reached your viewing limit for this book. 5.64 w Control Systems Engineering Type 0 System Type 0 system is written as K, (i+52,)(1+52,).. = A A GHG) = Tee leap) From equation (5.43) K, = Lt GH(s)= K, R R oT) TAK From equation (5.44) Type 1 System K,(1+52,)(1+52,).. F I stem, let = ‘or a type I system, let GH(s) Tenia K, = LtGH(s)=0 R "TS Type 2 System The type 2 system is considered as K,(+sz,)(1+s2,)... = Killtse (lt s2,).. GHO)= Ses lap, K, = LAGH(s)== R t= $o2 2g, (6.43) (6.46) (6.47) (6.48) (5.49) Hence for step input, other than type 0 system, the steady state error is zero. Only for type 0 system, it gives finite steady state error. @ 5.21 STEADY STATE ERROR FOR VELOCITY INPUT Let the velocity input »(t) = Rt where R is the stope of the ramp in rad/sec. Hence R(s)= = # sR F.> E()= MoT GAG] oF R. O+sGH(s) Ky where = LesGM(s)| and Ege. kK, (6.50) (5.51) K, is called the velocity error constant and has a unit of sec”'. The steady state error for various types of the system is determined in terms of K,, aa You have either reached a page that is unavailable for viewing or reached your viewing limit for this book. aa You have either reached a page that is unavailable for viewing or reached your viewing limit for this book. aa You have either reached a page that is unavailable for viewing or reached your viewing limit for this book. 5.68 & Control Systems Engineering Example 5.34 For a control system (0) = 10u(0) ~ St where u(0) is unit step. The output C(s) is 75(s +a) (2s~1) given as C(s)= 3 (s+16) (5430) Determine the steady state error £.,. Solution r(Q)=10u(s)—5¢ and u(e)=1 rye 5 pic Error E(s)= RG) c(s) _5(2s 15(s+ a)(2s—1) - *(s+16)(s+30) £(s)= 5(2s-1)[(s+16)(s+30) ]-75(s+a)(2s-1) 2 (s+16)(s+30) Steady state error g = Lise(s) fe = ue 25(28-1) 1) [(s-+16)(s+30)]-75(s+a)(2s-1}s rw ar (s+16)(s+30) Boe (-5)[(16%30)]+75a_ a 0x16 x30 Example 5.35 Find the response as ¢ approaching infinity of the system having a T.F 15 OTe a ae 4 ©) 70)= Fas) Comment on the responses. Solution Since the type of input is not mentioned, the response is determined for impulse, step and velocity. (a) For impulse input R(s) = 1. If we denote C(s) as the output, then cs) R(s) Clo)= poe sC(s)= tt soa =2 T(s)= For unit step input, R(s)=4 = ’ 8 OTH Clee)= Lt sCis)= uae For velocity and acceleration inputs, the output is infinity as f° aa You have either reached a page that is unavailable for viewing or reached your viewing limit for this book. aa You have either reached a page that is unavailable for viewing or reached your viewing limit for this book. aa You have either reached a page that is unavailable for viewing or reached your viewing limit for this book. 5.72 m™ Control Systems Engineering Hence we need to determine only C, and C,. (541) F(s)= now" Gell C= LF (s)= u-th- a aF(s)_(U+s)-(s+1) ds (+s) _ ) (il+s)=(s41)_ 10 Oe esy) TT Constants C,, C, etc., need not be determined since #(), #0) etc. are zero, Hence, £, = LifCr(1)+Gr(1)] = Lt iva] “10 li 121 a ett) Hence the error is given in terms of time ¢, It gives an idea of how the error accumulates with time. Example5.38 A unity feedback systems has G(s) = at 5 The input tothe system is described s+ by (0) = 4 + 6¢ + 2P. Find the generalized error coefficients and hence the steady state error. Solution r(t)=4+ 6142" F=6+6" F(t) = 120 ' F(Q=12 #()=0 The constants C,, C,, C, and C, are to be determined. 1 (st+s F(s)= Tee = oy C= Lt F(s)=0 AF (5) _(s +541) Qs4I)-(s? +5) 2541) __ 2s41) as (f+st) (st+s41) gay) ve (s+se1) 2(2st)'(s'+s41) -6(s?+5) (easel) (6 stl) aa You have either reached a page that is unavailable for viewing or reached your viewing limit for this book. aa You have either reached a page that is unavailable for viewing or reached your viewing limit for this book. aa You have either reached a page that is unavailable for viewing or reached your viewing limit for this book. 5.76 m Control Systems Engineering Solution The inner loop of Fig. 5.31 is reduced as follows: LE) _ gy Ze 7) eh __G)___ 6410) 14+GH(s) ,, Ks s(s-+10) K K 6@-—— = eID B k= LpsG=% uty = SOT Steady state error, Example 5.43 A certain unity negative feedback control system has the following forward path TR. Gls) Me) .When the system is subjected to unit step input, it is observed that the steady s+ state error is 0.25. The system is to have a natural frequency of oscillation , =2 rad/sec and damping factor £ =0.6. Determine the value of K, a and b. Solution The closed loop T.F. of the given system is obtained as, chs) Gs) Rs) 1+G(s) _ K(s+a) _ K(s+a) © (stb) K(st+a)) (s? +(K+26)s+ (Kat 5%] Compare the characteristic equation with that of the standard second order system. Thus, 8? +200,5+@} = 5s? +(K +2b)s+(Ka+e") aa You have either reached a page that is unavailable for viewing or reached your viewing limit for this book. aa You have either reached a page that is unavailable for viewing or reached your viewing limit for this book. aa You have either reached a page that is unavailable for viewing or reached your viewing limit for this book. }0 @ Control Systems Engineering Consider the system represented in block diagram form as shown in Fig. 5.33. G(s) = TF. of the controller Gy(s)= T.F.of the plant R(s) = Input tothesystem D(s) = Disturbance E(s) = Error C(s) = System output From Fig. 5.33 the following equation is written. C(s) = E{s)G\(8)G,(s)+ DCS) G(s) But Cls) = R(s)~ Els) « R(s)~ E(s) = E(s) G(s) G,(s) + D(s) G(s) oro Espen ee fie UD Gy 1+G,(s) G(s) 1+G,(3) G3) The first term on the right hand side represents the error due to R(s) and the second term due to the disturbance D(s). By applying final value theorem. £,, is obtained. Eqg= Lt sE(3) Example 5.45 Consider the system represented in Fig. 5.34, Determine the steady state error due to unit step input and a step disturbance of 10 unit. Des) Ris) as) Fig. 5.34 Block Diagram of Example 5.45 Solution The error due to input disturbance is, Ris) eh 2O"TG@ Gan ere ROn; 1 ; (s+215+4) gi ttt mete te Fi 1005) 5] [sle+2Xs+4)+ 5006+5)] (s+2) s(s+4), Steady state error is, = & bye SENT) Boo, = LES BUS) = Ut ey ays +4)+5006645)] =0 aa You have either reached a page that is unavailable for viewing or reached your viewing limit for this book. aa You have either reached a page that is unavailable for viewing or reached your viewing limit for this book. aa You have either reached a page that is unavailable for viewing or reached your viewing limit for this book. 5.84 m Control Systems Engineering The closed loop transfer function of a second order system with PD controller is GEO) ay (4) (Kp +Kgs)o} eR £ RO G.Ge | 2 4(2l0, + Kyo? )s+ Kpar Kp From equation (5.77) it is evident that a zero at -—" is added to the system. We have already 4 shown that a system with addition of a zero executes more oscillations with increased amount of overshoot and decreased rise time and fastness of response as K,, is reduced, However from equation (5.77) it is evident that the coefficient of s of the characteristic polynomial is modified from 2¢w, to (go, + K,~2) which increases the effective damping of the system. The increased damping ultimately reduces the peak overshoot. The transient response curves of a typical second order system with =0.6 and @, =2 for unit step input are given in Fig, 5.38. ee ; ti Fig. 5.38 Transient Response with PD Comroller ™@ 5.28 Pl CONTROLLER The transfer function of a PI controller is given by _ Ky) _[ Ks 4K, c.O)=(«+4 } [ +8] (6.78) The closed loop transfer function of a second order system with Pl controller is obtained as, é GING) of (K,s+K,) Cpa GIGS) _ yy RO) Tea aM)" 4 2Go,5+0iK,ss0iK, (5.79) ee . a Ky This is equivalent to adding zero to the closed loop transfer function at s =~. From equation ie (5.79) it is evident that the forward path transfer function G(s) G(s) has its type incréased from one to two, and the order of the system from two to three. By increasing the type of the system by one, the steady state error of the original system (without integral controller) is improved by one. That is aa You have either reached a page that is unavailable for viewing or reached your viewing limit for this book. aa You have either reached a page that is unavailable for viewing or reached your viewing limit for this book. aa You have either reached a page that is unavailable for viewing or reached your viewing limit for this book. 5.88 M Control Systems Engineering x ee — 3 = 0.2416 rad/sec 8.666 For2%error et 4 208 sec fo, 05x10 where 7 =< fu, - aa 05x fet %M, =e" * x100 = x100= 16.3% The period of oscillation is given by, 2n_ In 7-22 aZZ 207 om ggg 7 O 72SEC The number of oscillations completed before reaching the settling time is given by, qr 08 21 1 eycles % 0.725 With Derivative Feedback Control The new damping factor with derivative feedback is given by, (from (5.80a)) " K, gecetto, where ¢=0.5, 6’=0.8 and @, =10 Substituting these values, we get (/-6)2 _ (08-0.5)x2_ K, = tt = CATE 0.06 ®, 10 Now, = cos 0.8 = 0.6435 rad i=? = 10V1=0.64 = Gradisec 0.6435 6 = 0.4163 sec 4 08x10 4 =4T= = 05sec Percentage M, = e* x 100 = 1.52% aa You have either reached a page that is unavailable for viewing or reached your viewing limit for this book. aa You have either reached a page that is unavailable for viewing or reached your viewing limit for this book. aa You have either reached a page that is unavailable for viewing or reached your viewing limit for this book. 5.92 m Control Systems Engineering d) Both a, and a,’are complex with their real parts being negative. The pole location for this case is shown in Fig. 5.44(d). The following expression is derived for c(t). eli)= Ke sinar o The above expression is plotted and shown in Fig. 5.44(d°). e) Both a, and a, are complex with their real parts being positive. ‘The pole location for this case is shown in Fig. 5.44(e). The following expression for c(t) is derived. K saat g yak ot sina e()= Set sin The above expression is plotted and shown in Fig. 5.44(¢’). From the above plots, it is evident that case (a) indicates stable operation. Case (b) and case (e) denote unstable operation where the response curve increases exponentially with respect to time. Case (c) is purely oscillatory and case (d) belongs to stable operation with exponentially decaying oscillations @ 5.33 THE SENSITIVITY ‘ The change in environment, aging of components used and drift in operating conditions cause variation of parameters in the system. These variations in the system parameters alter the T.F. of the system which in turn gives a totally different characteristics from the predicted ones with the original parameters. The system should be insensitive to such variations. A measure of system sensitivity is called sensitivity function. Thus, 1 _ Ye change in Mf “ee change in G A) 5 / x yo a5 o%s) H(s) Fig. 5.45 Block Diagram of a Feedback System For the block diagram shown in Fig. 5.45, the closed loop T.F. is derived as cw Rs) M(s) Gs) 1+GH(s) 1. The sensitivity of M to change in G is obtained as, AM sf = te G aa You have either reached a page that is unavailable for viewing or reached your viewing limit for this book. aa You have either reached a page that is unavailable for viewing or reached your viewing limit for this book. aa You have either reached a page that is unavailable for viewing or reached your viewing limit for this book. 3.96 @ Control Systems Engineering CHAPTER SUMMARY |. A second order system dynamic equation is derived in a standard form and expressed in terms of two parameters namely the damping ratio ¢ and natural frequency of oscillation @, . 2. The steady state error of a second order system for step and ramp inputs are derived under no load and load conditions. 3. Todetermine the tranisent response of a system for any input, the transfer function of the system is put into partial fraction and the residues are determined. Both conventional (Comparing the coefficients of s) as well as graphical methods are used for this. The above two methods are explained with examples. However, it is emphasized that the graphical method of determining the residues is much simpler than the conventional method. The graphical method is especially convenient when the given TF. is complex and has repeated poles. It is to be noted here that one need not draw any graph for this. Further, the solution is obtained very quickly and expressed in compact form. 4. Time response of a first order system for step and ramp inputs are dervied and performance specifications are defined. 5. Expressions for the transient response of a second order system for step, ramp and impulse input signals are derived. 6. The transient response curves of a second order system to step input are plotted for various damping factors. Time response specifications such as rise time, settling time, time delay, peak- overshoot, the time at which the peak overshoot occurs, time constant, the period of oscillations and the number of oscillations before the settling time is reached are defined and the expressions for these specifications in terms of damping factor and natural frequency of oscillations are derived. 7. The steady state performance of any system for step, ramp and parabolic inputs are determined using static error constants K,, K, and K,, respectively. To know the variation of the error with respect to time, dynamic error constants C,,C,,C, ... C, are used. 8. The transient and steady state performance characteristics of a second order system with and without PID controller are determined and compared. 9. The effect of pole locations in the s plane of a second order system is studied in terms of transient response and stability. Finally, the sensitivity of the system for variation of parameters is discussed and illustrated with examples. EXERCISE Give brief answers in about.three lines 1. Compare the step responses of the first order and second order systems. The step response of the first order system is free from any oscillation. However, for the second order system, the system response contains oscillations for under damped systems as shown in Fig. 5.49. aa You have either reached a page that is unavailable for viewing or reached your viewing limit for this book. aa You have either reached a page that is unavailable for viewing or reached your viewing limit for this book. aa You have either reached a page that is unavailable for viewing or reached your viewing limit for this book. 5.100 m Control Systems Engineering 25, What is the time constant of a control system? Time constant is the time when the output reaches 63.2% of its final value for the very first time Lower the time constant, faster the response of a control system is. 26. Define overshoot. Overshoot of a system indicates the relative stability of the system. It is often expressed as percent age overshoot. Peak overshoot = (Maximum peak value ~ Final value). Percentage peak overshoot (Maximum peak value-Final value) %M p = 100 , Percentage overshoot is also defined as the amount Final value that the under damped step response overshoots the steady state or final value at the peak time expressed as a percantage of the final value. 27, What is the importance of impulse response curve of a system? From impulse response, the transfer function of a system is easily derived. By plotting the impulse response curve one concludes that the system is stable if the area under this curve is finite. It is also possible to derive response due to other inputs from this. 28, Explain why a system is tested for step input? 1. Step input is easy to generate and apply to the system. 2. Itis sufficiently drastic. If the system gives satisfactory performance for step input, it is likely to give satisfactory response to other type of commonly used signals. 3. Ifthe response to step is known, from this it is possible to derive the response to other type of inputs. . 4. The application of step input is equivalent to the application of numerous sinusoidal signals with a wide range of frequencies. 29, What do you understand by damped frequency of oscillation? @, =0, ¢°) is called the damped frequency of oscillation of the system where @, Re type 0 2nd order tstonder type 1 t (a) (b) Fig. 6.6 (a) Terminating Part of Polar Plot (b) Starting Part of Polar Plot ™@ 6.6 MINIMUM AND NON-MINIMUM PHASE TRANSFER FUNCTIONS Aminimum phase transfer function is one whose poles and zeros all fall in left half of the complex s plane (LHP). G(s)= "S75 is aminimum phase TF. and G(s)= ZA is anon-minimum phase TF. If atleast one pole or a zero of G(s) falls in RHP, then the LF. is called a non-minimum phase T-F. aa You have either reached a page that is unavailable for viewing or reached your viewing limit for this book. aa You have either reached a page that is unavailable for viewing or reached your viewing limit for this book. aa You have either reached a page that is unavailable for viewing or reached your viewing limit for this book. 6.10 m Control Systems Engineering @ 6.7 MINIMUM PHASE TRANSFER FUNCTION Transfer functions having poles and zeros all in the left half s plane (LHP) are said to be minimum phase transfer functions. Those transfer functions having any pole or zero in the right half s plane (RHP) or in the imaginary axis are said to be non-minimum phase transfer functions. Systems with minimum phase transfer function are called minimum phase systems, whereas those systems with non-minimum phase transfer functions are called non-minimum phase systems. The following T.F belongs to minimum phase category. K(s+2) ‘The following transfer functions belong to non-minimum phase category. a) GG)= K(s-2) s(s+1)(5-+3) K(s+2) 5(8=1)(s+3) K(s~2) s(5=1Xs+3) It is to be noted that the magnitude of the zero of the sinusoidal TF. (j@~2) and that of a) G(s)= b) G(s)= c) G)= (jo+2) is Vo? +4 in both the cases. However the phase angle for (ja-2) is 180°—tan” ra whereas itis tan" 2 forthe sinusoidal TF. (w+ 2). Therefore itis advisable to put the poles and zeros ofthe sinusoidal non-minimum phase T-F. in thes plane and the corresponding phase angle is determined. ™ 68 ALL PASS TRANSFER FUNCTION The transfer functions which have poles and zeros located symmetrically with respect to the imaginary axis are called all pass transfer functions. The following transfer functions are examples of all pass transfer functions, Their pole-zero locations are shown in Fig, 6.8. (5-2) (5-2) (543) (st +2544) G(s)= (b) G(s) = ——————— aj=> @ =O MGO= a ays-3) =O a544) 1 p -l+j3 0 x +H+p3 2 » ae S32 [) wae Be o -1L-N3 0 x +p Gte)= 3 Gi EAE ote) FBTR @ © @ Fig. 6.8. Representation of All Pass Transfer Function in s Plane aa You have either reached a page that is unavailable for viewing or reached your viewing limit for this book. aa You have either reached a page that is unavailable for viewing or reached your viewing limit for this book. aa You have either reached a page that is unavailable for viewing or reached your viewing limit for this book. 6.14 @ Control Systems Engineering 3. Now consider the zero (s~2) and pole 7,5. Theit sinusoidal transfer functions are (jo~2) I and Fig. 6.11(a). From Fig. 6.1 1(a), the vector OA makes an angle of | (190-1 2) from the reference and the vector OB makes an angle of -tan' from the reference. respectively and these vectors are represented as OA and OB respectively in The angle of G(/o) is therefore, For @=0,9=0 and for @ =, =~90". The following table is prepared, transferred and plotied in Fig. 6.11(b). Table 6.2 @fo,i [2s 8 [.@ @ | 0 | -24 |-40° | -62° | =70.6° | -90° la] | = | 1.06 | 0.29 [0.047 | 0.015 | 0 When @=0, the phase angle ¢=0 and the magnitude of |G(jo)|=~. When o = the phase 1@ , is dominating over tan’ =. Hence ¢ isalways negative which is further increased by ~ tan”! — Hence the polar plot is always in the fourth quadrant and is sketched as shown in Fig. 6. 1. angle @=-90° andthe magnitude of | G(jo) = 0 .In the expression for phase, the term tan Example 6.9 Draw the polar plot for the transfer function 10 (+1) (s+10) G)= (A.U. Nov. 2004) Im 10(s+1) (+10) Fig. 6.12 Polar Plot for G{s)= aa You have either reached a page that is unavailable for viewing or reached your viewing limit for this book. aa You have either reached a page that is unavailable for viewing or reached your viewing limit for this book. aa You have either reached a page that is unavailable for viewing or reached your viewing limit for this book. 6.18 m Control Systems Engineering mn. lus co _ya0 Ro Fig. 6.16 Polar Plot of a Second Order System Just like the transient response curves, we have frequency response curves in the frequency domain. The magnitude versus frequency, phase angle ¢ versus frequency plots are shown in Fig. 6.14 and 6.15 respectively for various values of damping. The polar plot is shown in Fig, 6.16 for second order system for various damping factors. Example 6.11 Asecond order system has a natural frequency of oscillation of 2 rad/sec. with its damped frequency of oscillation as 1.6 rad/sec.calculate (a) the % over-shoot when subjected to step input (b) the maximum ratio of output when sujected to a sinusoidal input. Solution Given In Chapter 5, we have derived that (a) % over-shoot M,= sev) x 100 = 9.48% (b) From expression (6.5), resonant peak 5 1 Wace 2x06x08 Answers © =0.6and My = 9.48% M, = 1.042 aa You have either reached a page that is unavailable for viewing or reached your viewing limit for this book. aa You have either reached a page that is unavailable for viewing or reached your viewing limit for this book. aa You have either reached a page that is unavailable for viewing or reached your viewing limit for this book. aa You have either reached a page that is unavailable for viewing or reached your viewing limit for this book. aa You have either reached a page that is unavailable for viewing or reached your viewing limit for this book. aa You have either reached a page that is unavailable for viewing or reached your viewing limit for this book. aa You have either reached a page that is unavailable for viewing or reached your viewing limit for this book. aa You have either reached a page that is unavailable for viewing or reached your viewing limit for this book. aa You have either reached a page that is unavailable for viewing or reached your viewing limit for this book. aa You have either reached a page that is unavailable for viewing or reached your viewing limit for this book. aa You have either reached a page that is unavailable for viewing or reached your viewing limit for this book. aa You have either reached a page that is unavailable for viewing or reached your viewing limit for this book. aa You have either reached a page that is unavailable for viewing or reached your viewing limit for this book. aa You have either reached a page that is unavailable for viewing or reached your viewing limit for this book. aa You have either reached a page that is unavailable for viewing or reached your viewing limit for this book. aa You have either reached a page that is unavailable for viewing or reached your viewing limit for this book. aa You have either reached a page that is unavailable for viewing or reached your viewing limit for this book. Frequency Domain Analysis of Control Systems m 6.41 7. At point ¢, again a pole interferes altering the slope from +20 dB/dec to 0 dB/dec. Hence horizontal line ed is drawn. 8. At w=5, the dB =28. The gain K should make this dB = 0 at @ =5 to meet this requirement. In other words 20log K = -28 or K= 0.04. The Bode plot is shown in Fig. 6.33. Example 6.23 For the transfer function shown below, sketch the Bode phase angle plots. 10+ ses s(1+0.2s) Gs)= 10+s)e Fig. 634° Phase Plot for Oe ea Solution For the transportation lag ¢~®'s_ the expression for the magnitude and phase angle are, ee = -o.lorad or 1|-0.10%57.9° Hence |G(jo) = tan" @ = 0.1057.3° -90°- tan“"0.20. Using the above expression ¢ is calculated for various values of » as given in Table 6.7. The phase-angle curve is sketched and shown in Fig. 6.34, Table 6.7 @| 0 1 2 10 100 @ | —90° | —62° | - 60° | -126° | - 303° ™@ 615 DETERMINATION OF TRANSFER FUNCTION FROM BODE MAGNITUDE PLOT If one is able to draw the Bode magnitude plot for the given transfer function then it should not be difficult to get the transfer function if the Bode magnitude plot is given. However, the method described below is valid only for transfer functions of minimum phase. The following steps are followed to get the transfer function from Bode magnitude plot. 1. Identify the low frequency response portion (start) of the Bode plot. 0 dB/dec. (Horizontal plot) corresponds to K +20 dBidec. to Ks, 40 dB/dec. to Ks? etc. and —20 dB/dec. corresponds to 73 ~40 aBldec. to % ete. 2. Wherever there is a change of slope, assign the comer frequencies 7,, 7, etc. aa You have either reached a page that is unavailable for viewing or reached your viewing limit for this book. aa You have either reached a page that is unavailable for viewing or reached your viewing limit for this book. aa You have either reached a page that is unavailable for viewing or reached your viewing limit for this book. Frequency Domain Analysis of Control Systems m 6.51 Consider the following TF. of a closed loop second order system. N=, O= Fito,seet i [fig ene St f ay + 4g?oFa? |? Let @, be the frequency at which the gain is reduced to 1/V/2 times its low frequency value. This frequency is called as cut-off frequency. Thus putting w= a, .we get, 1 o V2 fe? -ot) +4%ate} Solving the above equation for m, we get, oy -e4[ 1-262 4a a Hence the bandwidth of a second order system whose magnitude is non-zero at @ =0 is given by q B.W.=0,[1-26¢ + fal" =a" F (6.16) It is to be noted here that for some system, large bandwidth is necessary for the system to respond to a wide frequency range of input signal. For certain other systems small bandwidth is necessary so that it provides attenuation to the high frequency noise in the input signal. Large bandwidth corresponds to a small rise time and fastness of response. However it requires high cost components, 8. Cut-off -Rate The slope of the log-magnitude curve near the cut-off frequency is called cut-off rate. It is the frequency rate at which the magnitude ratio decreases beyond the cut-off frequency. Example 6.29 Determine analytically, the gain margin and phase margin for the system with the following transfer function. Also determine the gain cross-over and phase cross-over frequencies. GH(s)= Solution (6.17) Let @p_ be the frequency at which the polar plot ex crosses the -ve real axis. Thus, 90° tan 2 — tan? Se = -180° 2 6 aa You have either reached a page that is unavailable for viewing or reached your viewing limit for this book. aa You have either reached a page that is unavailable for viewing or reached your viewing limit for this book. aa You have either reached a page that is unavailable for viewing or reached your viewing limit for this book. 6.58 m Control Systems Engineering ‘To find the phase margin and gain margin, the Bode plot is preferred. For this, the magnitude and phase plots are required. To draw the magnitude plot 1. For the given G(s), the corner frequencies are identified as 5, and 50. Vertical dotted lines passing through @ = 5 and w= 50 are drawn. At-these frequencies, the slope of the dB line changes. . The low frequency Bode plot is duc to —. The comer frequency is 1 and the slope is ~20dB/dec. At w= 5, the dB = 20 log i —144B. At =, the dB = 0. A straight line is drawn connecting these points. At w= 5, the dB slope changes from —20dB/dec. to -40dB/dec. and this goes upto w= 50, which is one dec. from @ = 5. Hence at w= 50, the dB = - 14 - 40 =—54. Hence a straight line with 40 dB/dec slope from —14 dB to ~ 54dB at w= 50 is drawn. |. At = 50, the slope changes from — 40 dB/dec. to — 60 dB/dec. w = 150 is taken. The increase in dB from @ = 50 to » = 150 is calculated as follows. Increase in dB =60 ere = 28.63 Hence at o= 150, dB = -54 — 28.63 = -82.63 dB. Hence a straight line from —54 dB at = 50 to — 82.63 dB at © = 150 is drawn and extended. This completes the Bode magnitude plot. To draw the Bode phase plot ‘The expression for the phase angle of the given T.F. is, $= —90°-tan“! 0.2@-— tan“! 0.0200 For @ 70, $= -90° For @ >, g=-270° The phase plot varies from ~90° to -270°. It is not necessary to calviilate the phase angle for all values of w . Only the following two cases need to be considered. lL The gain cross-over frequency @g- where the dB = 0 is at a= 1 as seen from the magnitude plot. Corresponding to this frequency, the phase angle is calculated as, $= -90~tan™!0.2-tan™'0.02 =-102° Hence, the phase margin ¢, = 180°-102° = 78° and it is positive. The phase cross-over frequency @p. where the phase angle is 180° is calculated as follows. -90°—tan“'0.20,, - tan“! 0.020, = -180°V2 or tan“'0.2m,, + tan™'0.020,, = 90° 1 0.20j_ + 0.020 1-0.20,,0.020,, 0.220, or ———#— = tan90° = 1-0.0040,. Oor Ope = 15.82 or tan e This is possible if 1-0.004a,. aa You have either reached a page that is unavailable for viewing or reached your viewing limit for this book. aa You have either reached a page that is unavailable for viewing or reached your viewing limit for this book. aa You have either reached a page that is unavailable for viewing or reached your viewing limit for this book. “Frequency Domain Analysis of Control Systems m 6.65 60, parm 0.5770, + 60,, =8X0.577 @ +1040, =8 Solving the quadratic equation we get, @, =0.719 Ata, — |G(je,)I=1 —— Hence, ,,4f(@2, + 4)(«2, +16) K =1 0.719(0.719° +4)(0.719° +16) K=621 Example 6.39 Determine the phase margin and gain margin using Bode plot for the T-F. 40(5 +1) s* +25 +41 +55) 1 Gs)= 90" + °180" aa You have either reached a page that is unavailable for viewing or reached your viewing limit for this book. aa You have either reached a page that is unavailable for viewing or reached your viewing limit for this book. aa You have either reached a page that is unavailable for viewing or reached your viewing limit for this book. Frequency Domain Analysis of Control Systems Ml 6.71 ju) plot (b) Fig. 6.54 To Obtcin Closed Loop Magnitude using Constant M Circles l r sR For M=1, we have from equation(6.21), » 5 which is the equation ofa straight line parallel 1 to the y axis and passing through (-£0) point. The constant M loci are shown in Fig. 6.54. This set of curves are readily available. The constant M loci are symmetrical with respect to M = I straight line and the real axis. The G( jw) plotis plotted in a transparent paper drawn to the same scale as the M cricles are drawn and superimposed on the constant M-circles. The point of intersection of the Gio) plot and the circle gives the value of M for that frequency. The tangential point of the Gia) plot with M circle gives the peak vlue Mand the corresponding frequency gives the resonant frequency «, . This is repeated for other values of « also. From Fig. 6.54(b) M_= 1.4and the resonant frequency is 4. The closed loop frequency response calcualted from Fig. 6.54(b) is shown in Table 6.11. Table 6.11 o [il2t3tatstoet7isto fioti M S Li fn2]rs}i4a sis} 12 ]ir}1}o9)o8|o7 1 1 ™ 6.17.2 The Constant N Circles The loci of constant phase shift of the closed loop system also called constant N circles, simplifies the laborious process of determining the closed loop phase shift. Just like the constant M circles, there are a set of readymade constant phase angle circles available which give the phase angle of the closed loop system. The equation of these circles is derived below. Let the closed loop phase shift be #. Let tan ¢=N of @= tan”! N . From Fig. 6.53. we get where. aa You have either reached a page that is unavailable for viewing or reached your viewing limit for this book. aa You have either reached a page that is unavailable for viewing or reached your viewing limit for this book. aa You have either reached a page that is unavailable for viewing or reached your viewing limit for this book. Frequency Domain Analysis of Control Systems ™ 6.79 Table 6.14 ao 05 1 12 1.732 2 | © in degrees 126° -153° -162° 180° 187.3" |eUja)| 3.33 1,342 1 0s 0372 20 |G(J0) N 235 0 ~ | 86 indB ‘The data prepared in Table 6.14 is transferred to Nichol’s chart in Fig. 6.63. 4 1. oy fot 4 Fig. 6.63 To Determine $,,and G,, from Nichol’s Chart To find the phase margin To find the phase margin, the phase angle corresponding to 0 dB is obtained. Corresponding to 0 dB, the G(je) plot is cut at w =1.2. This frequency is the gain cross-over frequency and the corresponding phase angle in the chart is obtained as -162°. Hence, the phase margin 9, = 180°-162°=18° which is +ve. To find the gain margin ‘To find the gain margin, the gain corresponding to -180° phase angle is obtained. Corresponding to 180° phase, the G(jo) plot is cut at @ = 1.732 and the corresponding dB = ~6. Hence, the gain margin is +ve. The phase cross-over frequency is @, = 1.732. aa You have either reached a page that is unavailable for viewing or reached your viewing limit for this book. aa You have either reached a page that is unavailable for viewing or reached your viewing limit for this book. aa You have either reached a page that is unavailable for viewing or reached your viewing limit for this book. i. 12. 13. 14. 15. 16, 17. Frequency Domain Analysis of Control Systems m 6.85 Define the terms resonant peak M, and. resonant frequency @,. Resonant peak is the peak value of the magnitude plot of the frequency response curve. Resonant frequency is the frequency at which resonant peak occurs. For a second order system the following expressions are derived. Resonant peak M,= Resonance frequency @, = w,/1-2¢" . 2 2(s+1) Determine the phase margin for GH = e =180°+ tan”! w. For M = ©, = 180° + 180°+ tan ‘2.2 = 65.6° This is positive phase margin. In such case the closed loop system is stable. What are the two Bode plots? Magnitude of the sinusoidal T.F. expressed in dB versus logw and phase angle of the sinusoidal LF. versus log @ are the two Bode plots. What are the uses of Nichol’s chart? By superimposing the open loop frequency response plot on the Nichol’s chart it is possible to get the closed loop magnitude and phase at various frequencies. Thus one could get closed loop freqency response by noting the points of intersection on the constant dB plots and constant phase plots. It is also possible to obtain the required gain for the ‘desired resonant peak M., It is also possible to determine the stability and other frequency domain specifications from Nichol’s chart. Define gain cross-over frequency. Gain cross-over frequency is the frequency at which the magnitude of the open loop sinusoidal transfer function is unity. How is location of roots of the characteristic equation related to sta ity? fall the characteristic roots fall in LHP of the s plane the system is stable. [fany one root falls in RHP, the system is unstable. The presence of complex conjugate roots in the imaginary axis indicates that the system is oscillatory. What is bandwidth? The frequency range 0a, in which the magnitude of the closed loop sinusoidal T.F. Ca) R(ja)| What is phase cross-over frequency? Phase cross-over frequency is defined as the frequency at which the phase angle of the open loop sinusoidal transfer function is 180°, M (jo does not drop below -3 dB is called bandwidth. aa You have either reached a page that is unavailable for viewing or reached your viewing limit for this book. aa You have either reached a page that is unavailable for viewing or reached your viewing limit for this book. aa You have either reached a page that is unavailable for viewing or reached your viewing limit for this book. aa You have either reached a page that is unavailable for viewing or reached your viewing limit for this book. aa You have either reached a page that is unavailable for viewing or reached your viewing limit for this book. aa You have either reached a page that is unavailable for viewing or reached your viewing limit for this book. aa You have either reached a page that is unavailable for viewing or reached your viewing limit for this book. aa You have either reached a page that is unavailable for viewing or reached your viewing limit for this book. aa You have either reached a page that is unavailable for viewing or reached your viewing limit for this book. aa You have either reached a page that is unavailable for viewing or reached your viewing limit for this book. aa You have either reached a page that is unavailable for viewing or reached your viewing limit for this book. aa You have either reached a page that is unavailable for viewing or reached your viewing limit for this book. aa You have either reached a page that is unavailable for viewing or reached your viewing limit for this book. aa You have either reached a page that is unavailable for viewing or reached your viewing limit for this book. aa You have either reached a page that is unavailable for viewing or reached your viewing limit for this book. aa You have either reached a page that is unavailable for viewing or reached your viewing limit for this book. aa You have either reached a page that is unavailable for viewing or reached your viewing limit for this book. aa You have either reached a page that is unavailable for viewing or reached your viewing limit for this book. aa You have either reached a page that is unavailable for viewing or reached your viewing limit for this book. aa You have either reached a page that is unavailable for viewing or reached your viewing limit for this book. aa You have either reached a page that is unavailable for viewing or reached your viewing limit for this book. aa You have either reached a page that is unavailable for viewing or reached your viewing limit for this book. aa You have either reached a page that is unavailable for viewing or reached your viewing limit for this book. aa You have either reached a page that is unavailable for viewing or reached your viewing limit for this book. aa You have either reached a page that is unavailable for viewing or reached your viewing limit for this book. aa You have either reached a page that is unavailable for viewing or reached your viewing limit for this book. aa You have either reached a page that is unavailable for viewing or reached your viewing limit for this book. aa You have either reached a page that is unavailable for viewing or reached your viewing limit for this book. aa You have either reached a page that is unavailable for viewing or reached your viewing limit for this book. aa You have either reached a page that is unavailable for viewing or reached your viewing limit for this book. aa You have either reached a page that is unavailable for viewing or reached your viewing limit for this book. aa You have either reached a page that is unavailable for viewing or reached your viewing limit for this book. aa You have either reached a page that is unavailable for viewing or reached your viewing limit for this book. aa You have either reached a page that is unavailable for viewing or reached your viewing limit for this book. aa You have either reached a page that is unavailable for viewing or reached your viewing limit for this book. aa You have either reached a page that is unavailable for viewing or reached your viewing limit for this book. aa You have either reached a page that is unavailable for viewing or reached your viewing limit for this book. aa You have either reached a page that is unavailable for viewing or reached your viewing limit for this book. aa You have either reached a page that is unavailable for viewing or reached your viewing limit for this book. aa You have either reached a page that is unavailable for viewing or reached your viewing limit for this book. aa You have either reached a page that is unavailable for viewing or reached your viewing limit for this book. aa You have either reached a page that is unavailable for viewing or reached your viewing limit for this book. aa You have either reached a page that is unavailable for viewing or reached your viewing limit for this book. aa You have either reached a page that is unavailable for viewing or reached your viewing limit for this book. aa You have either reached a page that is unavailable for viewing or reached your viewing limit for this book. aa You have either reached a page that is unavailable for viewing or reached your viewing limit for this book. aa You have either reached a page that is unavailable for viewing or reached your viewing limit for this book. aa You have either reached a page that is unavailable for viewing or reached your viewing limit for this book. aa You have either reached a page that is unavailable for viewing or reached your viewing limit for this book. aa You have either reached a page that is unavailable for viewing or reached your viewing limit for this book. aa You have either reached a page that is unavailable for viewing or reached your viewing limit for this book. aa You have either reached a page that is unavailable for viewing or reached your viewing limit for this book. aa You have either reached a page that is unavailable for viewing or reached your viewing limit for this book. aa You have either reached a page that is unavailable for viewing or reached your viewing limit for this book. aa You have either reached a page that is unavailable for viewing or reached your viewing limit for this book. aa You have either reached a page that is unavailable for viewing or reached your viewing limit for this book. aa You have either reached a page that is unavailable for viewing or reached your viewing limit for this book. aa You have either reached a page that is unavailable for viewing or reached your viewing limit for this book. aa You have either reached a page that is unavailable for viewing or reached your viewing limit for this book. aa You have either reached a page that is unavailable for viewing or reached your viewing limit for this book. aa You have either reached a page that is unavailable for viewing or reached your viewing limit for this book. aa You have either reached a page that is unavailable for viewing or reached your viewing limit for this book. aa You have either reached a page that is unavailable for viewing or reached your viewing limit for this book. aa You have either reached a page that is unavailable for viewing or reached your viewing limit for this book. aa You have either reached a page that is unavailable for viewing or reached your viewing limit for this book. aa You have either reached a page that is unavailable for viewing or reached your viewing limit for this book. aa You have either reached a page that is unavailable for viewing or reached your viewing limit for this book. aa You have either reached a page that is unavailable for viewing or reached your viewing limit for this book. aa You have either reached a page that is unavailable for viewing or reached your viewing limit for this book. aa You have either reached a page that is unavailable for viewing or reached your viewing limit for this book. aa You have either reached a page that is unavailable for viewing or reached your viewing limit for this book. aa You have either reached a page that is unavailable for viewing or reached your viewing limit for this book. aa You have either reached a page that is unavailable for viewing or reached your viewing limit for this book. aa You have either reached a page that is unavailable for viewing or reached your viewing limit for this book. aa You have either reached a page that is unavailable for viewing or reached your viewing limit for this book. aa You have either reached a page that is unavailable for viewing or reached your viewing limit for this book. aa You have either reached a page that is unavailable for viewing or reached your viewing limit for this book. aa You have either reached a page that is unavailable for viewing or reached your viewing limit for this book. aa You have either reached a page that is unavailable for viewing or reached your viewing limit for this book. aa You have either reached a page that is unavailable for viewing or reached your viewing limit for this book. aa You have either reached a page that is unavailable for viewing or reached your viewing limit for this book. aa You have either reached a page that is unavailable for viewing or reached your viewing limit for this book. aa You have either reached a page that is unavailable for viewing or reached your viewing limit for this book. aa You have either reached a page that is unavailable for viewing or reached your viewing limit for this book. aa You have either reached a page that is unavailable for viewing or reached your viewing limit for this book. aa You have either reached a page that is unavailable for viewing or reached your viewing limit for this book. aa You have either reached a page that is unavailable for viewing or reached your viewing limit for this book. aa You have either reached a page that is unavailable for viewing or reached your viewing limit for this book. aa You have either reached a page that is unavailable for viewing or reached your viewing limit for this book. aa You have either reached a page that is unavailable for viewing or reached your viewing limit for this book. aa You have either reached a page that is unavailable for viewing or reached your viewing limit for this book. aa You have either reached a page that is unavailable for viewing or reached your viewing limit for this book. aa You have either reached a page that is unavailable for viewing or reached your viewing limit for this book. aa You have either reached a page that is unavailable for viewing or reached your viewing limit for this book. aa You have either reached a page that is unavailable for viewing or reached your viewing limit for this book. aa You have either reached a page that is unavailable for viewing or reached your viewing limit for this book. aa You have either reached a page that is unavailable for viewing or reached your viewing limit for this book. aa You have either reached a page that is unavailable for viewing or reached your viewing limit for this book. aa You have either reached a page that is unavailable for viewing or reached your viewing limit for this book. aa You have either reached a page that is unavailable for viewing or reached your viewing limit for this book. aa You have either reached a page that is unavailable for viewing or reached your viewing limit for this book. aa You have either reached a page that is unavailable for viewing or reached your viewing limit for this book. aa You have either reached a page that is unavailable for viewing or reached your viewing limit for this book. aa You have either reached a page that is unavailable for viewing or reached your viewing limit for this book. aa You have either reached a page that is unavailable for viewing or reached your viewing limit for this book. aa You have either reached a page that is unavailable for viewing or reached your viewing limit for this book. aa You have either reached a page that is unavailable for viewing or reached your viewing limit for this book. aa You have either reached a page that is unavailable for viewing or reached your viewing limit for this book. aa You have either reached a page that is unavailable for viewing or reached your viewing limit for this book. aa You have either reached a page that is unavailable for viewing or reached your viewing limit for this book. aa You have either reached a page that is unavailable for viewing or reached your viewing limit for this book. aa You have either reached a page that is unavailable for viewing or reached your viewing limit for this book. aa You have either reached a page that is unavailable for viewing or reached your viewing limit for this book. aa You have either reached a page that is unavailable for viewing or reached your viewing limit for this book. aa You have either reached a page that is unavailable for viewing or reached your viewing limit for this book. aa You have either reached a page that is unavailable for viewing or reached your viewing limit for this book. aa You have either reached a page that is unavailable for viewing or reached your viewing limit for this book. aa You have either reached a page that is unavailable for viewing or reached your viewing limit for this book. aa You have either reached a page that is unavailable for viewing or reached your viewing limit for this book. aa You have either reached a page that is unavailable for viewing or reached your viewing limit for this book. aa You have either reached a page that is unavailable for viewing or reached your viewing limit for this book. aa You have either reached a page that is unavailable for viewing or reached your viewing limit for this book. aa You have either reached a page that is unavailable for viewing or reached your viewing limit for this book. aa You have either reached a page that is unavailable for viewing or reached your viewing limit for this book. aa You have either reached a page that is unavailable for viewing or reached your viewing limit for this book. aa You have either reached a page that is unavailable for viewing or reached your viewing limit for this book. aa You have either reached a page that is unavailable for viewing or reached your viewing limit for this book. aa You have either reached a page that is unavailable for viewing or reached your viewing limit for this book. aa You have either reached a page that is unavailable for viewing or reached your viewing limit for this book. aa You have either reached a page that is unavailable for viewing or reached your viewing limit for this book. aa You have either reached a page that is unavailable for viewing or reached your viewing limit for this book. aa You have either reached a page that is unavailable for viewing or reached your viewing limit for this book. aa You have either reached a page that is unavailable for viewing or reached your viewing limit for this book. aa You have either reached a page that is unavailable for viewing or reached your viewing limit for this book. aa You have either reached a page that is unavailable for viewing or reached your viewing limit for this book. aa You have either reached a page that is unavailable for viewing or reached your viewing limit for this book. aa You have either reached a page that is unavailable for viewing or reached your viewing limit for this book. aa You have either reached a page that is unavailable for viewing or reached your viewing limit for this book. aa You have either reached a page that is unavailable for viewing or reached your viewing limit for this book. aa You have either reached a page that is unavailable for viewing or reached your viewing limit for this book. aa You have either reached a page that is unavailable for viewing or reached your viewing limit for this book. aa You have either reached a page that is unavailable for viewing or reached your viewing limit for this book. aa You have either reached a page that is unavailable for viewing or reached your viewing limit for this book. aa You have either reached a page that is unavailable for viewing or reached your viewing limit for this book. aa You have either reached a page that is unavailable for viewing or reached your viewing limit for this book. aa You have either reached a page that is unavailable for viewing or reached your viewing limit for this book. aa You have either reached a page that is unavailable for viewing or reached your viewing limit for this book. aa You have either reached a page that is unavailable for viewing or reached your viewing limit for this book. 9.30 m Control Systems Engineering Example 9.9 Consider the forward path transfer function of a certain unity feedback system i 6G)=_—"__. (52,5410) Design a Pl controller, Solution . The system is to have a phase margin of 50° at a frequency of 10 rad/sec. 200) — tan! tan M2 alte) yo? +4)(a? +100) Gijlo) = 24 104x200 =139[ 2123.7 = 180° +50" +123.7 _ 6086.3" ~ 139 _ -@sin@ _ 100.1097 © |e(foyy 1.39 Hence Gels) -(ons+ 9) = 0.715 1 = 0.789 Example 9.10 A unity feedback control system has the following forward path transfer function 9 100 Gsy- 0 (s+ D+ 2)(s-+10) phase margin should be greater than 50° at a frequeney of 5 rad/sec. Design a PID Controller. Solution The steady state error for unit ramp input is less than 0.08 rad and the +K,s") 100 (K+ GGls)= ae 5 (s+ Dis +2\(s+10) The velocity error constant ' steady state erro Li sG,G(s) 5 0.0! K.= K,= 100 (5+ 5+ 2075+ 10) = jokin Satan—15/2—tan"!5/10 26x29 125 = 0.325| =173.5 = [80° +50° +173.5°= 43.5° GUS) = aa You have either reached a page that is unavailable for viewing or reached your viewing limit for this book. aa You have either reached a page that is unavailable for viewing or reached your viewing limit for this book. aa You have either reached a page that is unavailable for viewing or reached your viewing limit for this book. Copyrighted material aa You have either reached a page that is unavailable for viewing or reached your viewing limit for this book. aa You have either reached a page that is unavailable for viewing or reached your viewing limit for this book. aa You have either reached a page that is unavailable for viewing or reached your viewing limit for this book. Control System Components ml 10.13 (10.16) Also, (10.17) Usually, the step movement is in the range of 1.8°, 2.5°, 5.0°, 7.59, 10° ete. 26- For example if NV, =26.N, =24, then @ = 20— 49) 369° =1.15° 26x24 360 Suppose the machine is wound for 2 phase and N= 24, then the step movement == 7.5°, The resolution of the stepper motor is the number of steps taken for one complete revolution. Thus Resolution’ Number of steps _ 360° 10.18 Revoh 6 (10.18) ‘Thus, with 7.5? step movement there would be 36° - 4g steps per revolution. The rotor shafi speed is given by Ox { f (10.19) rps | 360 P| where fis the pulse rate of the stator input signal expressed in number of pulses per second. Example 10.1 A stepper motor has 10 poles in the armature and 8 teeth under each pole. The rotor teeth are 100. Calculate the step angle. Solution Stator teeth = 10 * 8 = 80 The step angle is given by equation (10.16) as, (A, = N, )360° _ (100-80) Nx N 10080, 360° = 0.9% Example 10.2 A stepper motor has a step angle of 1.8°, Find the resolution of the stepper motor. What is the speed of the stepper motor if the pulse frequency is 300 pulses per second? Determine the number of steps moved in 15 revolutions. Solution 360 Resolution = Ts 200 steps/revolution axs 1.8300 Speed of the stepper motor n= 2% ips = e me 360° 360 Number of steps moved in 15 revolutions + Resolution x Revolution = 200 x 15 =3000 steps rps = 1.5 rps =1.5x 60 = 90 rpm aa You have either reached a page that is unavailable for viewing or reached your viewing limit for this book. aa You have either reached a page that is unavailable for viewing or reached your viewing limit for this book. aa You have either reached a page that is unavailable for viewing or reached your viewing limit for this book. 10.18 m Control Systems Engineering @ 10.3.12 How the Rotor Moves? Now we clearly understand how the rotor moves. We imagine two axes on the rotor at right angle to each other, one along which permeance is maximum and the other along which the permeance is minimum. It is obvious that the first one coincides with the tooth axis and the second with the slot axis. If the whole rotor is polarised to S (Fig. 10.17) the rotor tends to occupy that position where the maximum permeance axis (teeth) coincides with the stator north pole and the minimum permeance axis (slot) with the stator south pole. In short. the permeance axis keep itself in synchronism with the stator mmf. However, the actual rotor speed is much less because the rotor movement by only one rotor slot pitch makes the permeance axes move through two stator pole pitches. Thus if f is the frequency of stator supply. 1 Speed of the rotor = > "PS The revolving stator mmf is produced by connecting the two phases of the stator windings to two phase supply in which case the rotor runs with synchronous speed given by the above expression and the motor is usually referred to as synchronous inductor motor. On the other hand, for the operation as a stepper motor, the two phases are to be energised with D.C. with periodic changes in polarity. We have two sequences of switching. one for clockwise and the other for counter-clockwise rotations of the rotor. The two sequences are given in Table 10.3. Table 10.3 cw cow 1\ | ty | Angte of Stator mmf Ay 1y | Angle of Stator mmf 1 1 0 ul Wl 0 i +90" 1 : 90° i t =180° 1 U 180° 1 ol 270° +1 1 270° ‘The construction of commercially available stepper motor slightly differs in the winding connection of the stator. The schematic diagram of a two phase four stator winding (eight poles) stepper motor is shown in Fig, 10.17(a). This wiring technique is known as the four-step switching sequence, The nature of waveform of the stator supply is shown in Fig. 10.17(b). This is realised by the battery E and switches SW, and SW, in Fig. 10.17%(a). The windings A, and Aj belong to A phase and windings B, and Bj belongto B phase. Windings A, and A, are not energised at the same time since they create fields that are equal and opposite. The switches SW, and SW, are operated manually or by control logic circuit to generate waveform as shown in Fig. 10.17(b). Whenever there is a change from +E to -E or vice versa in any one phase as indicated in the figure. the stepper motor moves through one step. The switching sequence for CW and CCW is shown in the Table 10.4. aa You have either reached a page that is unavailable for viewing or reached your viewing limit for this book. aa You have either reached a page that is unavailable for viewing or reached your viewing limit for this book. aa You have either reached a page that is unavailable for viewing or reached your viewing limit for this book. aa You have either reached a page that is unavailable for viewing or reached your viewing limit for this book. aa You have either reached a page that is unavailable for viewing or reached your viewing limit for this book. A A.C. Motors 10.8 Absolute Stability 7.1 Aceeleration Error Constant 5.66, 6.91 Acceleration Input 5.67 Acceleration Signal 5.10 A.C. Servomotor 10.1 A.C, Stepper Motors 10.1 AC. Tachogenerator 10.23 Actuator 1.14 Addition of a Pole 5.81 Addition of a Zero 5.82 Additivity 1.13 All Pass Transfer Function 6.10, 6.89 Amplidyne 1.15 Angles of Arrival 8.6 Angles of Departure 8.6 Angular Displacement 2.39 Ant Colony 1.2 Armature Controlled D.C. Motor 2.27, 10.8 Asymptote of the Root Loci 8.6 Asymptotic Stability 7.3 Automatic Control Systems 1.1 B Bandwidth 1.12, 6.50, 6.85, 9.34 BELL Telephone Systems 7.3! BIBO 7.1 BIBO Stable 7.7 Biological System 1.8 Block Diagram 2.1,4.2 Bode Magnitude Plot 6.41 Bode Phase Plot. 6.58 Bode Plot 6.2, 6.20 Break in Point 8.29 Breakaway Points 8.7 Buffer Amplifier 2.24 © Capacitance 4.70 Cascade Connection 4.2 Centrifugal Governor 1.1 Index Centroid 8.6 Characteristic Equation 5.101, 7.6 Characteristic Polynomial 5.62, 7.6 Classical Control Theory 1.2 Closed Loop Regulator System 1.9 Closed Loop System [.5 Computer Control System 1.11 Conditionally Stable System 5.98 Conformal Mappings 7.24 Constant M Circle 6.69, 6.83 Constant N Circles 6.71 Contour in the s-plane 7.25 Control Action 1.6 Control Systems 1.3 Control Transformer 10.4 Control Valve 1.12 Controller 5.1 Comer Frequencies 6.24, 6.86 Critically Damped 5.24. 5.32 Cut-off-rate 6.51 D D'Alembert’s Principle 3.4 Damped Frequency 5.100 Damping Factor 5.97, 6.18 Damping Ratio 5.28 Dash-pot 2.3 D.C. Generators 2.27 D.C. Motor 1.10, 2.40, 10.8 D.C. Servomotor 2.40 D.C. Tachogenerator 10.22 Delay Time 5.98 Denominator Polynomial 5.62 Derivative Feedback 5.87, 5.101 Deterministic Systems 1.14 Discrete Signals 1.14 Discrete Systems 1.14 Displacement 2.4 Distributed Parameter Systems 1.14 Dominant Roots 7.2 Drag Cup Rotor 10.8 Dynamic Error Constants 5.73 * INX.2 m™ Control Systems Engineering E Electrical Analogous Circuits. 3.1 Electrical Network 2.21 Electrical System 3.4 Electro-mechanical System 2.18 Electronic Amplifiers 10.1 Esror 5.2 Emror Detector 1.6, 10.28 Error Signal 1.7 F Feed-forward Compensator 9.1 Feedback Compensator 9.1 Feedback Connection 4.3, 4.31 Feedback Control Theory 1.2 Feedback Controller 9.26 Feedback Element 16 Feedback Ratio 1.15 Feedback Transducer 10.1 id Comtrolled D.C. Motor 2.29, 10.8 Final Displacement 2.6 al Value 5.100 1 Value Theorem 5.12 Final Velocity 2.6 t Maximum Overshoot 5.27 1 Order System 5.20 Force-Current Analogy 3.2 Force-Voltage Analogy 3.1 Forced Sinusoidal Response 6.80 Forward Path 4.30 Forward Path Gain 5.2 Free Body Diagram 2.3 Frequency Domain 1.2 Frequency Response Method 6.1 Frequency Response Plot 6.86 Friction Coefficient 5.2 Fuzzy Logic 1.2 G Gain Adjustment 6.75 Gain Crossover Frequency 6.49 Gain Margin 6.83, 8.16 Gear Arrangement 2.18 Gear Ratio 5. Gear Trains 10.21 Generalized or Dynamic Error Coefficients 5.70 Generator 1.15 Genetic Algorithm 1.2 Graphical Method 5.96 Gun Positioning Systems 1.2 H Homogeneity 1.13 Hybrid Mechanical System 2.35 Hybrid Stepper Motor (HSM) 10.12 Hydraulic Amplifier 10.1 Hydraulic Servomoter 10.20 I Impulse Response 5.32 Impulse Response Function 7.4 Impulse Signal 5.8 Initial Displacement 2.6 Value Theorem 5.12 Initial Velocity 2.6 Input 1.4 Input Node 4.30 Input Variable 2.2 Intelligent Controllers. 1.2 Interaction of Blocks 4.29 Intermittent Variable 2.22 Inverse Root Locus 8.58 K Kinetic Energy 2.3 Kirchhoff’s Current Law 3.4 Kirchhof?’s Voltage Law 2.21, 3.4 L Ladder Network 4.69 Lag Compensator 9.9 Lag-Lead Compensator 9.16 Laplace Transform 2.2,5.11 Lead Compensator 9.2 Left Half of s-Plane (LHP) 7.3 Linear System 1.13, 5.98 Liquid Bulk Modulus 2.42 Load Compensators 9.35 Load Torque 5.2 Long Division 7.11 Loop 4.30 LoopGain 4.30,5.7 Loop System 1.43.2 Lumped Parameter 1.14 M Magnetic Amplifiers. 10.1 Magnitude 6.86 Magnitude Criterion 8.67 Magnitude Plot 6.58 Marginal Stability 7.3 Marginal Value 7.17 Mason's Gain Formula 4.32 Mathematical Modeling 2.1 Maximum Overshoot 5.98 Maximum Peak Value 5.100. Maximum Power Transfer 2.17, Mechanical Systems 2.2 MIMO 1.14 Minimum Phase Transfer Function 6.10 fixed Node 4.30,4.71 Moment of Inertia 5.2 Muitiple Input System 4.19 N Natural Frequency 5.97 Negative Feedback 1.5 Neural Network 1.2 3 Newton’s Law 2. Nichol’s Chart 6. Nodal System 3.2 Non-interaction of Blocks 4.71 Non-linear System 1.13 Non-minimum Phase Transfer Funciton 6.13 Non-reciprocity 4.71 Non-touching Loops 4.30 ‘Number of Encirclement 7.33 Number of Oscillations 5.101 Nyquist Contour 7.32 Nyquist Plot 7.2.7.37 Nyquist Stability Criterion 7.31 oO On-Off Control System 5.101 On-Off or Two Position Contioller 5.89 Open Loop System 14 Order of the System 5.3 Oscillatory System 7.10 Output 14 Output Node 4.30 Output Variable 2.2 Overdamped 5.25 Overshoot 5:29 Index ml INX.3 P Pair of Synchros 10.1 Parabolic Signal 5.10 Parallel Connection 4.2, 4.31 Partial Differential Equations 1.14 Partial Fraction Expansion 5.10 Particle Swarm 1.2 Path 4.30 Path Gain 4.30 , PD Controller 5.83, 9.31 Peak Overshoot 5.31 Peak Time 5.97 Percentage Maximum Overshoot 5.28 Period of Oscillation 5.31 Period of Oscillation 5.101 Permanent Magnet Stepping Motor 10.15 Phase 6.1, 6.86 Phase Angle 6.18 Phase Angie Criterion 8.66 Phase Angle Plot 6.36 Phase Crossover Frequency 6.48 Phase Margin 6.56, 8.16 Phase on Mode Operation 10.15 Phase Shift Plot 6.86 Pi Controller 5.84, 9.35 PID Controller 5.1, 9.32 Pneumatic Amplifier 10.1 Pneumatic Controllers 10.20 Polar Plot 6.2 Polarised Solenoid 2.18 Poles 5.11, 5.101 Pole Location 5.96 Position Control System 10.5 Position Error Constant 6.90 Positional Error Coefficient 5.63, 5.66 Positive Feedback System 1.5 Potential Energy 2.3 Potentiometer 1.4, 10.6. 10.29 Pressure Control System 1.11 Pressure Transmitter (PT) 1.11 Principte of Arguments 7.24 Proportional Error 5.38 Pure Integrators 5.62 R Radar Antenna 1,10 Ramp Input 5.4 Ramp Signal 5.9 Rate Controller 5.101 INX.4 m Control Systems Engineering Rate or Tachogenerator Feedback Control 5.85 Reciprocity 4.71 Regulator System 1.4 Relative Stability 7.2 Reset Controller 5.101 Residues 5.13 Resonant Frequency 6.50 Resonant Peak 6.50 Right Half s-Plane (RHP) 7.12 Rise Time 5.31 Root Locus 8.68 Root Locus Technique ‘8.1 Rotating Amplifiers 10.1 Rotational System 2.3 Rotor Moverment 10.15 Routh’s Array 7.8 Routh's Stability Criterion 7.8 s Second Order System 2.2, 5.22 Selsyns 10.29 Sensitivity 5.92, 5.101 Separately Excited D.C. Generator 2.26 ies Compensator 9.1 Servo Amplifier 10.1. 10.30 Servomechanism 1.4 Servomotor 10.1 Settling Time 5.22 Shift Theorem 5.13 Signal Frow Graph 2.1, 4.30 Sink Node 4.70 Sinusoidal Input 6.1 Solenoid 2.38 Solid tron Rotor 10.8 Spring 2.3 Squirrel Cage Rotor 10.8 Stability Condition 7.6 State Error 5.62 State Space Model 1.2 Static Error Coefficients 5.62 Static Error Constants 5.98 Steady State Error 5.3, 5.62 Steady State Response 5.1 Step Input 5.3. 5.100 Step Response 5 Step Signal 5.9 Stepper Motor 10.8, 10.29 Stochastic Systems 1.14 Superposition 1.13 Synchro Generator 10.2 ‘Synchro Transmitter 10.29 Synehros 10.2 System 1.3, 1.13 System Modeling 2.40 System Parameters 1.4 System Representation 1.4 System with Time Delay 7.80 T Tachogenerator 1.4, 10.22 Tachometer Feedback 9.26 ‘Temperature Control System 1.11 Test Signals 5.8 Time Invariant System 1:14 Time Constant 7° 5.29 Time Delay #, 5.29 ‘Time Domain 1.2 Time Invariant 7.4 Time Varying System 1.14 ‘Torque-Speed Characteristics 10.39 Transducer 1.7, 10.30 ‘Transfer Function 2.2, 2.7 ‘Transient Response 5.1. 5.99 ‘Translational Systems 2.3 Transmittance 4.69 ‘Transportation Lag 6.94, 7.23 ‘Two Phase Servomotor 10.8 ‘Two Stack Reluctance Type Stepper Motor 10.17 ‘Type 5.3, 5.99, 6.6 ‘Type and Order 5.62 U Underdamped 5.23, 5.32 Unit Step Input 5.27 v Variable Reluctance (V.R.) Stepper Motor 10.12 Variables 1.4 Velocity 2.4, 5.67 Velocity Error Constant 5.66 ZL Zero Displacement 2.7 Zero Initial Conditions. 2.1 Zero Input Stability 7.3 Zeros 5.11,5.101 a EDITIO At 7 | Control Engineering is a multi-disciplinary subject and finds widespread application in the guidance, navigation and control of missiles, spacecrafts, aeroplanes and ships, as well as in the pro control industry. This book presents clear theoretical concepts reinforced by worked out numerical | examples. It includes topics on Nyquist Stability Criterion, Signal Flow Graph, RootLocus Technique Pe eu Lenco cS cure eerie > Comprehensive treatment on System Modelling, Block Diagram and Signal PMC eC a mac em emu mel ETE eae Rev ole MRS Cees unl enn ul en » Linear System stability by Routh-Hunwitz and Nyqusit criterion are-illustrated STENNIS Provides astep-by-step approach to solving problems Pam OHIO L CIMA SS MUNSHI | > Pedagogical features include DAUR C us Yi) Rack res mete Rel cea ca ena Prreiercd + Summary atthe end of each chapter for quick reference. | BetUS cece) Iu) Mv || ! ‘fay Higher Education JUN,

You might also like